You are on page 1of 365

FINAL C.A.

– AUDIT

PROFESSIONAL ETHICS

I – INTRODUCTION :
Do’s and Don’ts for CA

Code of conduct for member of the institute based upon

CA Act, 1949 CA Regulations Council General Decision of ESB


1988 Guidelines, 2008

BASIC CONCEPTS

1. Who is a member of the Institute?

Cleared Final Exam+


Completed prescribed training

Name in Register of Member


2. Who are disqualified from having name in ROM?
( MR. ICU )

M – Minor ( < 21 years )


R- Removed by ICAI due to misconduct
I- Insolvent
C-Convicted by court
U- Unsound mind

3. STATUS AFTER BECOMING CA


Practice In service Other

1
FINAL C.A. – AUDIT

Who is deemed to be in Practice?


(HARAM)
H- Holds himself in practice of accountancy
A- Engages himself in practice of accountancy
R-Renders professional service or assistance in accounting procedure
A-Performs Audit/ other related services
M- Renders Management consultancy services

4. TYPES OF MISCONDUCT

Professional Misconduct Other Misconduct

FIRST SCHEDULE SECOND / BOTH SCHEDULE


Place the matter before Place matter before
Board of Discipline Disciplinary / Committee
If found guilty, it can If fund guilty, it can
s Reprimand member s Reprimand member
s Remove the name of member upto a s Remove the name of member permanently
period of 3 months or for any duration
s Impose fine upto 1,00,000 s Impose fine upto 5,00,000

II – SCHEDULES OF CA Act, 1949:


FIRST SCHEDULE SECOND SCHEDULE

PART I : PART I :
Professional misconduct in relation Professional misconduct in relation
to membersi n PRACTICE (12 to members in PRACTICE (10
Clause) Clause)

PART II :
Professional misconduct in relation
to members in SERVICE (2
x
Clauses)

2
FINAL C.A. – AUDIT

II – SCHEDULES OF CA Act, 1949:


FIRST SCHEDULE SECOND SCHEDULE

PART III : PART III :


Professional misconduct in Professional misconduct in
relation to members GENERALLY relation to members GENERALLY
(3 Clauses) (4 Clauses)

PART IV : PART IV :
Other misconduct in relation Other misconduct in relation
to members GENERALLY (2 to members GENERALLY (1
Clauses) CLAUSE)

III – FORMAT OF ANSWER :


As per clause .......................... of Part ............................schedule of charted Accountants Act,
1949 a CA in ...................................... is deemed to be guilty of ..................................... misconduct
if he............................
In this case , ......................................................................................................................................
....................................................................................................................................................
..Conclusion.

IV – SCHEDULES TO THE ACT :


FIRST SCHEDULE :
PART I : PROFESSIONAL MISCONDUCT IN
RELATION TO MEMBERS IN PRACTICE

CLAUSE 1 : s Allows any person to practice in his name as a CA,


s Unless such a person is also a CA in practice, and
s Is in partnership with, or
s Employed by himself.

3
FINAL C.A. – AUDIT

IV – SCHEDULES TO THE ACT :


FIRST SCHEDULE :
PART I : PROFESSIONAL MISCONDUCT IN
RELATION TO MEMBERS IN PRACTICE

CLAUSE 1 : OFU
Non – CA (X)
CA (Not in Practice) (X)
CA in Practice – Employed (X)
CA in Practice – Partner ( )
CA in Practice – neither ( )
Partner nor employed (X)

QUESTIONS ASKED IN PAST/RTP/MOCK TEST


Question 1:
CA Sant, a newly qualified professional with certificate of practice, approached CA Pant,
the auditor of his father’s company M/s Max Ltd., to allow him to have some practical and
professional knowledge and experience in his firm before he can set up his own professional
practice. CA Pant allowed him to sit in his office for 6 months and allotted a .small chamber
with other office infrastructure facility. In the course of his association with CA Pant’s office,
he used to provide tax consultancy independently to the client of the firm and also filed few IT
and GST return and represented himself before various tax authorities on behalf of the firm
although no documents were signed by him. During his association an CA Pant’s office he did
not get any salary or share of profit or commission but only reimbursement of usual expenses
like conveyance, telephone etc. was made to him. After the end of the agreed period, he was
given a lump stun amount of Rs. 3,00,000 by CA Pant for his association out of gratitude

Answer:
Mr. Pant will be guilty of professional misconduct by Virtue of clause 1 of Part I of First
Schedule as he allows another person to practice in his firm name, whereas other person (Mr.
Sant) is neither a partner nor an employee.

CLAUSE 2 :
v Pay or allows or agrees to pay or allow,
v Directly or Indirectly,

4
FINAL C.A. – AUDIT

v Any share, commission or brokerage in


v Fees or Profits of his professional business
v to any person other than
s A member of the institute, or
s Partner, or
s Retired partner, or
s Legal representative of deceased partner, or
s Member of any other professional bodies*, or

v With such other persons having


prescribed qualifications**
v For the purpose of rendering such
professional services from time to time
in or outside India.

v Professional bodies [Regulation 53A] :
ICSI, ICWAI, Bar council of India, India Institute
of Architects, institute of Actuaries of India.

v Person with prescribed qualifications.


[Regulation 53A]
CS, CWA, Actuary, B.E. Bachelor in Technology, Bachelor in
Architecture, Bachelor in Law and MBA.

EXCEPTION 1:
Sharing of profits by widow or legal representatives of deceased partner.
Provided agreement provides the same.

EXCEPTION 2:
Sharing % of Fees with the Govt. – Audit of Co-operative Societies

5
FINAL C.A. – AUDIT

EXCEPTION 3:
SALE OF GOODWILL IN CASE OF DEATH OF SOLE PROPRIETOR

NORMAL CASE DISPUTED CASE


l ICAI’s permission to l In case of dispute as to
practice in firm name should legal heir of the deceased
be taken within 1 year of proprietor.
death. l Intimation of dispute to
l Name of firm is kept in be received by ICAI with
abeyance till 1 year from date 1 year of death.
of death. l ICAI will keep the name
Note : PURCHASER CA in of firm in abeyance till year
practice from settlement of dispute.
CONSIDERATION No
future share + perm of council

Questions asked in past/rtp/mock test


Question 2:
A CA obtained a loan from a finance company for purchase of office building agreeing to pay
interest at 6% p.a. and 10% of his gross professional receipts till the loan is repaid

Answer:
Professional misconduct by virtue of Clause 2, Part 1 of First Schedule.

Question 3:
K, a practicing Chartered Accountant gave 50% of the audit fees received by him to L, who
was not a Chartered Accountant, under the nomenclature of office allowance and such an
arrangement continued for a number of years.

Answer:
Mr. K will be deemed to be guilty of professional misconduct under Clause 2 of Part I of First
Schedule

6
FINAL C.A. – AUDIT

Question 4: (FAQ)
l Mr. X who passed his CA examination of ICAI on 18th July, 2018 and started his
practice from August 15, 2018.
l On 16th August 2018, one female candidate approached him for articleship.

l In addition to monthly stipend, Mr. X also offered her 1% profits of his CA firm.

l She agreed to take both 1% profits of the CA firm and stipend as per the rate prescribed

by the ICAI.
l The Institute of Chartered Accountants of India sent a letter to Mr. X objecting the

payment of 1% profits.
l Mr. X replies to the ICAI stating that he is paying 1% profits of his firm over and above

the stipend to help the articled clerk as the financial position of the articled clerk is very
weak.
l Is Mr. X liable to professional misconduct

Answer:
Mr. X will be guilty of professional misconduct in terms of Clause 2 of Part I-of First Schedule
and reply of Mr. X that he is paying 1% profits over and above the stipend to help the article
clerk is not tenable.

Question 5:
A-Chartered Accountant who was in practice since last 20 years died in a road accident. His
widow sold the practice to another Chartered Accountant in practice for 30 lakhs. The price
also included the sight to use the firm name

Answer:
Widow of deceased member can sell the goodwill provided sale is completed within one year from
the death of member and permission of ICAI is obtained

CLAUSE 3 : l Accepts or agrees to accept


l Any part of the profit of the professional work
l Of a person who is not a member of the institute.

EXCEPTIONS :
l Member of any other professional bodies [as per clause 2]

l Person having prescribed qualifications [as per clause 2]

7
FINAL C.A. – AUDIT

Question 6:
Mr. X is a practicing Chartered Accountant. Mr. Y is a practicing Advocate representing
matters in the court of Law. X and Y decided to help each other in the matters involving their
professional expertise, Accordingly, Mr. X recommends Mr. Y in all litigation matters in the
court of law and Y consults X in all matters relating to finance and other related matters,
which comes to him in arguing various cases. Consequently they started sharing profits of
their professional work. Is Mr, X liable for professional misconduct?

Answer:
Mr. X will not be deemed to be guilty of professional misconduct as Clauses 2 & 3 permits a
CA in practice for profit sharing with members of any other professional bodies or with such
other persons having prescribed qualifications and advocates are prescribed under Regulation
53A.

8
FINAL C.A. – AUDIT

CLAUSE 4 :
Enter into partnership in or outside India.
With any person other than
l CA in practice
l Member of any other professional body having prescribed qualifications, or

l A person, who but his residence abroad would be entitled to be registered as member

under clause (V) of Sec 4(1) or


l A person whose qualifications are recognized by CG or council for the purpose of

permitting such partnerships.

Questions asked in past/rtp/mock test


Question 7:
Mr. P, a Chartered Accountant in practice entered into a partnership with Mr. L, an advocate
for sharing of fees for work sent by one to the other. However, due to some disputes, the
partnership was dissolved after 1 month without any fees haying been received.
Answer:
Mr. P will not be deemed to be guilty of professional misconduct as clause 4 permits a CA in
practice for entering into partnership with members of any other professional bodies or with
such other persons having prescribed qualifications and members of bar council of India and
persons having Bachelor of Law are prescribed.

CLAUSE 5 :
Secures any professional business
Through the service of a ‘Person’ who is not an employee or not his partner or
By means which are not open to a CA
Nothing contained in clause 5 shall be construed as prohibiting any arrangement
permitted in clauses (2), (3) and (4)

CLAUSE 6 :
Solicits clients or professional work
Either directly or indirectly
By circular, advertisement, personal communication or interview or by any other means.
Provided that nothing here in contained shall be construed as preventing or prohibiting.
(i) Any CA from applying or requesting for or inviting or securing professional work
from another* CA in practice, or

9
FINAL C.A. – AUDIT

(ii) A Member from responding to tenders** or enquiries issued by various user of


professional service or organization from time to time and securing
professional work as a consequence.

v Requesting another CA in practice to provide work :


Should not claim superiority over another firm.
Should not disclose name of clients in write up.

v Submitting Tenders :

Invitation from No invitation


client from client.

No Allowed
Area exclusively Area open for other
reserved for CAS professionals as well

Only if minimum fees Submitting tender


is quoted allowed

COUNCIL GUIDELINES W.R.T PERMITTED AND PROHIBITED FORMS OF


SOLICITATION :

[HA2TE C2OP2I]

H Issuing hand bills


l No hand bills to persons other than client in matters such as change in tax laws.

A1 Advertisement and note in press:


l Change in partnership, address and telephone (Only bare statement of facts and
no. of insertions should be Limited]
l Classified adv. In journal / news letter of institute for sharing professional work

or seeking partnership or salaried employment (it contains CA’s name, address,


telephone No. Fax or Email Address)

10
FINAL C.A. – AUDIT

A1 Advertisement Under Box Numbers (X)

T1 Publication of name or firm name in telephone / other directories.


l In section ‘Charted Accountant’
l In City / Town to which member belongs

l Bold / Abnormal type letter or in a box

l Order of entries Alphabetically and logical


l Payment Not unreasonable
l Open to all CA in City / Town

l Should not give impression of Publicity or advertisement.

E Empanelment for allotment of audit / professional work.


l Free to write to the concerned organization with a request to place his name on the

panel.
l Cannot make roving enquiries about existence of the panel.

l Allowed to quote fees on enquiries being received from such bodies.

l Printed or Xerox copies of scale of fees in reply to such enquiries is not permitted.

C 140(4) of companies Act, 2013


l Wordings of his representation

No Publicity
Does not solicit for his continuance.

C2 Advertisement for coaching activities.


l Advertising of their association with coaching / teaching activities through
hoardings, posters, banners and by any other means. (X)
l May put outside their coaching premises, sign board mentioning the name of

coaching / teaching institute contact details and subjects taught there in only.
()

O Acceptance of original professional work :


l The council has decided that a member should not accept the original professional

work emanating from a client introduced to him by another member.


l If any professional work of such client comes to him directly, it should be his duly

to ask the client that he should come through the other member dealing generally
with his original work.

11
FINAL C.A. – AUDIT

P Publication of books or Articles


l Indicate in a book or an article, published by him, the association with any firm of
CA. ()
l Mention nature of services and other designations. ()

l Highlight academic achievements and degrees. ()

P Public Interview
l Result in publicity ()
l Handover his profile & ask the presenter to read profile. ()

l Highlight academic achievements and degrees. ()

P Issue of greeting cards or invitations


l Designation ‘CA’ and name of firm ()
l Cards sent only to clients, relatives and close friends ()

Questions asked in past/rtp/mock test


Question 8:
Z, a. Chartered Accountant wrote several letters to Government Department, pointing out
seniority of his firm, sending his life sketch and stating that he had a glorious record of
service to the country as well as to the organisation of accountancy profession with a view to
get the audit work.
Answer:
Guilty of professional misconduct by virtue of Clause 6 of Part I of First Schedule which
prohibits., member in practice to solicits professional work by means of advertisement, circular,
personal - communication or roving enquiries, etc.

Question 9:
Mr. S, a Chartered Accountant published a book and gave his personal details as the author.
These details also mentioned his professional experience and his present association as partner
with M/s RST, a firm.
Answer:
Mr. S has violated the restriction imposed under Clause 6 of Part I of First Schedule and
hence held to be guilty of professional misconduct

12
FINAL C.A. – AUDIT

Question 10:
M/s LMN, a firm of Chartered Accountants, responded lo a tender from a State Government
for (computerization of land revenue records. For this purpose, the firm also paid Rs.50,000
as earnest deposit, as part of the terms of the tender.
Answer:
In the present case, since computerisation of property records does not fall within exclusive
areas for CAs, M/s. LMN can respond to tenders as well as deposit Rs.50,000 as earnest
deposit and shall not have committed any professional misconduct.

Question 11:
CA. S and CA. M are two partners of the CA firm ‘SM & Co. Being very pious, CA. S organised
a. religious ceremony at his home for which he instructed his printing agent to add his
designation “Chartered Accountant” with his name in the invitation cards. Later on, the
invitations were distributed to only the relatives, close friends and clients of both the partners
Answer:
Using the designation “Chartered Accountant” in invitation cards is permitted under Guidelines-
issued in Clause 6 of Part 1 of First Schedule provided cards are distributed to relatives, close
friends and clients of the concerned member. In the present: case, invitations are also given to
relatives, close friends and clients of partner also; hence misconduct arises on part of CA. S.

Question 12:
A special notice has been issued for a resolution at AGM of F Ltd. providing expressly that
CA. S shall not be re-appointed as an auditor of the company. Consequently, CA. S submitted
a representation in writing to the company, as provided under section 140(4) (iii) of the
Companies Act, 2013 .in the, representation, CA. S incorporated his independent working as
a professional throughout the term of office and also indicated his willingness to continue as
on auditor if reappointed by the shareholders of the Company.
Answer:
There is no misconduct on Part of CA. S as guidelines issued under Clause 6 of Part I of First-
Schedule allows a CA in practice to make the representation u/s 140(4) of Companies Act,
2013 which is not used by the auditor to secure needless publicity.

Question 13:
CA. N, in practice, started project consultancy work as a part of his practice and to advance
the same,, sent email to all the CAs in the country Informing them of his services and for
securing professional work.

13
FINAL C.A. – AUDIT

Answer:
There is no misconduct on Part of CA. N as exceptions to Clause 6 of Part 1 of First Schedule
allows a CA in practice to secure professional work from another CA in practice.

Question 14:
M/s LMN & Associates, a firm off Chartered Accountants responded to a tender issued
exclusively for Chartered Accountants by an organisation in the area of tax audit. However,
no minimum fee was prescribed in the tender document.
Answer:
LMN & Associates shall be guilty of professional misconduct as responding to tenders in
exclusive area in which minimum fee, is not pre scribed amounts to solicitation of work.

Question 15:
An advertisement was published in a Newspaper containing the photograph of Mr. X, a
member of the institute wherein he was congratulated on the occasion of the opening ceremony
of his office.
Answer:
Mr. X would be guilty under Clause 6 of Part I of the First Schedule to the Chartered
Accountants Act, 1949 as advertisement m the newspaper amount to solicitation

Question 16:
Mr. X, a Chartered Accountant and the proprietor of X & Co., wrote several letters to the
Assistant Registrar of Co-operative Societies stating that though his firm was on the panel
of auditors, no audit work was allotted to the firm and further requested him to look into the
matter.
Answer:
Mr. X would be held guilty under clause 6 of Part I of the Act, as roving inquiries for solicitation
of work is not allowed.

Question 17:
Mr. Sodhi, a Chartered Accountant in practice, who is proposed to be removed as the auditor
of a company makes unsubstantiated and derogatory remarks against the management of the
company in his representation u/s 140 of the Companies Act, 2013.
Answer:
In the present case, Mr. Sodhi if guilty of professional misconduct as he makes derogatory
‘remarks against the management of the company

14
FINAL C.A. – AUDIT

Question 18:
A letter is sent by a Chartered Accountant in practice to the Ministry of Finance inquiring
whether a panel of auditors is being maintained by the Ministry and if so to include his name
in the panel
Answer:
The member is guilty of nil conduct in terms of the Clause 6, Part 1 of First Schedule as he
has solicited professional work by making roving inquiries as to existence of panel.

Question 19:
M, a practicing Chartered Accountant sent a letter to another firm of Chartered Accountants,
claiming himself to be a pioneer in liaisoning with Central Government Ministries and its allied
Departments for getting various Government clearances for which he had claimed to have
expertise and had given a list of his existing clients and details of his staff etc.
Answer:
Mr. M was guilty of professional misconduct as per clause 6 of part I of First Schedule of the
Chartered Accountants Act, 1949

Question 20:
Mr. Honest, a Chartered Accountant in practice, wrote two letters to M/s XY Chartered
Accountants a firm of CA’s; requesting them to allot him some professional work. As he
did not have a significant practice or clients he also wrote a letter to M/s ABC, a firm of
Chartered Accountants for securing professional work. Mr. Clever, an another CA, informed
ICA1 regarding Mr. Honest’s approach to secure the professional work. Is Mr Honest wrong
in soliciting professional work?
Answer:
Mr. Honest is not wrong in soliciting professional work from another CA, hence there is no
professional misconduct.

Question 21:
During the opening ceremony of a new branch office of CA. Young, his friend CA, Old introduced
to CA. Young, his friend and client Mr. Rich, the owner of an Export House whose accounts
had been audited by CA. Old for more than 15 years. After few days, Mr. Rich approached
CA. Young and offered a certification work which hitherto had been done by CA. Old CA. Young
undertook the work for a fee which was not less than fee charged by CA. Old in earlier period.
Comment whether CA. Young had done any professional misconduct

15
FINAL C.A. – AUDIT

Answer:
Acceptance of original professional work emanating from a client introduced by another
member is not permitted. Mr. Young will be deemed guilty of professional misconduct under
clause 6, Part. 1 of First Schedule

16
FINAL C.A. – AUDIT

Website:

PERMITTED FEATURES :

1) CA / CA firm are free to create website.


2) It can be in any format, colour.
3) CA can mention website address on professional stationery.
4) Website should run on PULL MODE and not on push mode.

PERMITTED INFORMATION :
1) Name of member / firm
2) Member / Firms Address / Telephone No. / Fax / Email Id (Branch as well as Ho’s)
3) Partner’s name and their qualifications, year of qualification Home Address, Telephone
No. E-mail ID. (i.e Biodata of Partner)
4) Employee’s name and their qualification
5) Job vacancies including article ship
6) Articles etc. of professional int. such as budget highlights.
7) Bulletin board
8) Date upto which website is updated.
9) Chat room between client and CA or among CA’s
However confidentially should be maintained.

PROHIBITED INFORMATION : [POLAR]

P No Photograph other than passport size photo

O No reference of any other website other than :


ICAI
Its regional councils and branches
Government/ Government Department / Regulatory Authories
Other professional bodies, such as, American institute of certified public Accountants
[AICPA], the institute of chartered Accountants of England and Wales [ICAEW]
and the Canadian institute of chartered Accountants. [CICA]

L Logo other than that prescribed by ICAI.

17
FINAL C.A. – AUDIT

A No Advertisement

R Reference of clients and fee charged.

Note : As per amendment in August 2015, disclosure of names of clients and / or fees
charged, on the website is permissible only where it is required by a regular, whether
or not constituted under a statute, in India or outside India, provided that such
disclosure is only to the extent of requirement of the regulator.

OTHER POINTS :
1. SPECIFIC PULL REQUEST
Nature of services rendered
Nature of assignments handled
Area of expertise of partners and employees
No. of articled clerks
Year of establishment.

2. Address of website :
In name of CA / CA firm. It maybe different from firm / CA name but should be as
near as possible to their name.
Address should not result in soliciting client.

3. SEARCH ENGINE :
Allowed, but it should be non criteria such as CA, Indian CPA.

4. INTIMATION TO ICAI :
CA’s required to information website address to ICAI, while submitting annual
membership fee and form. (As per old provision, it was required to intimate ICAI
within 30 days of setting up website. )

18
FINAL C.A. – AUDIT

Questions asked in past/rtp/mock test

Question 22:
M/s XYZ, a firm of Chartered Accountants created a website “www.xyzindia.com”. The
website besides containing details of the firm and bio-data of the partners also contains the
photographs of all the partners of the firm.
Answer:
M/s XYZ, had complied with all the guidelines and there does not appear any violation of the
Chartered Accountants Act, 1949 and its Regulations.

Question 23:
XYZ & Associates, a firm with 5 partners developed a website www.xyzassociates.com. The
website also contained a link to “All India Chartered Accountants Association”, a voluntary
association where X, a partner of the firm is currently the Vice-president
Answer:
As per guidelines issued by ICAI for development and contents of website, a website may
provide a link to website of ICAI, its regional councils, branches and government departments
and other professional bodies like AICPA, ICAEVV etc. In the present case, website provide a
link to the “All India Chartered Accountants Association” which is not permitted. Hence the
firm is guilty of professional misconduct by virtue of Clause 6 of Part I of First Schedule.

Question 24:
A CA in practice created his own website in attractive format and colours and circulated
information contained in the website through. E-mail.
Answer:
Member would be held guilty of professional misconduct as circulation of information contained
in the website through e-mail would amount to solicitation. However no misconduct arises on
his part towards failure to intimate the website address to the institute.

Question 25:
Mr. X, a CA in practice, provides part-time tutorship under the coaching organization of
the Institute. On 30th June, 2019, he was awarded ‘Best Faculty of the year’ as gratitude
from the Institute. Later on, he posted his framed photograph on his website wherein he was
receiving the said award from the Institute.

19
FINAL C.A. – AUDIT

Answer:
Mr. X will be guilty of professional misconduct as the guidelines for the website issued under
Clause 6 of Part I of First Schedule, no photograph of any sort (except passport size photograph
of member’) is permitted to be displayed on website

Question 26:
M/s XYZ a firm in practice develops a website “XYZ. com”. The colour chosen for the website
was a very bright green and the website was to run on a “push” technology where the name
of the partners of the firm and the major clients were to be displayed on the web site without
any disclosure obligation from any regulation.
Answer:
As per guidelines of ICAI in relation to development of website, there is no restriction on
the use of colours. However, the website is required to run on a “Pull” technology and not on
“push” technology; and the name of clients and fees charged from them is not permitted to be
appearing on the website unless required by any regulator. Hence the firm would be liable for
professional misconduct

Question 27:
PQR & Associates, Chartered Accountants have their website and on the letterhead of the
firm it is mentioned that “Visit our website: www.pqr.com”. In the website the nature of
assignments handled, names of prominent ‘ clients and fees charged is also displayed.
Answer:
PQR & Associates w iii be held guilty of Professional Misconduct under Clause 6 of Part I of
First Schedule to the Chartered Accountants Act, 1949. The name of clients and fees charged
from them is not permitted to be appearing on the website unless required by any regulator.

CLAUSE 7 :
1) Advertises his professional attainment or services, or
2) Uses any designation or expressions other than the CA
3) On professional documents, visiting cards, letter heads or sign boards.
4) Unless it be a degree of a university established by law in India or recognized by CG or
5) A title indicating membership of ICAI or
6) Of any other institution that has been recognized by CG or maybe recognized by the
council.
7) Provided that a member in practice may advertise through a write up,
8) Setting out the services provided by him or his firm

20
FINAL C.A. – AUDIT

9) And particulars of his firm subject to such guidelines as maybe issued by the council.
10) [TAPAD2 – SL2AP2]

(D1) DESIGNATION :

CA in Practice CA not in Practice


l Must use designation ‘Chartered l Can use any designation provided they
Accountant’ are not using “Chartered Accountant”
l Cannot use any other designation. as designation
l Can mention degrees from universities l Can use any designation provided they
recognized by law in India or recognized are not using a”Chartered Accountant”
by CG [insolvency professional] as designation
l Any other letter or description indicating
membership of body approved by council
[eg. CISA ie. Certified info system
auditor]
l For Lawyers : Advocacy Cannot use CA
other matter CA

(D2) Date of establishment () Except website

T Training course and seminars :


Allowed but undue prominence should not be given to the name of CA. *

A Advertisement in Press
Not allowed except Recruiting staff in own office
On behalf of client to recruit / acquire / dispose property
Sale of business / Proportion when CA is acting as trustee,
liquidator or receiver

P Photograph and brief particulars of CA in magazine


Allowed provided No payment for such
No mention of professional attainments

21
FINAL C.A. – AUDIT

A Appearance on TV / Films / Radios / Press / Seminars


Allowed provided No reference to Name / Address / Services of firm
Not say anything to promote himself
Even host should not say

S Sign Board
Cannot use glow sign boards or large size boards
At residence, name of board of himself is allowed but not that of firm.

L Logo
Only prescribed by ICAI

L2 Publicity for appointment of position of local / national importance.


l May mention member of ICAI

l No mention of firm name

A Advertisement by Listing in directory or list of members of particular body.


l Allowed

l But cant provide name of clients and services offered by his firm.
Eg. In list of members of IT Appellate Tribunal, when CA is a member of ITAT

P2 Prospectus of co in which CA is director


l No firm name

l No Advertisement of professional achievements

P2 Press note on success of a candidate in exam. May contain


l His name and address and his background

l His success details

l Name and firm of his principal

l But no undesirable publicity of article / principal / firm

22
FINAL C.A. – AUDIT

Questions asked in past/rtp/mock test

Question 28:
Mr. Adnan a Chartered Accountant in practice, is a partner of 4 firms. While printing his
personal letter heads, Mr. Adnan gave the names of all the firms in which he is a partner.
Comment.
Answer:
There is no restriction as to mentioning of firm’s name on personal letter heads under Clause 7
of Part 1 of First Schedule, hence Mr, B (Adnan) is not guilty of any professional misconduct.

Question 29:
Mr. B, a practicing Chartered Accountant as well as a qualified lawyer, was permitted by the
bar council to practice as a lawyer also. He printed his visiting card where he mentioned his
designation as Chartered Accountant and Advocate.
Answer:
Mr. B is guilty of professional misconduct due to simultaneous use of designation as Chartered
Accountant and Advocate.

Question 30:
Mr. Nigal, a Chartered Accountant, in practice, delivered a speech in the national conference
organized by the Ministry of Textiles. While delivering the speech, he told to the audience that
he is a management expert and his firm provides services of taxation and audit at reasonable
rates. He also requested the audience to approach his firm of chartered accountants for these
services and at the request of audience he also distributed his business cards and telephone
number of his firm to those in the audience. Comment.
Answer:
Mr. Nigal will be held guilty ofprofessional misconduct under Clauses 6 & 7 of Part I of First
Schedule to the CA Act 1949, due to solicitation of professional work and advertisement of
services rendered by his firm.

Question 31:
Mr. SP, a Chartered Accountant obtains registration as category IV merchant banker
under the SEBI’s Rules and Regulations and act as Advisor to a capital issue of MB Co. Ltd.
He designated himself under the caption “Merchant banker” in client offer documents and
‘Advisor to issue’ in his own letterheads, visiting cards and professional documents.

23
FINAL C.A. – AUDIT

Answer:
Guilty of Professional Misconduct under Clause 7 of Part I of First Schedule due to use of
designation other than Chartered Accountant.

Question 32:
Mr. M, a chartered accountant in practice, has printed visiting cards which besides other
details also carries a Quick Response (QR) code. The visiting card as well the QR code contains
his name, office and residential address, contact details, e-mail id and name of the firm s
website. Comment.
Answer:
No misconduct arises on part of Mr. M in getting the visiting cards printed which contains
his name address, contact details, e-mail id and name of the firm website. Also, no misconduct
arises in printing QR code on visiting card if it does not contain any information that is not
otherwise permissibly to be printed on visiting card.

Question 33:
R, a practicing Chartered Accountant, Is a Director in X Ltd.; a Public Company. The prospectus
of X Ltd. mentions the name of Mr. R as a director along with his various professional
attainments, his areas of specialization and expertise in the fields of international taxation.
Answer:
Name of CA acting as director in the company is permissible to appear in the prospectus of
the company, however descriptions regarding his expertise, specialisation and knowledge in
any particular field is not permitted. Hence Mr. R will be deemed to be guilty of professional
misconduct under Clause 7, Part 1 of First Schedule.

CLAUSE 8 :
Accept a position a auditor
Previously held by another CA or certified auditor without
1st communicating with him
In writing

NOTES :
1) For all types of audit where previous auditor is a CA.

24
FINAL C.A. – AUDIT

2) Previous Auditor


Before accepting Send a written communication

Proposed Auditor

NOTES :
l By Registered post Acknowledgement due or

l By hand against acknowledgement in writing

l NOT under certificate of posting as he must have positive evidence that the letter had

reached the previous auditor


l Secondary communication can be in email form.

NOTES :
3) Reply of previous Auditor

Replies within a Does not reply in


Reasonable time reasonable time

Analyze the Perform alternative


reasons procedure to find reasons

Reasons May start his work

25
FINAL C.A. – AUDIT

NOTES :

Reasons :
i) Non compliance of provisions of sec 139 of company Act. ()

ii) Non payment of undisputed audit fee (except sick unit) CGG 2008 ()
(In case of disputed fees incoming partner shall use his influence in favour of predecessor
to have dispute settled)

iii) Issuance of qualified report May

iv) Under cutting of fees Accept

NOTES :

4) Conditional Acceptance :
In case of lack of time, the incoming auditor may give conditional acceptance and
commence the work which needs to be attended immediately.

Questions asked in past/rtp/mock test

Question 34:
MR. X a chartered Accountant accepted his appointment as tax auditor of a firm under
Sec. 44AB of the Income Tax Act and commenced the tax audit within two days of his
appointment since the client was in a hurry to file return of income before the due date. After
commencing the audit, Mr. X realised his mistake of accepting this tax audit without sending
any communication to the previous tax auditor. In order to rectify his mistake, before signing
the tax audit report, he sent a registered post to the previous auditor and obtained the postal
acknowledgement. Will; Mr. X be held guilty under the CA Act?

Answer:
Guilty by virtue of clause 8 of Par 11 of First Schedule as communication to previous auditor
is required before commencement of audit.

26
FINAL C.A. – AUDIT

Question 35:
Can a practicing Chartered Accountant be held guilty of professional misconduct under the
following circumstance: W, a chartered Accountant has sent letters under certificate of
posting to the previous auditor informing him about his appointment as an auditor before the
commencement of audit by him.
Answer:
Guilty of professional misconduct by virtue of clause 8 of Part 1 of First Schedule in accordance
with which communication need to be sent “Registered Post Acknowledgement Due (RPAD)” or
by “hand against a written acknowledgement”.

Question 36:
M/s CD & Co., a firm of Chartered Accountants, accepted an assignment for audit under
State level VAT Act and communicated the same over phone to the previous auditor, M/s AB
& Co., Chartered Accountants. Comment.
Answer:
Guilty of professional misconduct by virtue of clause 8 of Part 1 of First Schedule, as incoming
auditor is required to sent his communication by RPAD or by hand against an acknowledgement
in writing. Mere communication over phone is not sufficient to establish written communication.

Question 37:
CA. T, in practice, was appointed to carry out internal audit of a stock broker, listed with
BSE. However,he failed to intimate his appointment to the statutory auditors of the company.
The statutory auditor feels this is violation of professional ethics.
Answer:
There is no violation of professional ethics as Clause 8 of Part 1 of First schedule applies in
case of replacement positions and not in case of parallel positions.

CLAUSE 9 :
Accepts an appointment as auditor of a company
Without ascertaining* whether requirements of sec 139, 140, 141, 142 of companies
Act, 2013
In respect of such appointment have been duly complied with.

 To ascertain means to find out for certain. i.e. it is not sufficient to accept compliance
certificate from management. He may verify Board resolutions, General meeting
resolutions; etc.

27
FINAL C.A. – AUDIT

Questions asked in past/rtp/mock test


Question 38:
P, a Chartered Accountant had accepted appointment as an auditor of QRS Company limited
without ascertaining from the company whether the requirements of Sees. 139 and 140 of
the Companies Act, 2013 had been complied with. However, he realised this defect only after
acceptance.
Answer:
P will be held to be guilty of professional misconduct under clause 9 of Part 1 of First Schedule
to the Chartered Accountants Act, 1949. Realisation of defect subsequent to acceptance of
audit is immaterial.

Question 39:
CA X was appointed as the Auditor of ABC Ltd. for 2018-19, Since he declined to accept
the appointment, the Board of Directors appointed CA Y as the auditor in the place of CA X,
which was also accepted by CA Y.
Answer:
CA, Y is guilty of professional misconduct as per Clause 9 of Part 1 of the First Schedule as
he accepted the appointment without verification of statutory requirements of Sec. 139 of
Companies Act, 2013.

CLAUSE 10 :
Charges or offers to charge,
Accepts or offers to accept
In respect of any professional employment
Fees which is based on a % of profits or,
Which are contingent upon findings, or results of such employment.
Except as permitted under regulations*

Regulation 192 : [LCM FOD2]


1) Liquidator % of realized value
2) Co-operative society % paid up capital / working cap
/ Gross or net profit.
3) Cost optimization % of benefit derived
4) Management consultancy service % of result / as considered suitable
5) Fund raising service % of fund raised.
6) Other service as may be decided by council.

28
FINAL C.A. – AUDIT

7) Debt recovery service % of debt recovered


8) Direct taxes % of value (For eg: He can charge fees based on %
value of G/W for determining value of gift under Gift
tax as gift tax is a direct tax)

Questions asked in past/rtp/mock test

Question 40:
CA D, a Chartered Accountant prepared a project ‘report for one of his clients to obtain bank
finance (long-term) of Rs. 50 lakhs from a Commercial Bank. Consequent to the sanction of
the loan by the bank CA. D raised a bill for his services @ 2% of the loan sanctioned.
Answer:
Charging fees on a % basis amounts to misconduct under Clause 10 of Part! of First Schedule
except as permitted under regulation 192. Charging fees on % of fund raised is covered under
regulation 192. Hence no misconduct arises on part of Mr. D.

Question 41:
An auditor of a co-operative society has agreed to charge fees @ 5% of the profits of the
society.
Answer:
Audit of co-operative society is included in the exceptions stated in regulation 192, the auditor
is not guilty of any professional misconduct.

Question 42:
PQR Pvt. Ltd. approached CA. Y, a Chartered Accountant in Practice, for debt recovery
services. CA Y accepted the work and insisted for fees to be based on 2% of the debt recovered.
Answer:
No misconduct arises as charging fees in case of debt recovery services on the basis of
percentage of the debt recovered is permitted under regulation 192.

Question 43:
Mr. P a practicing chartered accountant acting as liquidator of AB & Co. charged his
professional fees on percentage of the realization of assets.
Answer:
No Misconduct on part of Mr. P under Clause 10 as charging fees as a percentage of realisation
of assets while acting as a liquidator is permitted by Regulation 192.

29
FINAL C.A. – AUDIT

Question 44:
Efficient Ltd. is running into losess and in order to optimize resource utilization and cost
reduction, approaches you to carryout the assignment and offers a fee of 5% of benefits
derived from the suggestions made by you.
Answer:
No Misconduct arises under Clause 10 as charging fees as a percentage of benefits received
in case of cost optimisation services is permitted by Regulation 192.

CLAUSE 11 :

Engages in any business or occupation


Other than profession of CA
Unless permitted by council so to engage.
Provided that nothing contained herein shall disentitle a CA
From being a director of a company unless he or any of his partner is interested in such
company as an auditor (Not MD or whole time director)

GAITONDE

Regulation 190A:
General Resolution No Specific permission
[SLAP4 – RE2HANA]

S Acting as surveyor and loss assessor under insurance Act, 1938

30
FINAL C.A. – AUDIT

L Holding Life insurance agency for Limited purpose of getting renewal commission

A Authorship of books and articles

P1 Private tutorship

P2 Holding of Public elective office such as M.P, M.L.A

P3 Valuation of papers, acting as paper – setter, head-examiner or a moderator, for


any examination

P4 Part – Time tutorship under the coaching organization of the institute

R Acting as recovery consultant in the banking sector

E1 Employment under CA in practice

E2 Editorship of professional journals

H Honorary office leadership of charitable – educational or other non-commercial


organizations.

A Attending classes and appearing for any examination.

N Acting as Notary Public, justice of the peace, special executive magistrate and the
like.

A Owing agricultural and carrying out agricultural activity

P1 Part time / Full time employment in business concern provided that member or his
relative do not hold substantial interest in such concerns.

P2 Part time / Full time employment in non-business concern

P3 Part time / Full time lectureship for course other than those relating to institute’s
examination conducted

31
FINAL C.A. – AUDIT

P4 Part time / Full time tutorship under any educational other than the coaching
organization of the Institute

R Any other business which requires permission

I1 Interest in family business

I2 Interest in educational institution

M Managing director or a whole time director of a body corporate. (Auditor of


subsidiary can’t be director of holding)

E Editorship of journal other than professional journals.

Questions asked in past/rtp/mock test


Question 45:
Mr. J started his practice as Chartered Accountant in 2013. During 2019, he got an offer
for the post of Chief Accountant of a Software Development’ Company, as a full time employee
for a salary of Rs. 60,000 p.m. On accepting the offer, Mr. J converted his practice into
a partnership firm by taking fresh Chartered Accountant as his partner. Mr. J neither
intimated the institute nor obtained permission from the Institute about his employment. Will
Mr. J be held guilty under the CA Act?
Answer:
Guilty of professional misconduct by virtue of Clause 11 as he has accepted the employment in
addition to the practice without obtaining permission of the Institute.

Question 46:
M, a Chartered Accountant in practice, is the Statutory Auditor of S Ltd. for the year ended
31st March 2019. In January 2019, he was appointed as a Director in H Ltd., which is the
holding Company of S Ltd.
Answer:
Auditor of a subsidiary should not accept the position as a director in holding company as it
might affect his independence.

32
FINAL C.A. – AUDIT

Question 47:
Mr. A, a Chartered Accountant in practice has been appointed editor of a monthly journal
which analyses performance of the Stock Market and Mutual Fund Schemes.
Answer:
Clause 11 permits editorship of professional journals, but in the instant case journal related
to performance analysis of stock market and mutual fund schemes cannot be treated as
professional journal hence, Mr. A would be held guilty of professional misconduct by virtue of
Clause 11.

Question 48:
CA. Preeti is a leading Income Tax Practitioner in Delhi. She is very much fond of cooking. Due
to this passion of her, she also wrote a cookery book “Delight your tummy” during the year.
But, she didn’t take any permission from the Council of the Institute for engaging herself into
authorship of such book. Comment.
Answer:
Clause 11 permits authorship of any book, hence no misconduct arises on part of CA Preeti.

Question 49:
CA Z, is a leading income tax practitioner and consultant for derivative products. He resides in
Mumbai near to the ABC commodity stock exchange and does trading in commodity derivatives.
Every day, he invests nearly 50% of his time settle the commodity transactions. Is CA Z liable
for professional misconduct?
Answer:
If Mr. Z, has, obtained specific permission of the council; then there is no misconduct,
otherwise he will be deemed to be guilty of professional misconduct under clause 11

Question 50:
A Chartered Accountant having COP entered into partnership with persons, who are not the
members of the institute, for the purpose of carrying on business. The share of the Chartered
Accountant in the profit and losses was 25%. He was to take part in the business and was
entitled to represent the firm before Govt, authorities etc. he was operating the bank account
of the firm was receiving moneys from the customers and was also looking after the affairs of
the partnership.
Answer:
Guilty of Professional Misconduct by virtue of clauses -4 & 11 of Part 1 of First Schedule due
to entering into partnership with persons other than prescribed under Clause 4 and carrying

33
FINAL C.A. – AUDIT

on a business without obtaining permission from Council of ICAI.

Question 51:
CA Raghu is practicing in the field of income-tax over a period of 12 years. He has gained
experience in this domain over others.
Sam, a student of Chartered Accountancy Course is very much impressed with the knowledge of
CA Raghu. He approached CA Raghu to take guidance on some topics of Income-tax related to
his course. CA Raghu, on request decided to spare time and started providing private tutorship
to Sam and some of his friends along with. However, he forgot to take specific permission from
the ICAI, for such private tutorship.
Is CA Raghu, professionally liable for misconduct ?
Answer:
Mr. Raghu will not be liable for professional misconduct as for imparting private tutorship
no specific permission is required to be obtained from Council of ICAI as provided under
Regulation 190A.

CLAUSE 12 :
A person not being a member of the institute in practice or
A member not being his partner
To sign on his behalf or on behalf of his firm,
Any BS, Profit and Loss, Report or FS

Note 1 : FS means FS and Reports finalized and submitted

Note 2 : Council has clarified that power to sign routine documents on which a professional
opinion or authentication is not required maybe delegated.

Questions asked in past/rtp/mock test

Question 52:
S. a practicing Chartered accountant gives power of attorney to an employee Chartered
accountant to sign reports and financial statements, on his behalf.
Answer:
S is guilty of professional misconduct under clause 12

34
FINAL C.A. – AUDIT

Question 53:
Mr. ‘A’ is a practicing Chartered Accountant working as proprietor of M/s A &Co. He went
abroad for 3 months. He delegated the authority to Mr. ‘Y’ a Chartered Accountant his
employee for taking care of routine matters of his office. During his absence Mr. ‘Y’ has
conducted the undermentioned jobs in the name of M/s A & Co.
i) He issued the audit queries to client which were raised during the course of audit.
ii) He issued production certificate to a client under GST Laws.
iii) He attended the Income Tax proceedings for a client as authorized representative before
Income-Tax Authorities.
Comment on eligibility of Mr. ‘Y’ for conducting such jobs in name of M/s A & Co. and
liability of Mr. ‘A’ under the Chartered Accountants Act, 1949.

ANSWER
i) Issuing audit queries during the course of audit falls under routine work, which can be
delegated by the auditor. Therefore, there is no misconduct in this case.
ii) Issuance of production certificate to a client under GST Laws, by Mr. “Y” is not a routine
work and it is outside his authorities. Thus, CA ‘A’ is guilty of professional misconduct
under clause 12 of Part I of First Schedule of the Chartered Accountants Act, 1949.
iii) Attending Income tax proceedings for a client as authorized representative before Income
Tax Authorities falls under routine work, hence Mr. Y, the employee of M/s A & Co. can
attend to routine matter in tax practice. Therefore there is no misconduct in this case.

CLAUSE 1 :

Pays or allows or agrees to pay


Directly or indirectly to any person
Any share in the emoluments of the employment undertaken by him.

CLAUSE 2 :
Accepts or agrees to accept
Any part of fees, profits or gains from
A lawyer, a CA or broker engaged by such company, firm or person or
Agent or customer of such company, firm or person
By way of commission or gratification

35
FINAL C.A. – AUDIT

Questions asked in past/rtp/mock test

Question 54:
Mr. ‘C’, a Chartered Accountant holds a certificate of practice while in employment also,
recommends a particular lawyer to his employer in respect of a case. The lawyer, out of the
professional fee received from employer paid a particular sum as referral fee to Mr. ‘C’.
Answer:
Mr. C is guilty of professional misconduct by virtue of clause 2 for accepting referral fees from
the lawyer of his employer.

PART III : PROFESSIONAL MISCOUNT IN RELATION TO MEMBERS GENERALLY

CLAUSE 1 :
Not being a fellow of the institute
Acts as a fellow of the institute

CLAUSE 2 :
Does not supply the information called for, or
Does not comply with the requirements asked for by
The institute, council or any of its committees, director (Discipline), board of discipline,
disciplinary committee, quality review board or the appellate authority. (ICD BDQA)

Questions asked in past/rtp/mock test

Question 55:
Mr. X, a Chartered accountant, employed as a paid assistant with a Chartered Accountant
firm. On 31st Dec. 2018, he leaves the services of the firm. Despite many reminders from
ICAI he fails to reply regarding the date of leaving of the services of the firm.
Answer:
Mr. X is held guilty of professional misconduct as per Clause 2 of Part III

Question 56:
Mr. ‘G’, while applying for a certificate of practice, did not fill in the columns which solicit
information about his engagement in other occupation or business, while he was indeed engaged
in a business.

36
FINAL C.A. – AUDIT

Answer:
Mr. G will be held guilty for professional misconduct under the Clause 2 of Part III

Question 57:
XYZ Associates, a Chartered Accountants Firm is having a relationship with a multi-national
accounting firm in India. The ICAI required that all firms having networking relationship
with any other entity need to furnish information online within the stipulated time. XYZ
Associated failed to respond. Comment.
Answer:
XYZ Associates will be held guilty for professional misconduct under the Clause 2 of Part III
of First Schedule for not providing the information to the institute.

PART III : PROFESSIONAL MISCOUNT IN RELATION TO MEMBERS GENERALLY

CLAUSE 3 :
While inviting professional work from another CA or
While responding to tenders or enquires or
While advertisement through a write up or anything as provided for in clauses (6) and
(7) of part I of this schedule,
Gives information knowing it to be false.

PART IV : OTHER MISCONDUCT IN RELATION TO MEMBERS GENERALLY

CLAUSE 1 :
Is held guilty by any civil or criminal court
For an offence which is punishable with imprisonment
For a term not exceeding six months.

PART IV : OTHER MISCONDUCT IN RELATION TO MEMBERS GENERALLY

CLAUSE 2 :
In the opinion of the council
Brings disrepute to the profession or the institute
As a result of his action
Whether or not related to his professional work.

37
FINAL C.A. – AUDIT

Examples :
[CA MR2 I2]

C Where a CA had adopted coercive methods on a bank for having a loan sanctioned
to him

A Uses the service of his article for purpose other than professional practice

M Misappropriation by office bearer of a regional council of the institute of a large


amount and utilization for his personal use.

R1 Retains books of accounts and documents of the client and fails to return these to
client on request without responsible cause.

R2 Not replying within a reasonable time and without good cause to the letter of the
public authorities.

I1 Cheque signed is returned due to Insufficiency of funds

I2 Assessment records income tax dept. belonging to client in bed-room of CA

Questions asked in past/rtp/mock test

Question 58:
CA D, a chartered accountant in practice availed of a loan against his personal investments
from a bank. He issued 2 cheques towards repayment of the said loan as per the instalments
due. However, both the cheques were returned back by the bank with the remarks “Insufficient
funds”. Comment.
Answer:
As the cheques were dishonoured due to insufficiency of funds, the drawer will he held guilty of
offence under Negotiable Instruments Act, 1881 and consequently CA D would be held guilty
of “Other Misconduct”.

38
FINAL C.A. – AUDIT

Question 59:
Mr. P, a Chartered Accountant in practice approached Manager of a Nationalised Bank for
a loan of Rs. 25 lakhs. He has also informed the Manager that if the loan is sanctioned, the
Income Tax return of the Manager and staff will be filed without charging any fees, as quid
Pro quo for the loan sanctioned.
Answer:
Mr. P will be held guilty of other misconduct under Clause 2 of Part IV of the First Schedule

Question 60:
Mr. A, a practicing Chartered Accountant, failed to return the books of account and other
documents of a client despite many reminders from the client. The client had settled his
entries fees dues also.
Answer:
Mr. A would be guilty of Other Misconduct under Part IV of First Schedule

Question 61:
CA Kumar who is contesting Central Council Elections of Institute, engages his Articled
Assistant for his election campaigning promising him that he will come in contact with
influential people which will help to enhance his career after completion of his training period.
Answer:
Mr. A would guilty of other Misconduct under Part IV of First Schedule and liable to disciplinary
action under Section 21.

How to memorize the clauses

First Schedule :
Part I: AP AP SAP ACES

Part II: PA

Part III: FSI

Part IV: CO

39
FINAL C.A. – AUDIT

SECOND SCHEDULE :

PART I : Professional Misconduct in relation members in practice

CLAUSE 1 :

Discloses the information


Acquired in the course of his professional engagement
To any person other than his client so engaging him
Without any consent of his client or
Otherwise than as required by any law for the time being inforce.

NOTE 1 :

WHO CAN GIVE CONSENT

Proprietor : Proprietor himself or his representative

Partnership Firm : Any of the partner

Company : MD can give only if he is authorized or BOD

NOTE 2 :

WORKING PAPERS

Auditor Working Paper Branch Subsidiary Holding

NOTE 3 :

DUTY IN RELATION TO ANY UNLAWFUL ACTS BY CLIENTS

No duty to inform tax authorities about tax frauds by clients


Also no duty of CA to shield him from consequences of fraud.
His responsibility is to advice client in persuasive way not to involve in tax frauds by
impressing upon him that :

40
FINAL C.A. – AUDIT

Disclosure by client may entail only penalties but non-disclosure may result even in
imprisonment
If CA informs tax authorities about his disassociation form matters, authorities may
start the investigation.
In case of genuine mistake client will himself disclose

NOTE 4 :

SUMMONS

If tax authorities summon CA for examining him on oath or for production of BOA, he
should take legal expert opinion.

NOTE 5 : FRAUD CG 143(12)


FRAUD RELATES TO

PAST YEARS RETURN CURRENT YEARS


/ ACCOUNT RETURN / ACCOUNTS

Where client was


Where client Which But CA’s
represented by was represented are being assignment was
some other CA by him only prepared done was done
before return /
audit.

41
FINAL C.A. – AUDIT

NOTE 5 :

FRAUD CG 143(12)

Where client was Where client was Which But CA’s assignment
represented by represented by are being was done was done
some other CA him only prepared before return / audit.

l CA should l CA should advise client to l CA should advice No Duty


advice client disclose client to disclose
to disclose l If he refuse to disclose, CA l If he refuses,
l However, should disassociate himself disassociate himself
he may with client and report and prepare audit
continue to tax authorities A/c etc. report accordingly
act for C.Y. previously reported by him (Qualified / adv.)
are unreliable and thus he is
disassociating himself. (But
no disclosure of exact fraud)

Questions asked in past/rtp/mock test

Question 62:
A CA in practice was engaged by a businessman to represent him before the tax authorities
on current matters and in the course of such “employment he came across Certain documents
pointing of tax frauds in the preceding years for which tire client was not represented by
him. Is the member liable to disclose the existence and contents of the documents to tax
authorities?
Answer:
In respect of such matters, the code of conduct recommends that “If. the fraud discovered by
the member, relates to the accounts; or-tax matters of the client for past year(s) for which
the client was not represented by the member the client should be advised to make a disclosure.
The member may however, continue to act for the client in respect of current matter, but is
tinder no obligation so to continue.

42
FINAL C.A. – AUDIT

Question 63:
XYZ LtD Co. Ltd. has applied to a bank for loan facilities. The bank on studying the financial
statements of the company notices that you are the auditor and requests you to call at the
bank for a discussion. In the course of discussions, the bank asks for your opinion regarding
the company and also asks for detailed information regarding few items in the financial
statements. The information is available in your working paper file. What should be your
response and why?
Answer:
There is no requirement compelling the auditor to divulge information obtained in the course of
audit and included In the working paper- to any outside agency except as and when required
by any law or permitted by the client.

Question 64:
Mr. C, a practicing Chartered Accountant, in the course of the audit of a listed company
discovered serious violations of the provisions of the Companies Act, 2013, informed the
registrar of companies out of public interest
Answer:
Mr. C will be guilty of professional misconduct covered by clause 1 of Part I of Second Schedule

Question 65:
Mr. Z, a Chartered Accountant was invited by ‘the Chamber of Commerce to present a
paper in a symposium on the issues facing Indian Leather Industry. During the course of
his presentation he shared some of the vital information of his client’s business under the
impression that it will help the Nation to compote with other countries at international level.
Answer:
Disclosing the client’s information without obtaining consent of client amounts to professional
misconduct under Clause 1 of Part 1 of Second Schedule

CLAUSE 2 :

Certifies or submits in his name or in the name of his firm


A report of an examination of financial statements
Unless the examination of such statements and the related records has been made by
l Him or

l By a partner or

l An employee in his firm or

43
FINAL C.A. – AUDIT

l By another CA in practice

NOTE : Not applicable for Joint auditor

Questions asked in past/rtp/mock test

Question 66:
Mr. L, a Chartered Accountant Issued a certificate of consumption which did not reflect
correct factual position.
Certificate was given on the basis of minutes of BOD meeting.
Answer:
Mr. L will be guilty of professional misconduct.

CLAUSE 3 :

Permits his name or the name of his firm


To be used in connection with an estimate of earnings
Contingent upon future transactions
In manner which may lead to the belief that he voucher for the accuracy of the forecast

NOTE : SAE 3400 “The examination of Prospective financial information.”

As per SAE 3400 on prospective financial info CA can


participate in preparation of financial forecasts and their
review. He should clearly indicate in his report. (SA2B)

S Source of information

A1 Major assumptions

A2 Not vouch for accuracy of forecast

B Basis of forecasts

44
FINAL C.A. – AUDIT

Questions asked in past/rtp/mock test

Question 67:
CA. Q certifies financial forecast of client and same was forwarded to bank by client. Based
on the certification loan was sanctioned.
Answer:
Mr. Q will be not be guilty of professional misconduct.

Question 68:
Mr. N , a practicing Chartered Accountant was requested by client to prepare projections
& report for next 5 years. He stated in his report the sources of info, basis of forecast &
major assumptions made to arrive at forecasts & does not vouch for accuracy of the forecast.
Comment.
Answer:
Mr. N will not be guilty of professional misconduct.

Question 69:
D, a practicing Chartered Accountant examined and reported on the prospective financial
statements for one of his clients to obtain a cash credit facility of Rs.75 lakhs from a
Private Bank. The bank has sanctioned the cash credit facility for Rs. 60 lakhs to his client.
Consequent to the sanction of loan by Bank, he charged a fees of Rs. 60,000 based on 1% of
the credit facility-sanctioned.
Answer:
Mr. D is allowed to examine the prospective financial information of the clients provided
conditions as stated in SAE are being fulfilled. Assuming that conditions stated in SAE 3400
are being fulfilled, no misconduct arises on part of Mr. D under Clause 3 of Part 1 of Second
Schedule. However, charging fees on a % age basis amounts to misconduct under Clause 10 of
Part 1 of First Schedule except as permitted under regulation 192. Charging fees on % age of
fund raised is covered under Regulation 192. Hence no misconduct arises on part of Mr. D.

CLAUSE 4 :

Expresses his opinion


On FS of any business or enterprise
In which he, his firm or a partner in his firm has a substantial interest.

45
FINAL C.A. – AUDIT

NOTE :
1. Council Guidelines No. 1CA(7)/02/2008 dated
8th August , 2008. The said guidelines state that
a member of the Institute shall not express his opinion on financial statements of any business
or enterprise in which one or more persons, who are his “relatives” within the meaning of AS-
18

2. Many new areas of professional work have been added ,e.g. Tax audit, Concurrent Audit
of Banks, Concurrent Audit of borrowers of Financial institution, audit of non-corporate
borrowers of bank and financial institution, audit of stock exchange, brokers, etc. Clause 4
are equally applicable to all types of attest function by members

NOTE :
3) CA himself is owner /
CANT AUDIT
partner

CA himself is director CANT AUDIT


IN RELATION TO
BUSINESS ENT.
Partner / Relative (AS
– 18) of CA is director CANT AUDIT

Partner / Relative
CA has substantial CANT AUDIT
interest

NOTE :
4) If CA is employee of concern He cannot audit FS of employer
CA is a part time lecturer in college He cannot accept auditorship of college
CA is appointed as liquidator Cannot audit of that company
A partner of CA is trustee of trust Cannot audit FS of trust
Writing books of accounts Cannot audit
CA is internal auditor Cannot be statutory auditor

46
FINAL C.A. – AUDIT

NOTE :
5). This clause is meant for report as well as for certificates which are to be submitted to
any outside authority, but not where statements are prepared by members in employment
for their employers.

6). The council has clarified that the members are not permitted to write books of their
audit clients

Questions asked in past/rtp/mock test

Question 70:
AP & Co., a firm of Chartered Accountants, was appointed by D Ltd., to evaluate the cost of
a new product manufactured by it for their information system and fixation of fair market
price. Partner ‘P of the CA firm is a non-executive director of the Company. Comment with
reference to Chartered Accountants Act, 1949 and Regulations thereto.
Answer:
There is no professional misconduct in evaluating the costs of a company in which one of the
partners of firm is a non-executive director.

Question 71:
Mr. W, is appointed as statutory auditor of Pvt Ltd co in which Spouse’s sisters husband holds
75% ownership.
Answer:
There is no professional misconduct

CLAUSE 5 :
Falls to disclose material fact
Known to him
Which is not disclosed in a FS
But disclosure of which is necessary
In making such FS not misleading
Where he is concerned with that FS in a professional capacity

NOTE 1 :
FS would cover both report + Certificates

47
FINAL C.A. – AUDIT

NOTE 2 :
Materially should be judged in relation to both Profit and Loss and Balance sheet. An item
can be material form point of view of profit and Loss but may not be material from point of
view of Balance sheet.
The determination of materiality has been provided in SA 320 “ Materiality In planning &
performing Audit.”

Questions asked in past/rtp/mock test

Question 72:
Mr. A was appointed by H Ltd. to audit the PF trust maintained by the company. While
conducting the audit he noticed that large number of loans have been given out of the trust
to the employer company in contravention of the rules of the PF Trust. He disclosed the
irregularities to the trustees and to the company but not to the individual subscribers of
the PF. When queried on his omission to disclose, he explained that he owed no duty to the
individual members.
Answer:
Guilty by virtue of Clause 5 of Part 1 of Second Schedule as he was required to disclose the facts
to beneficiaries of the fund just like he gives his report to the shareholders of the company.

Question 73:
Mr. J, a Chartered Accountant has identified that ABC Ltd. has taken a loan of Rs.15 lakhs
from Provident Fund Account, during the course of audit. The said loan was not reflected in
the books of account and statements were prepared ignoring the same.
Answer:
If Mr. J fails to disclose the fact in his report, he will be attracted by the professions of
professional misconduct under Clause (5) of Part 1 of Second Schedule .However, if he discloses
the fact in his report, there will not be any misconduct.

Question 74:
The superannuation-cum-pension fund for the employees of a company was under a separate
‘trust’. Both the company and the trust were under the same management. The auditor, who
was auditing the accounts of the company as well as the trust noted some irregularities in the
operation of the trust and commented upon these irregularities in the confidential report given
to the trustees, but did not mention about these irregularities in his report on the Annual
accounts of the Trust.

48
FINAL C.A. – AUDIT

Answer:
The Chartered Accountant is guilty of professional misconduct if the amount of irregularities
is proved material for not disclosing the facts in his report.

CLAUSE 6 :

Fails to report a material misstatement


To appear in FS
With which he is concerned in a professional capacity

Questions asked in past/rtp/mock test

Question 75:
Mr.X partner of X & Co. Chartered Accountants, has compiled and signed the balance sheet
of False Ltd. for submission to the bankers of the said company. Mr. X has also compiled
and signed at the request of the company another balance sheet inflating the value of assets
by 20%, for submission to a term lending institution. Both the Balance Sheets were not in
conformity with the books of account maintained by the company as they were not up-to-
date. Comment on Mr. X liability.
Answer:
Mr. X would be held guilty under Clauses 5 and 6 of Part 1 of Second Schedule as Mr. X had
compiled the two different BS for the same date without reference to the actual books of
account, but on instructions of the client. As per clause 5 he has failed to disclose material
fact known to him & as per clause 6 he has also failed to report a material misstatement
known to him.

Question 76:
A practicing Chartered Accountant was appointed to represent a company before the tax
authorities. He submitted on behalf of his clients certain information and explanations to the
authorities, which were found to be false and misleading
Answer:
The CA would not be held liable as he was not being engaged for examination of any information
and he had only submitted the infprmation acting on the instruction of the client as authorised
representatives.

49
FINAL C.A. – AUDIT

CLAUSE 7 :
Does not exercise due diligence, or
Is grossly negligent
In the conduct of his professional duties

l Due diligence means careful and thorough work or effort.Mere non-performance or


defective performance of a duty may be considered as failure to exercise due deligence.

l Gross negligent implies negligence of high degree, either arising out of recklessness or
deliberate failure to act honestly and reasonably on a material matter.

NOTE :
1) If a CA is appointed to carry out a audit and later an internal auditor detected
irregularities at a branch which is not detected by auditor. He is not guilty as he is not
required to check the matters relating to branch in dept.

2) He is a watchdog but not bloodhound. If there is anything to excite his suspicion he


should probe it to bottom, but in the absence of anything of that kind he is only bound
to be reasonably cautious and careful.

EXAMPLES :
1) CA gave clean exports whereas the reports on the special Audit conducted subsequently
revealed irregularities which amounted to failure to examine passbook and to verify cash
balance.

2) A CA adopted arbitrary valuation of closing stock and no verification was done by him.

3) Failure to point out contravention of requirements of schedule III of companies Act.

4) Failure to detect fraud committed by accountant which could have been detected if he
had properly checked cash book.

5) A CA relied upon IC without satisfying himself about the propriety and surrendered to
the pressure of management and certified the accounts without examining.

50
FINAL C.A. – AUDIT

6) Where CA in practice have signed 2 Balance sheet on 2 different dates for the
samefinancial year, the 1st one with a clean report and 2nd one with a qualified report.
Because he later on issued a clean report and did not refer the fact of having previously
issued a qualified report, in lieu of which a clean report was being issued.

7) Included order still under negation as sales to reflect better financial position. .

8) Cashier absconded with proceeds of sales, the auditor failed to discover it and an
investigation afterwards indicated that he did not exercise proper skill and care.

Questions asked in past/rtp/mock test


Question 77:
Mr. Fair a practicing CA, was appointed to carry out a Balance sheet Audit of a Non-profit
Organisation. The Internal Auditors detected certain irregularities at one of the branches of
the organization which Mr. Fair had failed to detect.
Answer:
Mr. Fair is not guilty as he was appointed to carry out the Balance Sheet Audit, hence he is
not required to check matters relating to branch in depth.

Question 78:
A search under section 132 of Income Tax Act in the premises of a leading merchant led to
the discovery of two sets of account books - one set to record all the income correctly and
second to record only limited income. A CA has issued audit report on the basis of second set
of books. What would be your answer if the first set of account books carried evidence of
checking by the CA?
Answer:
In the first case, he would not be deemed to be guilty of misconduct. But in the second case,
he would be deemed to be guilty of misconduct.

Question 79:
Mr. X, a CA in practice and statutory auditor of True Ltd., advised the Managing Director
of the company to include in sales, “Orders under negotiation” to reflect a better financial
position for obtaining bank loan. Mr.X, thereafter, gave clean reports on the balance sheet
prepared accordingly without examining the accounts
Answer:
Guilty of misconduct under Clauses 2 & 7 of Part 1, Second Schedule to the CA Act, 1949, as
51
FINAL C.A. – AUDIT

he issued the report without examining the accounts and has acted in a negligent manner.

He will also be deemed guilty of other misconduct under Clause (2) of part IV of First Schedule
for advising unethical practice to the client.

Question 80:
• The Cashier of a company committed a fraud and absconded with the proceeds thereof.
This happened during the course of the accounting year. The Chief Accountant of the
Company also did not know about fraud.
• In the course of the audit, at the end of the year, he auditor failed to discover the
fraud. After the audit was completed, however, the fraud was discovered by the Chief
Accountant. Investigation made at that time indicate that the auditor did not exercise
proper skill and care and performed his work in a desultory and haphazard manner. With
this background, the Directors of the company intend to file disciplinary proceedings
against the auditor. Discuss.
Answer:
The auditor has been grossly negligent in performing his duties which constitutes professional
misconduct. Thus, such instances require reference to Disciplinary Committee of the Council
of the Institute.

Question 81:
Mr. B a practicing CA expressed his opinion on the financial statements of M/s ABC Ltd. for
the year ended on 31st March 2018. It was later found that the closing stock was valued
arbitrarily by Management which was accepted by him without verification and large amount
of revenue expenditure was capitalized.
Answer:
Mr. B is guilty of gross negligence by virtue of Clause 7 of Part 1 of Second Schedule

Question 82:
Mr. D, a practicing Chartered Accountant, did not complete his work relating to the audit of
the accounts of a company and had not submitted his audit report in due time to enable the
company to comply with the statutory requirements.
Answer:
Mr. D is guilty of professional misconduct by virtue of Clause 7 of Part I of Second Schedule
to the CA Act, 1949.

52
FINAL C.A. – AUDIT

CLAUSE 8 :
Fails to obtain sufficient information
Which is necessary for expression of an opinion or
Its exceptions are sufficient material to negate the expression of an opinion.

NOTE :
1) A CA should express his opinion about fairness of statements of accounts only after
obtaining required data and information. He has to determine extent to which information
is required.

2) In case of inadequacy of information of data he should clearly express his disclaimer.

Questions asked in past/rtp/mock test

Question 83:
CA Z who conducted ABC audit of a Marathi daily ‘New Era’ certified the circulation figures
based on Management information System Report (M.I.S. Report) without examining the
books of Account.
Answer:
CA Z is guilty of professional misconduct as per Clauses 2, 7 and 8 of Part 1 of Second
Schedule as he certificate is issued without examination of related records, failed to exercise
due diligence and failed to obtain necessary information.

Question 84:
Mr. A, a CA was the auditor of ‘A Ltd’. During the financial year 2010-19, the investment
appeared in the Balance Sheet of the company of Rs. 10 lakhs and was the same amount as
in the last year. Later on, it was found that the company’s investments were only Rs. 25,000,
but the value of investments was inflated for the purpose of obtaining higher amount of Bank
loan.
Answer:
Mr. A, will be held liable for professional misconduct under Clauses (2), (7) and (8) of Part 1
of the Second Schedule

53
FINAL C.A. – AUDIT

CLAUSE 9 :
Fails to invite attention to any material departure.
From the generally accepted procedure of audit applicable to the circumstances.

NOTE 1 :
If he fails to perform the audit as per such procedures, his report should draw attention to
the material departure from such procedures.

NOTE 2 :
Failure to perform certain statutory functions and duties is not excused by giving a qualification
or reservation in auditors report, on failure he should clearly indicate reasons for failure to
perform audit as per generally accepted procedures and standards.

CLAUSE 10 :
Fails to keep money of his client
Other than fees or remuneration or money meant to be expended,
In a separate banking account or
To use such money for purpose for which they are intended within a reasonable time*

v Reasonable Time would depend upon circumstance of each case.

NOTE 1 :
An advance received by a CA Against services to be rendered does not fall under clause (10)
of Part I of 2nd schedule.

NOTE 2 :
Money received for expenses to be incurred, for example, Payment of prescribe statutory fees,
purchase of stamp paper, etc. Which are intended to be spent within a reasonably short time
need not be put in a separate bank account.

NOTE 3 :
Money received by a CA, in his capacity as the trustee, executor etc., must be put in a separate
bank account immediately.

54
FINAL C.A. – AUDIT

Questions asked in past/rtp/mock test

Question 85:
A charitable institution entrusted Rs 10 lakhs with its auditors M/s Ram and Co., a Chartered
Accountant firm, to invest in a specified securities. The auditors pending investment of the
money deposited it in their Savings bank account and no investment was made in the next
three months.
Answer:
M/s Ram & Co. will be held guilty of professional misconduct as he deposited the client money
in his saving bank account.

Question 86:
Mr. Ram, a CA in practice, received Rs. 15,00,000 on 15th December, 2018 on behalf of one
of his clients, who has gone to USA. Mr. Ram deposited the said amount in his saving bank
account .As per instruction of the client, the said amount is to be returned to the client on
March 31, 2019 when he will return to India. On the occasion of birthday of his wife Sita,
Mr. Ram withdrew Rs. 5,00,000 and spent on Birthday party. He re- deposited Rs. 5,00,000
in the said SB account on 25th March, 2019 and then returned the entire amount of Rs.
15,00,000 to the client on March 31, 2019.
Answer:
Mr. Ram will be guilty of professional misconduct under Clause 10 of Part 1 to Second Schedule
due to utilisation of Rs. 5 Lacs for personal purposes.

Question 87:
Mr. Z, a practicing Chartered Accountant received a sum of Rs. 1 lac on 01.09.2018 from a
client who intends to leave abroad for a period of year, with a request that his advance tax
liabilities to be paid over the three instalments, on 15th Sept., 2018, 15th Dec. 2018 and
15.03.2019. After remitting the 1st instalment of advance tax on 15.09.2018, Z did not keep
the money in a separate bank account and he is of the opinion that he will remit the money
within reasonable time as per schedule of advance tax
Answer:
Mr. Z is guilty of professional misconduct as per Clause 10 of Part 1 of Second Schedule of the
Chartered Accountants Act, 1949.

55
FINAL C.A. – AUDIT

PART II : PROFESSIONAL MISCONDUCT IN RELATION TO MEMBERS OF THE


INSTITUTE GENERALLY.

CLAUSE 1 :
Contravenes any of the provision of this Act or
The regulations made there under or
Any guidelines issued by the council.

NOTE :
REGULATION 10 : Cancellation of COP

REGULATION 19: Restoration of membership

REGULATION 43 : Engagement of Article Assistant

REGULATION 46 : Registration of Articled Assistant

REGULATION 47 : No premium from Article Assistant

REGULATION 48: Stipend to Articled Assistant

REGULATION 56 : Termination or assignment of Articles

REGULATION 65: Articled Assistant not to engage in any other occupation

REGULATION 67 : Complaint against the employer (From articled Assistant)

REGULATION 68-80: Engagement of Audit Assistant

REGULATION 190A: CA’s not be engage in any other business or occupation.

REGULATION 191 : Part time employment a CA may accept

REGULATION 192 : Restriction on Fees

56
FINAL C.A. – AUDIT

POINTS TO BE NOTED :

1) Monthly payment of stipend to every article. It must be confirmed beyond all doubts
that payment has been made.

2) It is duty of CA to forward article deed to ICAI

3) A CA cant’s take loan from any enterprise in which article is interested. However, he
may accept the same from any enterprise where in article’s relative is interested. But it
must not be taken as a consideration for admitting the article into firm.

4) Now, no premium can be accepted by CA from the article

5) Practice work should be performed only after obtaining COP

COUNCIL GENERAL GUIDELINES, 2008:


1) If he accepts more than specific tax audit > 60

Note: According to clarifications by Committee on ESB, if there are 10 partners in a firm,


then all partners of the firm can collectively sign 600 tax audit reports. This max limit maybe
distributed between the partners in any manner. For instance, 1 partner signs 600 tax audit
reports remaining 9 are not signing any reports

2) If company audit assignment > 20

3) A CA in practice, has to maintain BOA including cash book / ledger

4) He shouldn’t be cost auditor of company in which he is disqualified under


141(3) (a) to (d)

5) He can’t be auditor if he is an employee of cost auditor of company.

6) In case of no statue. Indebted / Guarantee > `10,000

7) Accepts statutory audit of PSU / Listed Company / Government Company or other public
company having turnover >50 crore and accepts any other work or service with regard

57
FINAL C.A. – AUDIT

to same undertaking on a remuneration which in total exceeds the fees payable for
carrying out the statutory audit of the same.

8) A member in service shall be deemed to be guilty if he is willfully and grossly negligent in


conduct of his duty as employee.

9) A member in practice shall follow the direction given, by the council or an appropriate
committee or on behalf of any of them, to him being the incoming auditor not to accept
the appointment as auditor, in case the undisputed removal of the earlier auditor.

10) Shall not accept the appointment as auditor of an entity in case the undisputed audit fee
of another auditor has not been paid except in case of sick company.

OTHER SECTOR OF CA ACT, 1949

SEC 4 : Entry of name in registrar.


SEC 5 : Fellow and Associate classes
SEC 6 : Certificate of practice
SEC 7 : Designation CA
SEC 8 : Disabilities (MR. ICU)
SEC 20 : Removal of Name from the Register
SEC 21 : Disciplinary Mechanism
SEC 22 : Professional / Other misconduct
SEC 24 : Penalty for falsely claiming to be a member
SEC 25 : No company
SEC 26: Unqualified person not to sign documents
SEC 27: Maintenance of branch office

MAINTENANCE OF BRANCH OFFICE BY A PRACTICING MEMBER :

How many Any number provided each branch must have separate CA in charge*
branches : or partner in whole time employment of CA concerned.

Incharge: Who either attends the said office or resides in the city where such office is
situated at least for 182 days in a year.

58
FINAL C.A. – AUDIT

EXCEPTIONS : (No in charge)

A) TEMPORARY OFFICE :
a) For members practicing in hilly areas
b) Temporary office may be opened in plains in winter season only for 3 months
c) Temporary office not be mentioned as place of business on professional stationery
d) Correspondence may continue at perm office
e) Before coming to plain and at close of such temporary office inform ICAI.
f) Name board of firm to be displayed at temporary office only during these 3 months

B) SECOND OFFICE : If it is situated


a) In same premises; or
b) In the same city; or
c) Within 50 kms from municipal limits of city in which 1st office is situated.

KYC NORMS
Where Client is an Individual/ Proprietor
A. General Information
Name of the Individual
PAN No. or Aadhar Card No. of the Individual
Business Description
Copy of last Audited Financial Statement

B. Engagement Information
Type of Engagement

2. Where Client is a Corporate Entity


General Information
Name and Address of the Entity Business Description
Name of the Parent Company in case of Subsidiary Copy of last Audited Financial Statement

B. Engagement Information
Type of Engagement
Regulatory Information
Company PAN No. Company Identification No.
Directors’ Names & Addresses Directors’ Identification No.

59
FINAL C.A. – AUDIT

3. Where Client is a Non-Corporate Entity


a. General Information
Name and Address of the Entity
Copy of PAN No.
Business Description Partner’s Names & Addresses
(with their PAN/Aadhar Card/ DIN No.)

Copy of last Audited Financial Statement

a. Engagement Information
Type of Engagement

60
FINAL C.A. – AUDIT

DECISIONS OF ETHICAL STANDARD BOARD

1) A CA in practice
l equity research adviser ()

l publish retail report ()

(as it would amount to other business or occupation )

2) CA member of a Trust cannot be the auditor of the said trust

3) CA in practice engages himself


as Registration Authority (RA) for obtaining digital signatures for clients ()

4) A CA hold credit card of a bank provided the O/S balance on the said card does
not exceed Rs 10,000
beyond the prescribed credit period limit on credit card given to him

5) A CA in practice act as mediator in Court ()


(since acting as a “mediator” would be deemed to be covered within the meaning of
“arbitrator’; which is permitted to members in practice as per Regulation 191 of the
Chartered Accountants Regulations, 1988 )

6) A CA in practice accept audit assignment of a bank in case he has taken loan against
a Fixed Deposit held by him in that bank ()

7) Rule 11U of Income Tax: (Qualified Merchant Banker) Statutory auditor /tax auditor
cannot be the valuer of unquoted equity shares of the same entity.

l The Board has at its Meeting (Jan 2017) decided that where law prohibits such
prohibition on statutory auditor/tax auditor to be the valuer will continue
l but where there is no specific restriction under any law permissible

8) The Ethical Standards Board had decided in 2011 :


l Member who has been Director of a Company resignation from the Co
cannot be appointed as an auditor of the said Company the cooling period for
the same may be 2 years.

61
FINAL C.A. – AUDIT

l The Board has at its recent Meeting (Jan 2017) has reviewed the above, and
noted that the Sec 141 of Companies Act, 2013 on disqualification of auditors does
not mention such prohibition; though threats pertaining to the said eventuality
have been mentioned in Code of Ethics.

l Further, the Board was of the view that a member may take decision in such situation
based on the provisions of Companies Act, 2013 and provisions of Code of Ethics.

9) A CA in practice
l Act as Financial Advisors and receive fees/commission from Financial Institutions

such as Mutual Funds, Insurance Companies, NBFCs etc. ()

10) A CA in practice
l Exercise lien over the client documents/ records for non-payment of his fees. ()

11) CA Firm to print its vision and values behind the visiting cards ()
l It would result in solicitation as per the provisions of Clause (6) of Part-I of First

Schedule

12) A CA in practice
l take agencies of UTI, GIC or NSDL. ()

13) A CA in practice
l Permissible for a member to be a settlor of a trust. ()

14) A CA in practice
l Hold Customs Brokers License under section 146 of the Customs Act, 1962 ()

15) A CA in service
l Appear as tax representative before tax authorities on behalf of his employer ()

l on behalf of other employees of the employer ()

16) A CA statutory auditor of a bank


l for the same financial year accept stock audit of the same branch of the bank or

any of the branches of the same bank or sister concern of the bank, for the same
financial year. ()

62
FINAL C.A. – AUDIT

17) A CA Firm the internal auditor of a PF Trust by a Government Company


l Appointed as its Statutory Auditor ()

18) A concurrent auditor of a bank ‘X’ cannot be appointed as statutory auditor of bank ‘Y’,
which is sponsored by ‘X’

19) A CA/CA Firm the internal auditor of a company


l Statutory auditor of its employees PF Fund under the new Companies Act (2013).
()

20) The ESB while noting that there is requirement for a Director u/s 149(3) to reside
in India for a minimum period of 182 days in the previous calendar year, decided that
such a Director would be within the scope of Director Simplicitor (which is generally
permitted as per ICAI norms),
if he is non – executive director, required in the Board Meetings only, and not paid any
remuneration except for attending such Board Meetings

21) CA in practice cannot act as internal as well as GST auditor of the same entity

CLAUSE 2 :
Being an employee of any company, firm or person,
Discloses confidential information
Acquired in the course of his employment
Except as and when required by law or except as permitted by the employer.

CLAUSE 3 :
Include in any information, statement return or form to be submitted to
The institute, council or any of its committees, director (Discipline), Board of discipline,
disciplinary committee, quality review board or the Appellate Authority. (ICD BDQA)
Any particular knowing them to be false.

CLAUSE 4 :
Defalcates or embezzles money received in his professional capacity

63
FINAL C.A. – AUDIT

PART III : OTHER MISCONDUCT IN RELATION TO MEMBERS OF THE INSTITUTE


GENERALLY.

CLAUSE 1 :
If he is held guilty by any civil or criminal court
For an offence which is punishable for a term exceeding 6 months.

FLOW CHART OF DISCIPLINES PROCEDURE MECHANISM



Complaint against member of ICAI of alleged misconduct along with prescribed fee.

Disciplinary Directorate
The Director (Discipline) shall arrive at a prime facie opinion on the occurrence of alleged
misconduct and decide whether he is guilty of professional or other misconduct falling in

FIRST SCHEDULE SECOND / BOTH SCHEDULE


Place the matter before Place matter before
Board of Discipline Disciplinary / Committee
If found guilty, it can If fund guilty, it can
l Reprimand member l Reprimand member

l Remove the name of member upto a period l Remove the name of member permanently

of 3 months or for any duration


l Impose fine upto 1,00,000 l Impose fine upto 5,00,000

Any member aggrieved by order of Board of Disciplinary committee can prefer an appeal
within 90 days Appellate Authority.

Questions asked in past/rtp/mock test

Question 88:
Mr. S, a practicing CA agreed to provide “Portfolio management Services” to his client M/s.
D Limited. Comment with reference to the Chartered Accountants Act, 1949.
Answer:
P would be guilty of misconduct under the Chartered Accountant Act, 1949 as a practicing
CA is not permitted to render portfolio management services.

64
FINAL C.A. – AUDIT

Question 89:
CA Natraj, in practice, accepted an assignment as advisor and consultant to the public issue
of shares by his client M/s Super Ltd. Besides helping the company as an advisor, he also
underwrote the public issue of the company to the extent of 25% at a commission of 1%.
Remaining shares were underwritten by banks and other financial institutions at the same
rate of commission. He contends that above assignments are part of management consultancy
work permitted by the council of the Institute. Do you agree with the View of CA Natraj ?
Decide in the light of applicable code of conduct.
Answer:
CA Natraj would be guilty of misconduct under the Chartered Accountants Act, 1949 as a
practicing CA is not permitted to render underwriting services.

Question 90:
Mr. A, CA in practice has been suspended from practice for a period of 6 months & he had
surrendered his COP for the said period. During the said period of suspension, though he did
not undertake any audit assignments, he undertook representation assignments for income
tax whereby he would appear before the tax authorities in his capacity as a CA.
Answer:
A would not be allowed to represent before the income tax authorities for the period he remains
suspended. Accordingly, in the present case, he is guilty of professional misconduct.

Question 91:
Mr. Dice, a practicing Chartered Accountant was ordered to surrender his certificate of
practice and he was suspended for one year on certain professional misconduct against him.
During the period of suspension, Mr. Dice, designating himself as GST consultant, did the work
of filing GST returns and made appearance as a consultant before various related authorities.
He contended that there is nothing wrong in it as lie, like any other GST consultant, could take
such work and his engagement as such in no way violate the order of suspension inflicted on
him. Is he right in his contention?
Answer:
Contention of Mr. Dice is correct as long as he is not working in his capacity as a CA, during
the period he was suspended from practice as CA

65
FINAL C.A. – AUDIT

Question 92:
Mr. G, a Chartered Accountant in practice as a sole proprietor has an office in Mumbai near
Church Gate. Due to increase in professional work, he opens another office in a suburb of
Mumbai which is approximately 80 kilometers away from the municipal limits of the city. For
running the new office he employs three retired Income-tax Officers.
Answer:
As the second office Is situated beyond 50 km ;. of municipal limits of Mumbai city, Mr. G
would be liable for committing a professional misconduct

Question 93:
XY & Co., a firm of Chartered Accountant having 2 partners X & Y, one in charge of Head
Office and another in charge of Branch at a distance of 80 kms., puts up a name-board of
the firm in both premises and also in their respective residences.
Answer:
The chartered accountants are guilty of misconduct as name board of the firm cannot be put
in place of residence.

Question 94:
Mr. Dheeraj, an aspiring student of ICAI, approached Mr. Murli, a practicing CA, for the
purpose of articleship. Mr. Murli, the principal, offered him stipend at the rate of Rs. 2,000
p.m. to be paid every 6th month along with interest at the rate of 10% p.a. compounded
monthly to compensate such late payment on plea that cycle of professional receipts from
clients is 6 months. Mr. Dheeraj agreed for such late payment in the hope of getting extra
stipend in the form of interest. Mr. Murli, however, used to disburse salary to all of his
employees on time.
Answer:
Mr. Murli (Mr. X) has violated the Regulation 48 of CA Regulations, 1988, hence would be
guilty of professional misconduct under Clause 1, Part II of Second Schedule.

Question 95:
The manager of ABC (P) Ltd. approached CA, X in the need of a certificate in respect of a
consumption statement of raw material. Without having certificate of practice (COP), CA. X
issued the certificate to the manager of the company, acting as a CA in Practice and applied
for the COP to the Institute on very next day to avoid any dispute.

66
FINAL C.A. – AUDIT

Answer:
Mr. Murli has violated the provisions of Sec. 6 of CA Act, 1949 and hence would be guilty of
professional misconduct under Clause 1, Part-II of Second Schedule

Question 96:
PQ & Co., a firm of Chartered Accountants, included the name of Ras a partner while filing
application for empanelment as auditor for Public Sector banks branches. It was subsequently
noticed that on the date of application, R was not a partner with PQ & Co.
Answer:
CA P & CA Q will be held guilty of professional misconduct as per Clause 3, Part II of Second
Schedule for submitting false information with the Institute.

Question 97:
Mr. Brilliant, a practicing CA received a major professional assignment. to complete the said
assignment he was required to buy four computers. Due to his inability to provide funds for
acquiring the same he borrowed money from a firm, where one of the articled clerk and his
father were interested. What will be the Chartered Accountants liability.
Answer:
Accepting a loan from an articled clerk in case of an engagement of an article clerk is prohibited
under the Regulations. But, in the present case, it appears from the facts that the articled
clerk is already been engaged and serving under him, and thus, Mr. Brilliant will not be held
guilty of professional misconduct under Clause 1 of Part II of Second Schedule of CA Act,
1949.

Question 98:
X, a practicing Chartered Accountant in an application for permission to study submitted
by his Articled Assistant to the Council had confirmed that the normal working hours of his
office were from 11 A.M. to 6 P.M. and the hours during which the Articled Assistant was
required to attend classes were 7.00 A.M. to 9:30 A.M. According to the information from
College, the Articled Assistant attended the College from 10 A.M. to 1.55 P.M. on all week
days. About the Articled Assistant attending the classes even during office hours, X pleaded
ignorance.
Answer:
Mr. X will be deemed to be guilty of professional misconduct under Clause 1 of Part II of
Second Schedule for contravention of regulations of the Institute and under Clause 3 of Part
II of Second Schedule for submission of false information to the ICAI.

67
FINAL C.A. – AUDIT

Question 99:
Mr. Brilliant, a practicing CA received a major professional assignment. to complete the said
assignment he was required to buy four computers. Due to his inability to provide funds for
acquiring the same he borrowed money from a firm, where one of the articled clerk and his
father were interested. What will be the Chartered Accountants liability.
Answer:
Accepting a loan from an articled clerk in case of an engagement of an article clerk is prohibited
under the Regulations. But, in the present case, it appears from the facts that the articled
clerk is already been engaged and serving under him, and thus, Mr. Brilliant will not be held
guilty of professional misconduct under Clause 1 of Part II of Second Schedule of CA Act,
1949.

Question 100:
X, a practicing Chartered Accountant in an application for permission to study submitted
by his Articled Assistant to the Council had confirmed that the normal working hours of his
office were from 11 A.M. to 6 P.M. and the hours during which the Articled Assistant was
required to attend classes were 7.00 A.M. to 9:30 A.M. According to the information from
College, the Articled Assistant attended the College from 10 A.M. to 1.55 P.M. on all week
days. About the Articled Assistant attending the classes even during office hours, X pleaded
ignorance
Answer:
Mr. X will be deemed to be guilty of professional misconduct under Clause 1 of Part II of
Second Schedule for contravention of regulations of the Institute and under Clause 3 of Part
II of Second Schedule for submission of false information to the ICAI

Question 101:
Ms; P. a Chartered Accountant, did not maintain any books of account on the ground that
his income did not exceed the limits prescribed u/s 44AA of the Income-tax Act, 1961.
Answer:
Mr. P will be held guilty of professional misconduct by virtue of Clause 1 of Part I) of Second
Schedule, due to contravention of Chapter V of Council General Guidelines, 2000 for non-
maintenance of books of account.

68
FINAL C.A. – AUDIT

Question 102:
Mr. Gaurav is a partner in M/s XYZ & Co., a firm of Chartered Accountants with 6 partners.
During the assessment year 2019-20, Mr. Gaurav alone had signed 290 tax audit reports
consisting of both corporate and non-corporate assesses. Comment.
Answer:
Mr. Gaurav is eligible to sign 290 tax audit reports on behalf of the firm as the eligibility of
the firm is to accept 360 tax audits, ( Refer Chapter VI of Council General Guidelines, 2008).

Question 103:
A is the auditor of Z Ltd., which has a turnover of Rs. 200 crores. The audit fee for the year
is fixed at Rs. 50 lakhs. During the year, the company offers A an assignment of management
consultancy within the meaning of Section 2(2) (iv) of the CA Act, 1949 for a remuneration
of Rs.1 crore. A seeks your advice on accepting the assignment.
Answer:
It would be a misconduct on As part due to breach of Chapter IX of Council General Guidelines
2008 as he accepts the management consultancy assignment at a fee higher than audit fees.

69
FINAL C.A. – AUDIT

COMPANY AUDIT FINAL CA

COMPANY ACT, 2013

Sec 141(1)& 141(2) : Eligibility & qualification ( Jksc Pg 89 )

Company Auditor

Proprietor PF (Incl LLP)

Majority
CA + COP Partner CA +
COP


CA PIYUSH GOYAL CA KUMAR MANGALAM BCOM DISHA
BIRLA PATNI

Questions asked in past/rtp/mock test


Question 1:
Praveen, a member of the ICAI, does not hold a certificate of practice. Is his appointment as
an auditor valid.
Answer:
Praveen does not hold a certificate of practice and hence cannot be appointment as an auditor
of the company.
70
FINAL C.A. – AUDIT

Sec 141(3): Disqualifications ( Jksc Pg 90 )


a) body corporate other than LLP
b) an officer or employee of the company
c) a person who is a partner or who is in employment of an officer or employee of the
company
d) Person/ partner / relative
Any security Indebtness to Guarantee to
in CHASS CHASS > 5 lac CHASS > 1 lac

Note 1: Relative can hold security Upto Rs.1,00,000


(All relatives in agg)

Note 2: Grace period of 60 days for corrective action shall apply only in respect of securities
held by relative. This would not apply to auditor or his partner.

Note 3: Relative can hold security in Company only and not in HASS as per bare reading.
No question asked so far on this controversial point.

Note 4: As per GN issued by ICAI, audit fees received onprogressive basis i. e after beginning
the engagement is not treated as an advance of the fees.

Note 5: Definition of Relative 2(77)


s Member of HUF
s Spouse
s father (Including step)
s Mother (Including step)
s Brother (Including step)
s Sister (Including step)
s Son (Including step)
s Daughter
s Son’s wife
s Daughter’s husband

71
FINAL C.A. – AUDIT

IMPORTANT CONCEPT
A lot of questions has been asked on the same!!!



Questions asked in past/rtp/mock test
Question 2:
“Mr. A” a practicing Chartered Accountant, is holding securities of “XYZ Ltd.” Having face
value of RS 900. Whether Mr. A is qualified for appointment as an auditor of “XYZ Ltd”?
Would your answer be different, if instead of Mr. A: Mr. B the Step father of Mr. A. holding
the securities?
Answer:
Mr. A is holding security of RS 900 in the XYZ LTD, therefore he is not eligible for appointment
as an auditor of “XYZ LTD”. However, in the second case, Mr. A is eligible, as relative may
hold securities of face value up to RS 1 lac.

Question 3:
A, a Chartered Accountant has been appointed as auditor of Laxman Ltd. In the AGM of
the company held in September2018, which assignment he accepted. Subsequently in January,
2019 he joined B, another Chartered Accountant, who is the manager Finance of Laxman
Ltd. as partner.
Answer:
In the present case A, an auditor of Laxman Ltd, joined as partner with B, who is manager
finance of Laxman Limited, will be disqualified by sec 141(3)(c)

Question 4:
An auditor purchased goods worth RS 5,01,500 on credit from a company being audited by
him. The company allowed him one month’s credit, which it normally allowed to all known
customers.
Answer:
In instant case, auditor has become indebted to the company and consequently he has deemed
to have vacated his office.

72
FINAL C.A. – AUDIT

Question 5:
Mr.Aditiya, a practicing Chartered Accountant is appointed as a “tax consultant” of ABC
LTD, in which his father Mr.Singhvi is the Managing director?
Answer:
Mr. Aditiya can be appointed as a tax consultant irrespective that his father is the managing
director of the company.

Question 6:
Ram & Hanuman Associates, CA in practice have been appointed as Statutory Auditor of
Krishna Ltd. For the year 2019-20. Mr. hanuman holds 100 equity shares of Shiva Ltd, a
subsidiary company of Krishna Ltd.
Answer:
Disqualified, because one of the partner Mr. Hanuman is holding equity shares of its subsidiary.

Question 7:
CA Mr. X hey was indebted to ABC limited for a sum of rupees 5,00,000 as on 1/4/2019.
however Mr. X having come to know that he might be appointed as auditor of the company
, he squared the amount on 10/07/2019. Later on, he was appointed as an auditor of the
company for the year ended 31/3/2020 at the annual general meeting held on 16/07/2019.
Subsequently, one of the shareholders complaints that the appointment of Mr. X is invalid
because he incurred disqualifications under section 141 of the Companies Act, 2013. Comment.
Answer:
Appointment of Mr.X is valid as no disqualification attracts as on the date of appointment

Question 8:
Mr. Y, a practicing CA, has been appointed as an auditor of said limited on 12th June 2018
for the year ended 31st March 2019 .
Daughter of Mr. Y Purchased securities on 10th September 2018 of FV Rs. 45,000 (MV Rs.
90,000)
Husband of daughter of Mr. Y purchased securities on 10th December 2018 of FV Rs.
90,000 (MV Rs.1,90,000)
All the above share were sold on 10th March 2019 for rupees 3,00,000. Comment.
Answer:
Disqualified on expiry of 60 days from 10th Dec 2018 as he fails to take corrective action.

73
FINAL C.A. – AUDIT

SEC 141(3): DISQUALIFICATIONS

e) Person/firm Business Relationship CHASS

Exception 1: Commercial transaction which are in nature of services rendered by auditor


under CA Act 1949

Exception 2: ALP transaction with HATH


s H- Hospitals
s A-Airlines
s T- Telecommunications
s H- Hotels

SEC 141(3): DISQUALIFICATIONS

GOODS BOUGHT ON CREDIT


> Rs 5,00,000 < Rs 5,00,000

Disqualified u/s
141(3)(d) On ALP Not ALP

Not disqualified Disqualified u/s 141(3) (e )

74
FINAL C.A. – AUDIT

QUESTIONS ASKED IN PAST/RTP/MOCK TEST


Question 9:
PQ and Co. is an audit firm with P and Q as partners. For the financial year 2018- 19, the
firm has been appointed as the statutory auditor of M/s Mango Orchards Hotel Ltd. The
audit firm is a regular customer of the hotel and the partners usually stays in the same
hotel at various location in the course of travelling for their various professional assignments.
Normally payments of such stay are settled against quarterly bills raised by the company.
Answer:
No disqualification arises as the services availed are in ordinary course of business of client

SEC 141(3): DISQUALIFICATIONS


f) Person Relative DIR/KMP Co.

g) Audit of >20 company except (PODS)


s P- Pvt. Co have PSC < 100cr
s O- One person co.
s D- Dormant Company
s S- Small Company

SEC 141(3): DISQUALIFICATIONS

Note1 : The above exemption is not available if the co. has not filed its annual statement
u/s 137 & 92of Co. Act 2013.

Note2 : If there are 3 partner, limit will be 60 & accordingly.

Note 3: Where any partner is also holding office in individual capacity or partner at various
firms number of assignments shall not exceed 20

Note 4: Number of partner of a firm on date of acceptance of audit assignment shall be


taken into account

Note 5: A CA in full time employment elsewhere shall not be taken into account

Note 6: Cant divide like in case of limit of 60 in tax audit

75
FINAL C.A. – AUDIT

QUESTIONS ASKED IN PAST/RTP/MOCK TEST


Question 10:
Can ABC & Co. accept the appointment as an auditor in 60 private companies having paid- up
share capital less than Rs.100 Cr. Which has not committed default in filling its financial
statement under section 137 or annual return under section 92 of the Companies Act with
the Registrar, 2 small companies and 1 dormant company?
Answer:
No disqualification arises as the services availed are in ordinary course of business of client
and cannot be considered as business relationship.

Question 11:
KBC & Co a firm of CA are 3 partners K,B & C, K Is also in Whole time employment elsewhere.
the form is offered the audit of ABC limited and is already holding audit of 40 companies.
comment
Answer:
Cant accept the offer for audit of ABC LTD.

Question 12:
Mr. A signs the balance sheet of 10 small companies and 10 private companies having paid up
share capital of less than RS 100 crores. Mr. B signs the balance sheet of 10 private company
having paid up share capital of less than RS 100 crore and 5 private companies having paid
up share capital of more than RS 100 crore. Mr. C signs the balance sheet of 10 private
companies having paid up share capital of more than RS 100 crore and 5 public companies.
what is the maximum number of audits that the firm as a whole can accept and what is the
maximum number of audits each individual partner can accept?
Answer:
l Firm can accept 40 more audit of public companies and private companies having paid

up capital of more than RS 100 crore. Audit of small companies and private companies
having paid up share capital less than RS 100 crore. are not considered for the purpose
of ceiling.

l Partner A can accept 20 audit of public companies and private companies having paid up
capital of more than RS 100 crore. Partner B can accept 15 audits of public companies
and private companies having paid up capital of more than RS 100 crore.

76
FINAL C.A. – AUDIT

l Partner c can accept 5 audits of public companies and private companies having paid
up capital of more than RS 100 crore. Audit of small companies and private companies
having paid up share capital less than RS 100 crore. are not considered for the purpose
of ceiling.

SEC 141(3): DISQUALIFICATIONS


h) Convicted by court for fraud 10years have not elapsed

Only individual gets disqualified, not whole Partnership firm

I) Is engaged in consulting & specialized service as provided u/s 144

Sec144: Prohibited Services ( MA3RDI3)


M- Management service
A1- Accounting & book keeping services
A2- Actuarial Services
A3- Any other kind of services
R- Rendering of outsourced services
D-Design & implementation of any financial Info
I1- Internal Audit
I2-Investmebt advisory services
I3- Investment banking services

QUESTIONS ASKED IN PAST/RTP/MOCK TEST

Question 13:
M/s Duster & co. chartered accountants, appointed as a statutory auditor of R Ltd. for the
financial year 2019-20.The company is also need of some actuarial services. consequently, the
board of directors of the company offered the same to M/s Srivastava & co., an associate to
M/s duster & co. which has been duly accepted by the firm. Comment.
Answer:
M/s Duster & co. Becomes disqualified u/s 141(3)(I) of companies Act, 2013 and needs to
vacate the office as required u/s141(4).

77
FINAL C.A. – AUDIT

SEC 139: APPOINTMENT OF AN AUDITOR OF GOVERNMENT CO.( PG 94)


SEC 139(7): FIRST AUDITOR SEC 139(5): SUBSEQUENT AUDITOR

C& AG shall appoint within 60days C&AG shall appoint an auditor an


of date of registration auditor within 180 days of the
commencement of financial year
Failing which BOD shall appoint
within 30 days Tenure up to Next AGM

Failing which SH’S shall appoint
within 60 days

Tenure up to 1ST AGM


NOTE 1: C&AG means Comptroller and Auditor of India. He has the same status as
that of Chief Justice of India

NOTE 2: In case the SH’s fail to appoint the 1st auditor for a govt. co the government co
will also have to pay penalty.

NOTE 3: For a govt. co, the subsequent auditor shall be appointed every year, i.e concept of
rotation is not applicable.

NOTE 4: If no new auditor is appointed, in absence of any provision existing auditor shall
continue to hold office.
If existing auditor does not want to hold the office, it will lead to casual vacancy.

78
FINAL C.A. – AUDIT

SEC 139: APPOINTMENT OF AN AUDITOR OF NON GOVT CO.


SEC 139(6): FIRST AUDITOR SEC 139(1): SUBSEQUENT AUDITOR

BOD shall appoint within 30days BOD recommends


of date of registration
SH’S shall apoint susbequent
Failing which SH’S shall appoint auditor at 1ST AGM
within 90 days in an EGM
Who shall continue to hold office
Tenure up to 1 AGM
ST
up to the conclusion of 6th AGM

Tenure 5 AGMs




NOTE 1: SH’s can appoint by passing ordinary resolution

NOTE 2: Notice period for EGM is included in 90 days

NOTE 3: The Co. shall file form ADT-1 with ROC within 15 days from AGM

NOTE 4: Obtain written certificate from auditor stating


s Auditor is qualified
s Appointment is as per the act
s Appointment is within limits
s Details of pending proceedings against him or his partner

79
FINAL C.A. – AUDIT

SEC 177: AUDIT COMMITTEE


The following companies need to constitute an audit committee:
s Listed Company
s Public Company if
s PUSC ≥ 10cr or
s T/O ≥ 100 cr or
s Borr ≥ 50 cr


Majority of members have to be independent

WHAT DOES AUDIT COMMITTEE DO? (SARI)


S – Determining scope of an auditor
A – Adoption & review of accounting policies
R- Reviewing related party transactions
I – Appointing Internal Auditor

SEC 139(11): MANNER OF SELECTION V

IS THE COMPANY SEC 177 CO


YES NO

The Audit Committee BOD shall appoint/
shall recommend the name recommend
of the auditor to the BOD


BOD Agrees BOD Disagrees

AC’s auditor gets appointed/ The BOD shall state its reasons
recommended & give their own recommendations
80
FINAL C.A. – AUDIT

The BOD shall state its reasons &


give their own recommendations


AC agrees AC disagrees

BOD’s auditor gets BOD shall appoint/
appointed/recommended recommend

BOD’s auditor gets appointed/
recommended. The BOD shall submit a
statement in writing to SH reasons for
disagreement


SEC 139(2): EXTERNAL ROTATION
APPLICABILITY



Listed Public Co Pvt Co Borrowings

PUSC ≥ 10cr PUSC ≥ 50cr ≥ 50cr

Excluded:
l One person company

l Small co.

81
FINAL C.A. – AUDIT

IS ROTATION APPLICABLE


Yes No
Auditor including network firms Auditor can be appointed for “N ” n
cannot be reappointed for a period umber of years
of 5 years after the term allowed

What is the term allowed


Proprietor Partnership Firm

1 term of 2 consecutive tenures
5 years of 5 years


Network Firm:
s Common Partner of 2 or more firms
s Firms under the same brand name

Cooling Period:
5 years from the date of expiry of term allowed.

If a partner, who is in charge of an audit firm &


also certifies the FS of the company, retires
from the said firm & joins another firm of
chartered accountants, such other firm shall also
be ineligible. (Cant say we r on a break)

82
FINAL C.A. – AUDIT

SEC 139(4): TRANISITIONAL PROVISION


Grace period of 3 years allowed to comply with the provisions of sec 139( 2)

Period Already Served Grace Period Total


10 3
9 3
8 3
7 3
6 3
5 3
4 3
3 3
2 3
1 3

SEC 139(3): INTERNAL ROTATION (PG 97)

The company may resolve that in the audit firm appointed by it, the partners and his audit
team shall be rotated as may be resolved by the members of the company

SEC 139(8): CASUAL VACANCY

Non Govt Co Govt Co

Due to Due to any C&AG shall appoint


resignation other reason within 30 days

BOD shall file the BOD shall appoint an If failed, BOD shall
vacancy within 30 days auditor within 30 days appoint within 30 days

Members have to approve


within 3 months

83
FINAL C.A. – AUDIT

SEC 139(9): REAPPOINTMENT OF A RETIRING AUDITOR


The retiring auditor may be reappointed. Provided that all of the following conditions are
compiled with:
l The auditor is not disqualified u/s 141

l The auditor has not shown his written unwillingness for reappointment

l A special notice has not been received by the company that

l The retiring auditor shall not be reappointed

l Specifically stating that some other auditor shall be appointed in place of the

retiring auditor

SEC 139(10): CONTINUANCE IN THE OFFICE OF THE AUDITOR


If in any AGM, the retring auditor has not been reappointed & no new auditor has been
appointed in his place, the retiring auditor shall continue to hold office of the auditor.

QUESTIONS ASKED IN PAST/RTP/MOCK TEST


Question 14:
KM PVT LTD. Engaged in the manufacturing business of silk Shirts, is a newly incorporated
company dated 1-9-2019. On 28-9-2019, the members of KM PVT LTD. Themselves appointed
CA Raj, a renowned practitioner, as the first auditor of the company apposing that board
is not authorized to appoint the auditor. You are required to comment on the action of the
members.
Answer:
Appointment of CA Raj by the Members of the company is not in order as per provisions of
section 139(6) of companies Act, 2013.

Question 15:
THE First auditor of M/s Healthy Wealthy LTD. A government company, was appointed by
the Board of directors.
Answer:
The appointment of first auditors made by the board of directors of M/s Healthy Wealthy
LTD is invalid

Question 16:
At the AGM of ICI LTD, Mr. X was appointed as the statutory auditor. He, however, resigned
after 3 months since he wanted to give up practice and join industry. State, how the New

84
FINAL C.A. – AUDIT

auditor will be appointed by ICI LTD. And the conditions to be compiled for
Answer:
In this case the casual vacancy has been created on account of resignation. Therefore, board
of directors will have to fill the vacancy within thirty days and such appointment shall be
approved by the company at the general meeting within three months of the recommendations
of the board, The new auditor so appointed shall hold office only till the conclusion of the Next
AGM.

Question 17:
M/S Young & co. chartered Accountant firm, a statutory Auditors of Old LTD.is dissolved on
1-4-2019 due to differences of opinion among the partners. The board of directors of Old
LTD. In its Meeting on 6-4-2019 appointed another Firm M/S sharp & co. as their New
auditors for one year.
Answer:
In the instant case the action of the board of Directors in appointing M/S sharp
co. to fill up the casual vacancy due to dissolution of M/S young & co. is correct.
Howevwe, the board of directors are not correct in giving them appointment for one year.
M/S sharp & co. can hold office until the conclusion of next AGM only.

Question 18:
X LTD. Is an unlisted public company. Its balance sheet shows paid up share capital of RS 7.5
crore and public deposits of RS 70 crore. The company appointed M/S ABC & co, a CA firm,
as the statutory auditor in its annual general meeting held at the end of september,2019 for
11 years. Comment.
Answer:
Company is covered u/s 139(2) as deposits exceeds RS 50 crore,so appointment can be only
for 1 term of 5 years( in case of individual) and 2 terms of 5 years each ( in case of firm).

Question 19:
C.A. Ashwin was a appointed as auditor of Bristol LTD. For the year 2019-20. Since he
declined to accept the appointment, the board of directors appointed C.A. John as the auditor
in place of C.A. Ashwin and the appointment was accepted by C.A john Discuss.
Answer:
Board of directors are not authorized to fill up the vacancy in case the auditors appointed at
AGM Refuse to accept the appointment.

85
FINAL C.A. – AUDIT

Question 20:
CA.X is a partner in M/s AB & Associated and M/s MN & Associates simultaneously. M/s
AB & Associates has completely its tenure of 10 years as an auditor in XYZ Ltd. Immediately
preceding the current financial year. It may be noted that the provisions for applicability of
rotation of auditors are applicable to XYZ Ltd. Now, the company wants to appoint M/s MN
& Associates as auditor for 5 years.
Whether M/s MN & Associates is allowed to accept the appointment as auditor of XYZ Ltd?
Would your answer be different from above if CA. X being in- charge of M/s AB & Associates
and certifying authority of financial statement of XYZ Ltd., retries from the partnership in
M/s AB & Associates and Joins M/s MN & Associates?
Answer:
(a) Not allowed due to rotation provision of sec 139(2):
(b) Not allowed

Question 21:
ABC.Pvt. Ltd., a new company, incorporated on 1-7-2019 is engaged in the manufacturing
business. On 30-7-2019, the Managing director of ABC Pvt Ltd. himself appointed CA Mohan,
his daughter’s husband, as the first auditor of the company. you are required to-
State the provisions of the companies Act, 2013 relating to appointment of first auditor.
Comment on the action of the Managing director.
Answer:
Appointment of Mr. Mohan is not valid as per provision of section 139(6) of the companies
Act,2013.

SEC 140(1): REMOVAL OF AN AUDITOR BEFORE EXPIRY OF HIS TENURE



Call for a BOD meeting & pass Board resolution

File form ADT-2 within 30 days with CG for its prior approval

Call for an EGM within 60 days of approval of CG

At EGM members shall pass special resolution

Auditor shall be given reasonable opportunity of being heard

86
FINAL C.A. – AUDIT

SEC 140 (2): RESIGNATION BY AUDITOR


The auditor who has resigned from the co. shall file within a period of 30 days
From date of resignation in form ADT – 3 indicating reasons and other facts to
l Registrar

l Company

l C&AG in case of govt co

SEC 140(3): PENALTY FOR NON COMPLIANCE OF 140 (2)


l Rs. 50,000 or

l An amount equal to the remuneration of the auditor

l whichever is less

SEC 140(4): REMOVAL OF AN AUDITOR AFTER EXPIRY OF HIS TENURE



If members have given a special notice stating that

l The retiring auditor shall not be reappointed or

l Specifically stating that other auditor shall be appointed in place of the retiring auditor


The co shall forward this to the auditor & seek his reply. The auditor can give his written
explanation which the co. needs to circulate to all its members along with a copy of the special
notice at least 7 days prior to AGM

If the co. fails to circulate the auditor can read his representation to the AGM

Note 1: If the tribunal is satisfied on an application either of the company or any other
aggrieved person that the rights are being abused by the auditor, then , the copy
of the representation may not be sent & representation need not be read out at
the meeting.

Note 2: Special Notice u/s 115


It can be sent by member or members holding 1% of voting power or shares with
paid up value of at least Rs.5,00,000

87
FINAL C.A. – AUDIT

SEC 140(5): REMOVAL OF AN AUDITOR BY TRIBUNAL



In case an Auditor acted in a Fraudulent Manner

The tribunal may, by order, direct the co. to change its auditors
Either suo motto or
l On an application made to by CG

l by any person concerned


Auditor shall not be eligible to be appointed as an auditor of any co. for a period of 5 years
from date of passing of the order and shall be liable for action u/s 447

If application is made by CG & tribunal is satisfied, it shall within 15 days of receipt of such
application make an order that he shall not function as an auditor & CG may appoint another
auditor

QUESTIONS ASKED IN PAST/RTP/MOCK TEST

Question 22:
PQR Company LTD. Removed their first auditor by passing a resolution in the meeting of
the board of directors for his removal without obtaining prior approval from the central
government. Offer your comments in this regard.
Answer:
Removal of Auditor is invalid as special resolution has not been passed and approval of central
Govt. not obtained.

SEC 142 REMUNERATION OF AN AUDITOR



1st Auditor: Shall be fixed by BOD
Subsequent Auditor: Shall be fixed in its GM

Remuneration includes expenses, if any incurred in connection with
the audit of the company

It does not include any remuneration paid to him for any other
services rendered by him at the request of the co.

88
FINAL C.A. – AUDIT


As per Sch III of Companies Act 2013 detailed information
of amount paid to be given. These may be classified as

a) As Auditor
b) For taxation matters
c) For company law matters
d) For management services
e) For other services
f) For reimbursement expenses

RIGHTS OF AN AUDITOR

143(1): Right to access 142: Right to receive 146: Right to attend a


to BOA, records & remuneration GM (in person or proxy)
documents of the co at all
times including branch & 140(1)(4): Right to be 177(7): Right to attend
subsidiary at anytime heard in an AGM at on Audit committee
which he is being removed meeting in which a/c
143(1): Right to enquire are discussed
from any employee or officer 146: Right to receive
of the co. or any 3rd party all notices and
having a business relation communications for
with the co. general meeting

Note 1: Right of Lien


As clarified by council , CA cannot exercise right of lien on books of accounts on non
receipt of Fees.

89
FINAL C.A. – AUDIT


As per Sch III of Companies Act 2013 detailed information
of amount paid to be given. These may be classified as

a) As Auditor
b) For taxation matters
c) For company law matters
d) For management services
e) For other services
f) For reimbursement expenses

RIGHTS OF AN AUDITOR

143(1): Right to access 142: Right to receive 146: Right to attend a


to BOA, records & remuneration GM (in person or proxy)
documents of the co at all
times including branch & 140(1)(4): Right to be 177(7): Right to attend
subsidiary at anytime heard in an AGM at on Audit committee
which he is being removed meeting in which a/c
143(1): Right to enquire are discussed
from any employee or officer 146: Right to receive
of the co. or any 3rd party all notices and
having a business relation communications for
with the co. general meeting


Note 1: Right of Lien
As clarified by council , CA cannot exercise right of lien on books of accounts on non
receipt of Fees.

90
FINAL C.A. – AUDIT

DUTIES OF AN AUDITOR
143(1): DUTY TO ENQUIRE UPON CERTAIN MATTERS
(LT DIES)
L- Loans and advances
l Have been properly secured &

l Whether T&C are prejudicial to interest of members

T- Transactions
l Whether transactions are represented by mere book entries

l Whether transactions are prejudicial to interest of members

D- Deposits
l whether loans & advances made by company

l have been shown as deposits.

I-Investments
l where the company is not an investment company or banking co

l Whether shares, debentures and other securities

l Have been sold at a price less than at which they were purchased by the company

E-Expenses
l Whether personal expenses have been

l Charged to revenue accounts

S-Shares
l Where it is stated in the books and documents of the company that any shares have

been allotted for cash,


l Whether cash has actually been received, whether the position as stated in the accounts

books and the balance sheet is correct, regular and not misleading.

DUTIES OF AN AUDITOR
143(3): DUTY TO REPORY UPON CERTAIN MATTERS
(ABCDEF-O)

A- All the info & explanations which to the best of his knowledge & belief were necessary.

91
FINAL C.A. – AUDIT

B- Whether in his opinion proper books of accounts as required by law have been kept.

C- Whether to company’s B/S & P/L dealth with in the report are in confirmation with the
books of accounts

D- Whether in his opinion, the financial statements are done with Accounting standards

E- Eligibility-Whether any director has been disqualified from being appointed as a director
under subsection(2) of sec 164

F- Whether the company has adequate internal financial controls with reference to financial
statements

O- Other matters to be included in the auditors report


(a) Whether the company has disclosed the impact, if any of the pending litigations or
financial position

(b) Whether the company has made provision, as required under any law or AS, for material
forseeable losses if any

(c) Whether there has been any delay in transferring any amounts, required to be transferred
to the Investor Education and Protection Fund by the co.

(d) Whether the co. had provided any requisite disclosures in its financial statements as to
holdings as well as dealings in specified bank notes during the period from 8th November
to 30th December 2016

143(2): Reporting over financial statements to members

143(4): Reasons to be given for qualifications

143 (5) : Auditor of govt. co to submit report to C&AG

143(6): C&AG shall within 60 days from receipt of audit report have right to conduct
supplementary audit by such person or any other person in this behalf

92
FINAL C.A. – AUDIT

l And for the purpose of such audit, require information or additional information to be
furnished to any person so authorised

143 (9) : Duty to comply with auditing standards

143(11): Duty to report upon any other matter prescribed by CG

QUESTIONS ASKED IN PAST/RTP/MOCK TEST

Question 23:
Directors of T LTD. draws an advance of US $200 per day in connection with the foreign trip
undertaken on behalf of the company. On his return he files a declaration stating that entire
advance was expended without any supporting or evidence. T LTD. books the entire expenses
on the basis of such declaration. As the auditor of T LTD. how do you deal with this?
Answer:
Auditor is required to inquire whether the payment made by the company for the foreign trip
is personal expense or not and collect the necessary supporting evidences. If it appears to be
personal expense, auditor is required to ascertain whether such expense is properly authorized
or not. If not authorized, auditor should state the matter in his report.

Question 24:
The auditor of X LTD. Did not report on the matters, specified u/s 143(1) of the companies
Act, 2013, on which he inquired into, because of the reason that he was satisfied. But the
Management of the company wanted the auditor to report on those matters so that the
members can also be aware of the true position of the company. Comment as to whether
the auditor is required to report the matters, specified under the Act, he inquired into and
whether the contention of the management is sustainable.
Answer:
The auditor of X LTD. Is correct in non-reporting on the matters specified in sec. 143(1) of
the act and hence, the contention of the management is not suitable.

143(12): DUTY TO REPORT FRAUD TO CG (PG 110)

1. CA Gabbar FRAUD Report Immediately AC or BOD seeking


(max 2 days) reply within 45 days

93
FINAL C.A. – AUDIT

2. IF REPLY RECEIVED
Report
CA Gabbar ≤ 15 days* CG Reply

Comments

* Count from the date of reply

3. IF REPLY RECEIVED
≤ 15 days* Report
CA Gabbar CG Fact of non receipt of reply

* Count from end of 45 days
4. The report shall be sent to The Secretary, Ministry of Corporate Affairs, in a sealed
cover by RPAD or by speed post followed by an email in confirmation of the same

5. File form ADT – 4

6. Responsibility is on CA/CS/CMA

7. Penalty :
Min: 1,00,000
Max: 25,00,000
GUIDANCE NOTE BY ICAI
Fraud Detected

US Others ( CA/CMA)

<1 crore ≥1 crore <1 crore ≥1 crore

AC/BOD AC/BOD Generally no duty Generally no duty

CARO Reply CARO Whether fraud
remediated
CG – ADT4

CARO

94
FINAL C.A. – AUDIT

WHETHER FRAUD REMEDIATED


SATIESFIED UNSATIESFIED

No Duty Request management to
perform additional procedure

CARO
DONE NOT DONE

CARO Report to CG

CARO

SEC 143(8): BRANCH AUDIT (PG 112)


Accounts of branch office shall be audited either by
l The auditor of the company or

l Any person qualified as an auditor u/s 139 or

l Where branch is situated outside India, the A/c of the branch may be audited by person

qualified to act as auditor in that country

SEC 145: DUTY TO SIGN AUDIT REPORT


The person appointed as an auditor of the co. shall sign the auditor’s report or sign or certify
any other document of the co.

QUESTIONS ASKED IN PAST/RTP/MOCK TEST

Question 25:
X Ltd. has a branch in Malaysia. The company has appointed Mr. X, who is qualified to audit
accounts as per Malaysia laws. Mr. Z, the statutory auditor objects to the same, contending
that he alone can audit the branch office accounts. Discuss.
Answer:
Mr. Z contention that he alone can audit the branch office accounts is not valid.

95
FINAL C.A. – AUDIT

Question 26:
M/s. Seeman & Co. had been the company auditor for Amudhan Company Ltd. For the year
2018-19. The company had three branches located at Chennai, Deli and Mumbai. The audits
of branches Chennai, Delhi were looking after by the company auditors themselves. The audit
of Mumbai branch had been done by another auditor M/s.Vasan & Co., a lo0cal auditor
situated at Mumbai. The branch auditor had completed the audit and had given his report
too. After this, but before finalization, the company auditor wanted to visit the Mumbai
branch and have access to the inventory records maintained at the branch. The management
objects to this on the grounds of the company auditor is transgressing the scope of audit
areas agreed. Comment.
Answer:
Managements objection that the company auditor is transgressing the scope of audit areas
agreed, is absolutely, wrong. The right of company visiting and accessing the records of branch
cannot be forfeited. Even where the branch accounts are audited by another local auditor, the
company auditor has right to visit the branch and can have access to the books and vouchers
of the company maintained at the branch office.

SEC 147: PENALTIES (PG 111)


PENALTY UNDER CO’S ACT,2017 (FOR VIOLATING 139-146)

For Company For Auditor For Officer in default

Unintentional offence Intentional offence

Fine: Fine: Fine: Fine:


Min: 25,000 Min: 25,000 Min: 50,000 Min: 10,000
Max: 5,00,000 Max: Max: 25,00,000 Max: 1,00,000
5,00,000 Or 8 times Or
Or 4 times remuneration imprisonment
remuneration + imprisonment upto 1 year
upto 1 year Or
both

96
FINAL C.A. – AUDIT

SEC 148: COST AUDIT ( PG 116)


RULE 3: (Cost records & audit) CRA Rules, 2014 – COST RECORDS

COMPANY (DOMESTIC + FOREIGN


REGULATED
NON-REGULATED
1. Sugar & industrial alcohol 1. Machinery used for defense
2. Generation, transmission & space atomic research
distribution of electricity 2. Turbo jets & turbo propellers
3. Telecommunication service 3. Arms & ammunition
4. Petroleum products 4. Aeronautical services
5. Drugs & pharma 5. Steel & cement
6. fertilizers 6. Rubber & allied products
7. Roads & other infra
projects
8. Ores & mineral products
9. Edible oil
10. Jute & jute products

REGULATED NON-REGULATED

Engaged in production of goods providing services



Overall T/o ≥ 35 crores

Maintain cost records

Exception: Micro & small enterprises per MSMED act

SEC 148: COST AUDIT ( PG 119)


RULE 4: CRA Rules, 2014 – COST AUDIT

REGULATED NON-REGULATED
Overall: 50 crores Overall: 100 crores
Individual: 25 crores Individual: 35 crores

97
FINAL C.A. – AUDIT

Exception: (SEE)
S- Which is operating from a special economics zone
E- Generation of electricity
E- Whose revenue from experts, exceeds 75% of its total revenue

A B C D E F G
REGULATED
X 2 - - 5 - 15 20
Y 5 - - 10 - 10 30
NON-REGULATED
P - 40 - - 20 - 40
Q - 15 - - 30 - 50
OTHERS 30 15 50 60 60 20 100
TOTAL 37 70 50 75 110 45 240

FORMS:
CRA 1: Cost records
CRA 2: Appointment/ casual vacancy/removal
CRA 3: Auditor Report BOD
CRA 4: BOD Report CG

OTHER PROVISIONS:
148(3) Appointment of Cost auditor

By BOD within 180 days of commencement of FY

He must be a Cost Accountant in practice.
(Auditor appointed u/s 139 cannot be appointed)

148(5) Qualifications & disqualification same as 141(1),141(2),141(3)

148(8) Punishment same as 147

98
FINAL C.A. – AUDIT

QUESTIONS ASKED IN PAST/RTP/MOCK TEST


Question 27:
Sugar LTD.is a top sugar Manufacturer and exporter in India operating from Noida Specific
economic zone, Uttar Pradesh . its revenue from Sale/export for the preceding year is given
Below;
Sale within India RS 1157 Lakhs
Sale outside India (Export) RS 1353 Lakhs
Total revenue RS 2510 Lakhs
Mr.X the statutory auditor of sugar LTD, is of the view that the company is Mandatorily
required to include cost records in their books of accounts and consequently conduct cost
Audit. He also suggested the name of his friend, who is a cost Accountant in practice, for the
purpose of such cost Audit. However, the management is of the view that the company neither
required including cost records in their books of accounts nor conduct cost audit. Comment.
Answer:
Company is neither required to maintain cost records nor records cost Audit since he is from
SEZ.

Question 28:
XYZ LTD. Having place of business in Delhi, is engaged in the production, trading, import and
export of orthopedic implants and pacemaker. The company’s revenue from export is usually in
foreign currency. Its total revenue classification for the immediate preceding financial year is
as follows:
intra-state sale RS 1400 Lakhs
inter-state sale RS 1550 lakhs
Export to US RS 4900 Lakhs
Export to UK RS 6900 Lakhs
Total Revenue RS 14750 Lakhs
The management of the company is of the opinion that the company is not required to maintain
cost records in their books of account. Consequently, there is no need to appoint cost auditor
and conduct cost audit. Comment.
Answer:
XYZ LTD.Is required to include cost records in their books of accounts in accordance with
Rule 3 of the companies (cost records and audit) Rules, 2014.

However, the company is not required to conduct cost audit as its revenue from exports, in
foreign exchange, exceeds 75 per cent of its total revenue.

99
FINAL C.A. – AUDIT

Question 29:
X.LTD Is engaged in the production of iron and steel. A CA Firm “M/s M & CO” was appointed
as the statutory auditor of X LTD. For the current financial year. During the year, the
management of the company realized that the company is required to maintain cost records in
their books of accounts and get it audited. Therefore, in a general meeting , the members of
the company appointed M/S M & CO. as the cost auditor of the company. You are required to
examine the validity of appointment of M/S M & CO. as the cost auditor.
Answer:
Appointment is not valid as a CA firm cannot be appointed as cost auditor. As per section
148(3) of companies Act, 2013 read with rule 6 of companies (cost records and audit) Rules,
2014, cost audit shall be conducted by a cost Accountant appointed by BOD.

100
FINAL C.A. – AUDIT

CARO 2020

FS AR CARO

1. Applicability : All companies including foreign companies

2. Excluded: (BIPS)
B- Banking Co
I- Insurance Co
PUSC + Res ≤ 1crore AND
P- Pvt Ltd C0 Loan ≤ 1crore AND

Revenue ≤ 10crore
S – Sec 8 co, small co, one person co

3. Status of the co shall be checked on the balance sheet date.

4. If applicable to holding co, it shall also apply to subsidiary.

5. The order shall not apply for Consolidated FS except in clause (xxi)

QUESTIONS ASKED IN PAST/RTP/MOCK TEST

Question 23:
As an auditor, how would you deal with the following: L pvt. Ltd. Which has an outstanding
loss of more than Rs. 100 lakhs from financial institution defaulted in repayment thereof to
the extent of 50%. The company holds that it being a private limited company, the Companies
Auditor Report Order (CARO) is not applicable.
Answer:
Contention of L Pvt. Ltd is not correct as borrowing from financial institutuion exceeds Rs.
1 Cr., and auditor is required to report the period and amount of default in repayment of dues
under Para 3 (viii) of CARO 2016.
101
FINAL C.A. – AUDIT

Question 24:
A Pvt. Ltd. Company reports the following position as on 31st March 2019:
Paid up capital: 60 Lacs
Revaluation reserves: 20 Lacs
Capital Reserves: 22 Lacs
P & LA /C(Dr. balance): 4 Lacs
The management of the company contends that CARO, 2016 is not applicable to it.
Answer:
CARO is not applicable as paid up capital and reserves does not exceed Rs. 1 Cr. (60 Lacs+ 20
Lacs + 22 Lacs – 4 Lacs).

Question 25:
Under CARO 2016, how as a statutory auditor would you comment on the following: X Pvt.
Ltd. Is a subsidiary of a listed entity. The management of the company believes that since X
Pvt. Ltd. is a private company and satisfies all condition under CARO 2016, reporting under
CARO is not applicable.
Answer:
CARO is applicable as extension is not available to a private company wich is a subsidairy or
holding of a public company.

(i) PROPERTY,PLANT, EQUIPMENT & INTANGIBLE ASSET

Quantitative Physical Deeds of immovable


details verification property in names of co
maintained conducted
at regular
intervals (ii) INVENTORY

Physical verification Material discrepancy is reported

(iii) LOAN U/S 189

T&C are not prejudicial Repayment of Int & If overdue> 90 day


To members prin is regular any steps for recovery
taken
102
FINAL C.A. – AUDIT

(iv) Compliance of sec 185 & 186

(v) PUBLIC DEPOSIT

Directives of RBI Provisions of Co’s Act Order of CLB/Tribunal

(vi) Cost records maintained as per sec 148

(vii) STATUTORY DUES

Undisputed Disputed

If O/S > 6m then report Always report amount & authority

(viii) DEFAULT IN REPAYMENT OF LOANS & BORROWINGS

Banks/Financial Institute/ Govt

Report period & amount of default

(ix) PUBLIC OFFER & TERM LOANS

Money used for proper purpose

If not report

(x) FRAUD

On company or by the company

By officer or employee

Report Nature & Amount of fraud

(xi) MANAGERIAL REMUNERATION

103
FINAL C.A. – AUDIT

Is as per sec 197 of Sch V of COA, 2013

If not, state amount involved and steps taken for refund of same

(xii) NIDHI COMPANY

Net owned funds: Deposits


1: 20
10% unencumbered term deposit (liquid)

(xiii) RELATED PARTY

Is as per sec 177, 188 and AS- 18

(xiv) Preferential Allotment/ Pvt placement/ Fully covertible debenture

As per sec 42

(xv) NON CASH TRANSACTION

With directors or persons connected with him

If so, whether the provisions of sec 192 of Co. Act 2013 have been complied with

(xvi) SEC 45 IA OF RBI, ACT 1934

NBFC Registration

104
FINAL C.A. – AUDIT

CARO 2020

FS AR CARO

1. Applicability : All companies including foreign companies

2. Excluded: (BIPS)
B- Banking Co
I- Insurance Co
PUSC + Res ≤ 1crore AND
P- Pvt Ltd C0 Loan ≤ 1crore AND

Revenue ≤ 10crore
S – Sec 8 co, small co, one person co

3. Status of the co shall be checked on the balance sheet date.

4. If applicable to holding co, it shall also apply to subsidiary.

5. The order shall not apply for Consolidated FS except in clause (xxi)
The auditor’s report of holding company shall also be an indicative of the companies
included in the consolidated FS whose CARO reports contain qualifications/adverse
remarks

QUESTIONS ASKED IN PAST/RTP/MOCK TEST

Question 1:
As an auditor, how would you deal with the following: L pvt. Ltd. Which has an outstanding
loss of more than Rs. 100 lakhs from financial institution defaulted in repaymentthereof to
the extent of 50%. The company holds that it being a private limited company, the Companies
Auditor Report Order (CARO) is not applicable.
105
FINAL C.A. – AUDIT

Answer:
Contention of L Pvt. Ltd is not correct as borrowing from financial institutuion exceeds Rs.
1 Cr., and auditor is required to report the period and amount of default in repaymentof dues
under Para 3 (viii) of CARO 2020.

Question 2:
l A Pvt. Ltd. Company reports the following position as on 31st March 2019:

l Paid up capital: 60 Lacs

l Revaluation reserves: 20 Lacs

l Capital Reserves: 22 Lacs

l P & LA /C(Dr. balance): 4 Lacs

l The management of the company contends that CARO, 2020 is not applicable to it.

Answer:
l CARO is not applicable as paid up capital and reserves does not exceed Rs. 1 Cr. (60

Lacs+ 20 Lacs + 22 Lacs – 4 Lacs).

Question 25:
Under CARO 2020, how as a statutory auditor would you comment on the following: X Pvt.
Ltd. Is a subsidiary of a listed entity. The management of the company believes that since X
Pvt. Ltd. is a private company and satisfies all condition under CARO 2016, reporting under
CARO is not applicable.
Answer:
CARO is applicable as extension is not available to a private company which is a subsidiary or
holding of a public company.
(i) PROPERTY,PLANT, EQUIPMENT & INTANGIBLE ASSET

106
FINAL C.A. – AUDIT

(i) PROPERTY,PLANT, EQUIPMENT & INTANGIBLE ASSET

Quantitative Physical Deeds of immovable Revalued its P, Proceedings


details verification property in names of co if P, E & IA initiated or
maintained conducted not , report in prescribed pending for
at regular format Revaluation holding benami
Of property, intervals based on property
plant & ü Description of property valuation by
equipment and Discrepancy ü Gross carrying value registered Disclosed in FS
intangible Are dealt with ü Held in name of valuer
assets Promoter, director Amount if
or their relative or change ≥ 10%
employee
ü Period held-indicate
range, where
appropriate
ü Reasons for not being
held in name of co

(ii) INVERNTORY

Physical verification Sanctioned working Capital limits


> 5 crores in aggregate, from banks or
Discrepancy ≥ 10% financial institutions

Discrepancy Whether quarterly returns or statements


Are dealt with are filed

Are in agreement with books of accounts

107
FINAL C.A. – AUDIT

(iii) Investments, Guarantee,Security & loans granted by co

a) Whether co has b.) T&C are not prejudicial c.) Repayment of


provided loans, guarantee To interest of the company Interest & principal is
or provided security to any regular
other entity. (NA to co.
whose principal business is
to give loans)

Aggregate amount Aggregate amount


given to subsidiaries, given to other
joint ventures & parties
associates

(iii) Investments, Guarantee,Security & loans granted by co

d.) If overdue> 90 day any e.) Loans & advances f.) If loans or advances
steps for recovery taken has fallen due during the granted in nature of
year has been renewed or loans either repayable
extended or fresh loans on demand or without
granted to settle the specifying terms or period
overdues of existing loan of repayment

% of the aggregate to total Specify the aggregate


loans or advances(NA amount
to co. whose principal
business is to give loans) % of total loans to loans
to promoter &related
parties

108
FINAL C.A. – AUDIT

(iv) Compliance of sec 185 & 186

(v) PUBLIC DEPOSIT(Including Deemed deposits)

Directives of RBI Provisions of Co’s Act Order of CLB/Tribunal

(vi) Cost records maintained as per sec 148

(vii) STATUTORY DUES (including GST)

Undisputed Disputed

If O/S > 6m then report Always report amount & authority

(viii) Disclosure of transactions not recorded

Whether any transactions not recorded in books have been surrendered or


disclosed as income during tax assessments under income tax act, 1961

If so, whether the previously unrecorded income has been properly recorded
in the books of accounts during the year

(ix) Default in repayment of loans or other borrowings

a. If default in repayment to any b. Whether the company c. Whether term loans


lender is declared willful defaulter were applied for which
by any bank or financial the loans were obtained
If yes, report the period and amount institute or other lender
of default as per prescribed format If not, the amount of
ü Nature of borrowing, including loan so diverted and the
debt securities purpose for which it is
ü Name of lender used.
ü Amount not paid on due date
ü Whether principal or interest
ü No. of days delay or unpaid
ü Remarks ,if any

109
FINAL C.A. – AUDIT

(ix) Default in repayment of loans or other borrowings

d. Whether funds e. Whether the funds are f. Whether the company


raised on short term used to meet obligations of has raised loans on the
basis have been its subsidiaries, associates pledge of securities held
utilized for long term or joint ventures in its subsidiaries, joint
purpose ventures or associate
If so, details thereof companies
If yes, the nature with nature of such
and amount to be transactions and the If so, give details thereof
indicated amount in each case. and also report default if
any

(x) Money raised by IPO, FPO & preferential allotment/private placement of


shares or convertible debentures

a. Whether money raised by IPO or FPO b. Whether the company has made any
are applied for the purpose for which they preferential allotment or private placement
are raised. of share or convertible debentures

If not, details together with delays or If so, whether sec 42 & 62 of companies
default and subsequent rectification, if any, Act,2013 have been complied with.
shall be reported
If not, provide details in respect of amount
involved and nature of non compliance

(xi) FRAUD

On company or by the 143(12) of companies Whether the auditor s


company is reported Act,2013 has been considered whistle blower
during the year complied with complaints, if any, received
during the year by the
If yes, Report Nature company
& Amount of fraud

110
FINAL C.A. – AUDIT

(xii) NIDHI COMPANY

a. Net owned funds: b.10% unencumbered term c. Whether there has been
Deposits deposit (liquid) any default in payment
1: 20 of interest on deposit or
principal

If yes, details thereof

(xiii) RELATED PARTY

Is as per sec 177, 188 and AS- 18

Sec 177: of Companies Act, 2013 amended to empower Audit Committee to give omnibus
approvals for related party transactions on annual basis

Sec 188: Prior approval of Board of Directors of the Company is required so as to enter into
any contract or arrangement with related party

AS 18: Related party disclosure

(xiv) Internal Audit System

a. Whether co has an ICS commensurate b. Whether reports of IA were considered


with the size and nature of its business by statutory auditor

Sec 138 of Companies Act, 2013 – Internal Audit

Listed Co Public co Pvt co


PUSC≥50cr or T/O≥200cr or
T/O≥200cr or Borr≥100cr
Borr≥100cr

111
FINAL C.A. – AUDIT

(xv) NON CASH TRANSACTION

With directors or persons connected with him

If so, whether the provisions of sec 192 of Co. Act 2013 have been complied with

Sec 192: prior approval for such arrangement is accorded by a resolution of the
company in general meeting 

(xvi) SEC 45 IA OF RBI, ACT 1934

a. Whether the b. Whether the co. c. Whether the co. is d. Whether the co.
co. is required to has conducted any a Core Investment Co Group has more than
be registered u/s Non-banking Financial (CIC) as defined by one CIC
45-IA of RBI Act, or housing Finance RBI
1934 (i.e NBFC) activities without if yes, indicate the
a valid certificate of if so, whether it number of CIC which
If so, whether the registration continues to fulfil are part of the group
registration has the criteria & if it is
been obtained exempted, it continues
to fulfil such criteria

CORE INVESTMENT CO
Core Investment Company (CIC) is a non-banking financial company carrying on the
business of acquisition of shares and securities and which

(a) holds not less than 90 per cent of its net assets in the form of investment in equity
shares, preference shares, bonds, debentures, debt or loans in group companies and

(b) its investments in the equity shares in group companies constitutes not less than 60 per
cent of its net assets as on the date of the last audited balance sheet.

112
FINAL C.A. – AUDIT

(xvii) Cash losses

Whether the co has incurred cash losses in the FY & immediately preceding FY

If so, state the amount of cash losses

(xviii) Resignation of Statutory Auditor

Whether there has been any resignation of the statutory auditors during the year

If so, whether the auditor has taken into consideration the issues, the objections or
concerns raised by the outgoing auditor

(xix) Capability of Co meeting its liabilities existing at the BS date

On the basis of the Financial ratios, ageing and expected dates of realization of financial
assets and payments of financial liabilities

Or other info, the auditors knowledge of BOD and managements plan

Whether the auditor is of the opinion that no material uncertainty exists as on the date of
Audit report that co is capable of meeting its liabilities existing at the date of BS

And when they fall due within a period of one year from the date of BS

(xx) Transfer amount remaining unspent u/s 135(5) to fund specified in Sch VII

Sec 135(5):The Board of every company shall ensure that the company spends, in
every financial year, at least two per cent. of the average net profits of the company made
during the three immediately preceding financial years, in pursuance of its Corporate Social
Responsibility Policy

SCH VII: Activities which may be included by companies in their Corporate Social Responsibility
Policies Activities

113
FINAL C.A. – AUDIT

(xx) Transfer amount remaining unspent u/s 135(5) to fund specified in Sch VII

a. whether, in respect of other than b. whether any amount


ongoing projects, the company has remaining unspent pursuant to
transferred unspent amount to a any ongoing project, has been
Fund specified in Schedule VII to the transferred to special account
Companies Act within a period of six
months of the expiry of the financial year

(xxi) Reporting on Consolidated Financial Statements

Whether there have been any qualifications or adverse remarks by the respective auditors in
the CARO reports of the companies included in the consolidated financial statements

if yes, indicate the details of the companies and the paragraph numbers of the CARO report
containing the qualifications or adverse remarks.

The change in reporting requirements clearly shows that regulator’s expectations from the
auditors are increasing significantly.

More emphasis is added on utilization of funds, financial stability of the company and regulatory
compliances.

Auditors need to be more conscious, skeptical and accurate while discharging his/her duties.

KOI SHAK?????

114
FINAL C.A. – AUDIT

SA 210
AGREEING TO THE TERMS OF
AUDIT ENGAGEMENTS

5. LETTER OF APPOINTMENT

X LTD 3. TERMS OF ENGANGEMENT ABC & CO

2. PROPOSAL

4. LETTER OF ENGAGEMENT

1. PRECONDITIONS

Determine whether Obtain agreement from management


FRF is acceptable regarding their responsibilities
or not 1. For preparation of FS
2. Internal control (IC)
3. To provide auditor with
a. Access with Info
b. Access to additional Inf0
c. Unrestricted access to persons within the entity

Note: If preconditions for an audit are not present, the auditor shall discuss with management.
Unless required by law or regulation to do so, the auditor shall not accept the proposed
audit engagement

2. FACTORS TO BE CONSIDERED FOR DETERMINING WHETHER FRF APPLIED BY


THE ENTITY IS ACCEPTABLE OR NOT
• The nature of the entity (for eg, whether it is a business enterprise, or a not for profit
organization)
• The purpose of the financial statements (for eg, whether they are prepared to meet the
common financial information needs of a wide range of users or the financial information
needs of specific users)

115
FINAL C.A. – AUDIT

• The nature of the financial statements (for eg, whether the financial statements are a
complete set of FS or a single FS)
• Whether law or regulation prescribes the applicable FRF

3. CONTENT OF ENGAGEMENT LETTER

The agreed terms of the audit engagement shall be recorded in an audit engagement letter or
other suitable form of written agreement and shall include: (R3OSI)

R- The responsibilities of the auditor;

R2- The responsibilities of management;

R3- Reference to the expected form and content of any reports to be issued by the auditor
and a statement that there may be circumstances in which a report may differ from its
expected form and content

OS- The objective and scope of the audit of the financial statements;

I- Identification of the applicable FRF for the preparation of the financial statements;
and

4. ADDITIONAL MATTERS THAT CAN BE REFERRED IN ENGAGEMENT LETTER


APART FROM THE MATTERS REQUIRED BY SA 210 (FIRE)

F- The basis on which fees are computed and any billing arrangements

I- The fact that because of the inherent limitations of an audit, together with the
inherent limitations of internal control, there is an unavoidable risk that some material
misstatements may not be detected, even though the audit is properly planned and
performed in accordance with SAs

R- The fact that the audit process may be subjected to a peer review under the Chartered
Accountants Act, 1949

E- Elaboration of the scope of the audit

116
FINAL C.A. – AUDIT

5. ACCEPTANCE OF CHANGE IN THE TERMS OF ENGAGEMENT

During the course of audit

If management requests to change the TOE

Inquire about the reasons

VALID INVALID

Inquire about the reasons

VALID INVALID

Accept the change Donot accept. If management
(In writing) & TCWG do not allow auditor
to continue the original engagement

ROSA

Restriction on scope of audit

WITHDRAW
After fulfilling legal obligation

6. RECURRING AUDIT. (MORONS)


When there is a need to remind the entity :
M- Entity misunderstands the objective & scope
0- A significant change in ownership
R- Revised or special terms of engagement
0- A change in other reporting requirement
N- A change in nature & size of the entity’s business
S- Recent change of senior management

117
FINAL C.A. – AUDIT

7. FACTORS THAT MAY INFLUENCE THE DECISION WHETHER TO SEND A


SEPARATE AUDIT ENGAGEMENT LETTER TO THE COMPONENT (O SALI)
When the auditor of a parent entity is also the auditor of a component, the factors that may
influence the decision whether to send a separate audit engagement letter to the component
include the following:
O- Degree of ownership by parent
S- Whether a separate auditor’s report is to be issued on the component
A- Who appoints the component auditor
L- Legal requirements in relation to audit appointments
I- Degree of independence of the component management from the parent entity

8. IF FINANCIAL REPORTING STANDARDS ESTABLISHED BY AN AUTHORISED


OR RECOGNISED STANDARDS SETTING ORGANIZATION ARE SUPPLEMENTED
BY LAW OR REGULATION

l The auditor shall determine whether there are any conflicts between the financial
reporting standards and the additional requirements.

l If such conflicts exist, the auditor shall discuss with management the nature of the
additional requirements and shall agree whether:

l (i) The additional requirements can be met through additional disclosures in the
financial statements; or

l (ii) The description of the AFRF in the financial statements can be amended
accordingly.

l If neither of the above actions is possible, the auditor shall determine whether it will be
necessary to modify the auditor’s opinion in accordance with SA 705

9. IF THE LAW OR REGULATION APPLICABLE TO THE ENTITY PRESCRIBES THE


LAYOUT OR WORDING OF THE AUDITOR’S REPORT
l In some cases, the law or regulation applicable to the entity prescribes the layout or

wording of the auditor’s report in a form or in terms that are significantly different
from the requirements of SAs. In these circumstances, the auditor shall evaluate:

118
FINAL C.A. – AUDIT

l Whether users might misunderstand the assurance obtained from the audit of the
financial statements and, if so,

l Whether additional explanation in the auditor’s report can mitigate possible


misunderstanding.

l If the auditor concludes that additional explanation in the auditor’s report cannot
mitigate possible misunderstanding, the auditor shall not accept the audit engagement,
unless required by law or regulation to do so.

10. FINANCIAL REPORTING FRAMEWORK PRESCRIBED BY LAW OR REGULATION


IS UNACCEPTABLE
l The auditor shall accept the audit engagement only if the following conditions are
present:

l Management agrees to provide additional disclosures in the financial statements


required to avoid the financial statements being misleading

l The auditor’s report on the financial statements will incorporate an Emphasis


of Matter paragraph, drawing users’ attention to the additional disclosures, in
accordance with SA 706

QUESTIONS ASKED IN PAST/RTP/MOCK TEST


Question 1:
Mr. Ram Kapoor, Chartered Accountant, has been appointed as the statutory Auditor by
XYZ Pvt. Ltd. For the Audit of their Financial statements for the year 2018-2019. The
company has mentioned in their Audit terms that they will not be able to provide the internal
audit reports to Mr. Ram during the course of audit. Further, company also imposed some
limitation on scope of Mr. Ram.
What are the preconditions Mr. Ram should ensure before accepting/refusing the proposal?
Also advise, wether Mr. Ram should accept the proposed audit engagement?

Question 2:
QAKJ Ltd. is a small sized 30 years old company having business of manufacturing of pipes.
Companies has a plant based out of Dehradun and have their corporate office in Delhi. Recently
the company appointed new firm of Chartered Accountants as their statutory auditors.

119
FINAL C.A. – AUDIT

The statutory auditors want to enter into an engagement letter with the company in respect of
their services but the management has contended that since the statutory audit is mandated
by law, engagement letter may not be required.
Auditors did not agree to this and have shared a format of engagement letter with the
management for their reference before getting the signed. In this respect management would
like to understand that as per SA 210 (auditing standard referred to by the auditors), if the
agreed terms of the engagement shall be recorded in an engagement letter or other suitable
form of written agreement, what should be included in terms of agreed audit engagement
letter?

Question 3:
Comment on the following: “It is not mandatory to send a new engagement letter in recurring
audit, but sometimes it becomes mandatory to send newsletter Explain those situations where
new engagement letter is to be sent.
OR
R & Co., a firm of Chartered Accountants have not revised the terms of engagements and
obtained confirmation from the clients, for last 5 years despite changes in business and
professional development. Please elucidate the circumstances that may warrant the revision in
terms of engagement.

120
FINAL C.A. – AUDIT

SA 220
ELEMENTS OF FIRM’S SYSTEM OF
QUALITY CONTROL


Engagement Quality General Quality
Control Review: Control measures.

Listed Company

I. IMPORTANT CONCEPTS
1. Engagement quality control review :
l a process designed to provide an objective evaluation, before the report is issued,

l of the significant judgments the engagement team made and the conclusions they
reached in formulating the report

2. ENGAGEMENT QUALITY CONTROL REVIEWER :


i. a partner,
ii. other person in the firm,
iii. suitably qualified external person, or
iv. a team made up of such individuals, with sufficient and appropriate experience and
authority to objectively evaluate provided such team should be headed by a member of
the Institute

3. ENGAGEMENT TEAM :
l All personnel performing an engagement, including any experts contracted by the firm

in connection with that engagement.

l The term “engagement team” excludes individuals within the client’s internal audit
function who provide direct assistance on an audit engagement when the external
auditor complies with the requirements of SA 610

121
FINAL C.A. – AUDIT

II. GENERAL REQUIREMENTS (LIE AAP)


L – LEADERSHIP RESPONSIBILITIES FOR QUALITY WITHIN THE FIRM:

l The engagement partner shall take responsibility for the overall quality on each audit
engagement to which that partner is assigned.

l He shall deliver the following messages to the audit team clearly :


l Performing work that complies with professional standards and regulatory and

legal requirements

l Complying with the firm’s quality control policies and procedures as applicable

l Issuing auditor’s reports that are appropriate in the circumstances; and

l The engagement team’s ability to raise concerns without fear of reprisals

I-Independence
The engagement partner shall form a conclusion on compliance with independence
requirements that apply to the audit engagement. In doing so, the engagement partner shall:

Obtain relevant information Evaluate information on Take appropriate action to
from the firm and, where identified breaches, if any, eliminate such threats or
applicable, network firms, of the firm’s independence reduce them to an acceptable
to identify and evaluate policies and procedures level by applying safeguards,
circumstances and to determine whether or, if considered appropriate,
relationships that create they create a threat to to withdraw from the audit
threats to independence independence for the audit engagement, where withdrawal
engagement is permitted by law or
regulation.
122
FINAL C.A. – AUDIT

E-Relevant Ethical Requirements as discussed in


Code of ethics issued by the ICAI

Throughout the audit engagement, If matters come to the engagement partner’s


the engagement partner shall remain attention through the firm’s system of quality
alert, through observation and making control or otherwise that indicate that members
inquiries as necessary, for evidence of of the engagement team have not complied with
non- compliance with relevant ethical relevant ethical requirements, the engagement
requirements by members of the partner, in consultation with others in the
engagement team firm, shall determine the appropriate action

A-ACCEPTANCE AND CONTINUANCE OF CLIENT RELATIONSHIPS AND AUDIT


ENGAGEMENTS:
Information such as the following assists the engagement partner in determining whether
the conclusions reached regarding the acceptance and continuance of client relationships and
audit engagements are appropriate: (C2SI)
C - Whether the engagement team is competent to perform the audit engagement and has
the necessary capabilities, including time and resources

C - Whether the firm and the engagement team can comply with relevant ethical requirements

S - Significant matters that have arisen during the current or previous audit engagement,
and their implications for continuing the relationship

I - The integrity of the principal owners, key management and those charged with governance
of the entity

A-Assignment of Engagement Teams:


The engagement partner shall be satisfied that the engagement team, and any auditor’s
experts who are part of the engagement team, collectively have the appropriate competence
and capabilities to:

Perform the audit engagement in Enable an auditor’s report that is appropriate


accordance with professional standards in the circumstances to be issued
and regulatory and legal requirements
123
FINAL C.A. – AUDIT

P-ENGAGEMENT PERFORMANCE:
Following Measures must be taken to improve the engagement performance and reduce audit
risk to an acceptably low level:

Direction - Direction of the engagement team involves informing the members of the engagement
team of matters such as:
1. Their responsibilities, including the need to comply with relevant ethical requirements,
and to plan and perform an audit with professional skepticism

2. Responsibilities of respective partners where more than one partner is involved in the
conduct of an audit engagement

3. The detailed approach to the performance of the engagement.

Supervision:
It includes matters such as
1. Tracking the progress of the audit engagement
2. Addressing significant matters arising during the audit engagement
3. Identifying matters for consultation or consideration by more experienced engagement
team members during the audit engagement

Reviews:
Timely reviews of the following by the engagement partner at appropriate stages during the
engagement allow significant matters to be resolved on a timely basis to the engagement
partner’s satisfaction on or before the date of the auditor’s report:

1. Critical areas of judgment


2. Significant risk as identified as per SA 315
3. Other areas the engagement partner considers important

Consultation:
It may be appropriate for the engagement team to consult outside the firm, for example,
where the firm lacks appropriate internal resources. For E.g. They may take advantage of
advisory services provided by other firms, professional and regulatory bodies, or commercial
organisations that provide relevant quality control services.

124
FINAL C.A. – AUDIT

Engagement Quality Control Review:


l For audits of financial statements of listed entities, and those other audit engagements,

if any, for which the firm has determined that an engagement quality control review is
required, the engagement partner shall determine that engagement quality control
reviewer has been appointed and not date auditor’s report until such review is completed.
The reviewer shall:

l Discuss significant matters with the engagement partner

l Review the financial statements and the proposed auditor’s report

l Review of selected audit documentation relating to the significant judgments the


engagement team made

l Evaluation of the conclusions reached in formulating the auditor’s report

Differences of Opinion
If differences of opinion arise within the engagement team, with those consulted or, where
applicable, between the engagement partner and the engagement quality control reviewer, the
engagement team shall follow the firm’s policies and procedures for dealing with and resolving
differences of opinion

Monitoring
An effective system of quality control includes a monitoring process designed to provide the firm
with reasonable assurance that its policies and procedures relating to the system of quality
control are relevant, adequate, and operating effectively.

Documentation
l Issues identified with respect to compliance with relevant ethical requirements and how

they were resolved

l Conclusions on compliance with independence requirements that apply to the audit


engagement

l Conclusions reached regarding the acceptance and continuance of client relationships


and audit engagements

125
FINAL C.A. – AUDIT

l The nature and scope of, and conclusions resulting from, consultations undertaken during
the course of the audit engagement including
1. The issue on which consultation was sought
2. The results of the consultation, including any decisions taken, the basis for those
decisions and how they were implemented

l Whether the engagement quality control review has been completed on or before the date
of the auditor’s report and findings of such review.

QUESTIONS ASKED IN PAST/RTP/MOCK TEST

Question 1:
During the Audit of FMP Ltd., a listed company, Engagement Partner (EP) completed his
reviews and also ensured compliance with independent requirements that apply to audit
engagement. The engagement files were also reviewed by the Engagement Quality Control
Reviewer (EQCR) except the independent assessment documentation. Engagement partner
was of the view that matters related to independent assessment are the responsibility of the
Engagement Partner and not the Engagement Quality Control Reviewer. Engagement Quality
Control Reviewer objected to this and refused to sign off the documentation. Please advise as
per SA 220.

Question 2:
M/s Sureshchandra and Co. has been appointed as an auditor of SC Ltd. For the financial
year 2018-2019. CA. Suresh, one of the partners of M/s Sureshchandra & Co., completed
entire routine work by 29th May, 2019. Unfortunately, on the very next morning, while roving
towards office of SC Ltd. To sign the final audit report, he met with a road accident and died,
CA Chandra, another partner of M/s Sureshchandra &Co., therefore, signed the accounts of
SC Ltd., without reviewing the work performed by CA Suresh.

Answer:
As per SA 420, “Quality Control for an Audit of Financial Statements’, the engagement partner
shall take responsibility for reviews being performed in accordance with the firm’s review
policies and procedures, Review procedures consists of the considerations, whether,
a) The work has been per formed in accordance with professional standards and regulatory
requirements
b) Significant matters have been raised for further consideration

126
FINAL C.A. – AUDIT

c) Appropriate consultations have taken place and the resulting conclusions have been
documented and implemented

Answer:
d) the work performed supports the conclusions reached and is appropriately documented
e) the evidence obtained is sufficient and appropriate to support the auditor’s report; and
f) the objectives of the engagement procedures have been achieved.
l When the Auditor delegates work to assistants or use work performed by other
auditors/experts he will continue to be responsible for forming and expressing his
opinion on the financial statements. However, he will be entitled to rely on the work
performed by others, provided he exercises adequate skill and care and is not aware
of any reason to believe that he should not have so relied

Answer:
l The auditor should carefully direct, supervise and review work delegated to assistants. He

should obtain reasonable assurance that work performed by other auditors/experts and
assistants is adequate for his purpose.

l In the instant case, M Suresh, a partner of the firm bad completed routine audit work
and died before signing audit report. Mr. Chandra another partner of the firm has
signed the accounts of SC Ltd, relying on the work performed by M: Suresh,

l Conclusion CA.Chandra is allowed to sign the audit report, though, will be responsible
for expressing the opinion. He may rely on the work performed by CA.Suresh provided he
further exercises adequate skill and due care and review the work performed by him

Question 3:
OP & Associates are the statutory auditors of BB Ltd. BB Ltd is a listed Company and started
its operations 5 years back. The field work during the audit of the financial statements of the
company for the year ended March 31, 2019 got completed on May 1, 2019. The Auditor’s
report was dated May 12, 2019. During the documentation review of the engagement, it
was observed that the engagement quality control review was completed on May 15, 2019.
Engagement partner had completed his reviews in entirely by May 10, 2019. Comment.

127
FINAL C.A. – AUDIT

SA 230
AUDIT DOCUMENTATION &
EVIDENCE

Q.1 MEANING:
l The record of audit procedures performed,

l Relevant audit evidence obtained,

l And conclusions the auditor reached

Q.2 OWNERSHIP & RETENTION OF AUDIT DOCUMENTATION

l As per SQC- 1 audit documentation is the property of the auditor.

l As per SQC- 1 he should retain documentation for at least 7 years

l He may at his discretion, make portions of, or extracts from, audit documentation
available to clients, provided such disclosure does not undermine the validity of the work
performed, or in the case of assurance engagement, the independence of the auditor or
his personnel. (i.e our papers should not be subject to undue scrutiny)

Q.3 ACCESS TO WORKING PAPER

Client Wants Principal Auditor 3rd Party Wants Law Wants


Access Wants Access Access Access

Depends Depends upon other l Depends


upon Must provide
upon auditors auditors (SA 600) auditors discretion as access.
discretion as per discretion as per per SQC-1
SQC-1 SQC-1 l client confidentiality
(SA 200)
l Offence under Clause

1, Part 1 to 2nd Sch

128
FINAL C.A. – AUDIT

Q.4 PURPOSE OF AUDIT DOCUMENTATION:

(FAD PIE)

F- Retaining a record of matters of continuing significance to Future audit.

A- Enabling the engagement team to be accountable for its work.

D- Assisting members of the engagement team to direct & supervise the audit work

P- Assisting the engagement team to plan & perform the audit.

I- Enabling the conduct of quality control reviews & inspections

E- Enabling the conduct of external Inspection in accordance with applicable legal


requirements

Q.5 THE FORM, CONTENT & EXTENT OF AUDIT DOCUMENTATION:

(I Need DRS)

I- Document how the auditor addressed the inconsistency to resolve the doubts as identified
during the course of audit

N- The nature, timing and extent of the audit procedures performed to comply with the SAs
and applicable legal and regulatory requirements. In documenting this, the auditor shall
record:
1. Identifying characteristics of the specific items or matters tested
2. Who performed the audit work and the date such work was completed
3. Who reviewed the audit work performed and the date and extent of such review

D- Document discussions of significant matters with the management, those charged with
governance, and others, including the nature of the significant matters discussed and
when and with whom the discussions took place.

R- the result of the audit procedures performed, and the audit evidence obtained

129
FINAL C.A. – AUDIT

S- significant matters arising during the audit, the conclusions reached thereon, and
significant professional judgements made in reaching those conclusions.

Q.6 FACTORS AFFECTING THE FORM, CONTENT & EXTENT OF AUDIT DOCUMENTATION:

(Neha & Neetu MISS)

N- Nature of Audit procedures to be performed


N- Nature and extent of exceptions identified
M- The audit methodology and tools
I- The identified risk of MMS
S- The size & complexity of the entity.
S- The significance of audit evidence obtained.

Q.7 WORKING PAPER FILES:

PERMANENT AUDIT FILE CURRENT AUDIT FILE

It contains matters which It contains matters relating to


don’t change very often. audit of single period.

EXAMPLES: (SLAM) EXAMPLES: (LIEN)


S: significant audit observation L: Letter of representation or
of earlier years. confirmation received from
L: legal documents agreement the client.
& Minutes relevant to the I: Important Matter in
audit. Minute of BM & GM.
A: Analysis of significant ratios E: Evidence of the planning
& trends process of audit & audit
M: MOA & AOA programme.
  N: Nature,timing & extent of
auditing procedures. 

130
FINAL C.A. – AUDIT

Q.8 ASSEMBLY OF THE FINAL AUDIT FILE

l As per SQC- 1 complete assembly of the final audit file within 60 days after the date of
the auditor’s report.

l After the assembly of the final audit file has been completed, the auditor shall not delete
or discard audit documentation of any nature before the end of its retention period

QUESTIONS ASKED IN PAST/RTP/MOCK TEST

Question 1:
Mr. A, a practicing CA, has been appointed as an auditor of True Pvt. Ltd. What factors
would influence the amount of working papers required to be maintained for the purpose of his
audit?
Answer:
Factors affect form and content
N- Nature of Audit procedures to be performed
N- Nature and extent of exceptions identified
M- The audit methodology and tools
I- The identified risk of MMS
S- The size & complexity of the entity.
S- The significance of audit evidence obtained.

Question 2:
Discuss the auditors responsibilities to provide access to his audit working papers to regulators
and third parties.
Answer:
Access to working papers to Regulators and Third parties:

l Clause (1) of part I of the Second Schedule to the Chartered Accountants Act, 1949
states that a CA in practice shall be deemed to be guilty of professional misconduct if he
discloses information acquired in the course of his professional engagement to any person
other than his client, without the consent of his client or otherwise than as required by
law for the time being in force.

l SA 200 on “Overall Objectives of an Independent Auditor and the conduct of an audit

131
FINAL C.A. – AUDIT

in accordance with Standards on Auditing” also states that, “the auditor should respect the
confidentiality of the information obtained and should not disclose any such information to any
third party without specific authority or unless there is a legal or professional duty to disclose”.
If there is a request to provide access by the regulator based on the legal requirement, the
same has to be compiled with after informing the client about the same.

l SQC-1, provides that, unless otherwise specified by law or regulation, Audit documentation
is the property of the auditor. He may at his discretion, make portions for, or extracts
from, audit documentation available to clients, provided such disclosures does not
undermine the validity of the work performed, or in the case of assurance engagements,
the independence of the auditor or of his personnel.

l As per SA 230, audit documentation serves a number of additional purposes, including


the enabling the conduct of external inspections in accordance with applicable legal,
regulatory or other requirements.

l Conclusion: it is auditor’s responsibility to provide access to his audit working papers to


Regulators when required by law whereas auditor is under no obligation to provide access
to working papers to third parties.

Question 2:
As an auditor, how do u deal with the following: The statutory auditor of the Holding Company
demands for the working papers of the auditors of the subsidiary company, of which you are
the auditor.
Answer:
Access to working papers:

l As per SA 230, “Audit Documentation” working papers are the property of the auditor.
The auditor may, at his discretion, make portion of or extracts of his working papers
available to his client.

l SA 600 “Using the Work of Another Auditor” also states that an auditor should respect
the confidentiality of the information acquired during the course of his audit work
and should not disclose such information unless there is a legal or professional duty to
disclose.

132
FINAL C.A. – AUDIT

l As per ICAI Guidelines, statutory auditor of an enterprise does not have right of access
to the audit working papers of the branch auditor. An auditor can rely on the work
of another auditor, without having any right of access to the audit working papers of
another auditor.

l Conclusion: Statutory Auditor of the Holding Company cannot have access to audit
working papers of the Subsidiary companies auditor. He can however, ask the auditor to
answer certain questions about the manner in which the audit is conducted and certain
other clarifications regarding audit.

Question 3:
l B is the Principal Auditor of ABC Co. Ltd., with 8 branches audited by 8 Branch

Auditors. B wanted to ensure that the works of Branch Auditors were adequate for the
purpose of his audit. Hence, he insisted on Branch Auditors to get familiar with a check
list he preparedfor branches, and besides,required them to share the working papers
compiled by them for his review and return. Is Principal Auditor within his right in
asking for such sharing of working papers?
Answer:
l Principal Auditor’s right to review the working papers of branch auditors:

l SA 600 “Using the Work of Another Auditor” guides principal Auditor regarding the
procedures to be performed when he is using the work of another auditor, he should
consider the professional competence of the other Auditor in the context of the specific
assignment if the other auditor is not a member of the ICAI. He should perform procedures
to obtain sufficient appropriate audit evidence, that the work of the other auditor is
adequate for the principal auditor’s purposes, in the context of the specific assignment

l As per SA 230 “Audit Documentation” guides principal auditor regarding the procedures
to be performed when he is using the work of another auditor. As per SA 600, when
principal auditor plans to use the work of branch auditor, he should consider the
professional competence of the other auditor in the context of specific assignment if
the other auditor is not a member of the ICAI. He should perform procedures to obtain
sufficient appropriate audit evidence, that the work of the other auditor is adequate for
the principal auditor’s purposes, in the context of the specific assignment.

133
FINAL C.A. – AUDIT

l As per SA 230, “Audit Documentation” and SQC 1 “Quality control of Firms that
perform Audits and Reviews of Historical Financial Information, and other Assurance
and Related services Engagements”, unless otherwise specified by law or regulation, audit
documentation is the property of the auditor. The Principal auditors of an enterprise do
not have right of access to the audit working papers of the branch auditors.

l In the present case, Mr. B requires the branch auditors to share their working papers
with him for the purpose of review

l Conclusion:

l Considering the requirements of SA 600, SA 230 and SQC 1, principal auditor is not
right in asking for sharing of working papers.

134
FINAL C.A. – AUDIT

SA 240
AUDITOR’S RESPONSIBILITIES IN
RELATION TO FRAUD IN AN AUDIT
OF FINANCIAL STATEMENTS

I. DISCUSSION AMONG THE ENGAGEMENT TEAM:


SA 315 requires a discussion among the engagement team members especially emphasizing on
following matters:

l About how and where they believe the entity’s financial statements may be susceptible to
material misstatement due to fraud.

l A consideration of the known external and internal factors affecting the entity that
may create an incentive or pressure for management or others to commit fraud, provide
the opportunity for fraud.

l Any unusual or unexplained changes in behavior or lifestyle of management or employees


which have come to the attention of the engagement team.

l A consideration of any allegations of fraud that have come to the auditor’s attention.

II. Risk Assessment Procedures and Related Activities:

Inquiries Unusual or unexpected Other Evaluation of fraud


relationship identified information risk factors

Inquiries of Inquiries of Inquiries of those


management Internal Auditor charged with
Governance

135
FINAL C.A. – AUDIT

INQUIRIES:

A. Inquiries of Management regarding:

1. Management’s assessment of the risk that the financial statements may be materially
misstated due to fraud

2. Management’s process for identifying and responding to the risks of fraud in the entity,
including any specific risks of fraud that management has identified or that have been
brought to its attention

3. Management’s communication, if any, to TCWG regarding its processes for identifying


and responding to the risks of fraud in the entity

4. Management’s communication, if any, to employees regarding its views on business


practices and ethical behavior

5. Whether they have knowledge of any actual, suspected or alleged fraud affecting the
entity

B. Inquiries of Internal Auditor:


In accordance with SA 610 following inquiries can be done:

1. The procedures performed, if any, by the internal auditors during the year to detect
fraud

2. Whether management has satisfactorily responded to any findings resulting from those
procedures

C. Inquiries of Those Charged With Governance:

1. How TCWG exercise oversight of management’s processes for identifying and responding
to the risks of fraud in the entity and

2. the internal control that management has established to mitigate these risks.

136
FINAL C.A. – AUDIT

3. Whether they have knowledge of any actual, suspected or alleged fraud affecting the
entity.

D. Unusual or Unexpected Relationship Identified:


l The auditor shall evaluate whether unusual or unexpected relationships that have been

identified in performing analytical procedures

E. Other Information:
l The auditor shall consider whether other information obtained by the auditor indicates

risks of material misstatement due to fraud.

F. Evaluation of fraud risk factors:


l The auditor shall evaluate whether the information obtained from the other risk

assessment procedures and related activities performed indicates that one or more fraud
risk factors are present.

l While fraud risk factors may not necessarily indicate the existence of fraud, however,
they may indicate ROMMS due to fraud.

What are fraud risk factors:


A. Incentives/Pressures:

l High degree of competition or market saturation, accompanied by declining margins.

l High vulnerability to rapid changes, such as changes in technology, product obsolescence,


or interest rates.

l Significant declines in customer demand and increasing business failures in either the
industry or overall economy.

B. Opportunities
l Significant related-party transactions not in the ordinary course of business or with

related entities not audited or audited by another firm.

137
FINAL C.A. – AUDIT

l A strong financial presence or ability to dominate a certain industry sector that allows
the entity to dictate terms or conditions to suppliers or customers that may result in
inappropriate or non-arm’s-length transactions.

l Assets, liabilities, revenues, or expenses based on significant estimates that involve


subjective judgments or uncertainties that are difficult to corroborate.

l Significant, unusual, or highly complex transactions, especially those close to period end
that pose difficult “substance over form” questions.’

C. Attitudes/Rationalizations

l Communication, implementation, support, or enforcement of the entity’s values or ethical


standards by management, or the communication of inappropriate values or ethical
standards, that are not effective

l Non-financial management’s excessive participation in or preoccupation with the selection


of accounting policies or the determination of significant estimates

l Known history of violations of securities laws or other laws and regulations, or claims
against the entity, its senior management, or TCWG alleging fraud or violations of laws
and regulations

III. RESPONSES TO THE ASSESSED RISKS OF MATERIAL MISSTATEMENT

Responses to the assessed risks of material misstatement at Financial Statement Level:

1 Assign and supervise personnel taking account of the knowledge, skill and ability of the
individuals to be given significant engagement responsibilities and the auditor’s assessment
of the risks of material misstatement due to fraud for the engagement.

2. Evaluate whether the selection and application of accounting policies by the entity,
particularly those related to subjective measurements and complex transactions, may
be indicative of fraudulent financial reporting resulting from management’s effort to
manage earnings.

138
FINAL C.A. – AUDIT

3. Incorporate an element of unpredictability in the selection of the nature, timing and


extent of audit procedures.

RESPONSES TO THE ASSESSED RISKS OF MATERIAL MISSTATEMENT AT ASSERTION


LEVEL:

1. The NTE of audit procedures to be performed may need to be changed to obtain


audit evidence that is more reliable and relevant or to obtain additional corroborative
information

2. For significant and unusual transactions, particularly those occurring at or near year-
end, investigating the possibility of related parties and the sources of financial resources
supporting the transactions

3. Conducting interviews of personnel involved in areas where a risk of material misstatement


due to fraud has been identified, to obtain their insights about the risk and whether, or
how, controls address the risk

4. Seeking additional audit evidence from sources outside of the entity being audited.

5. Performing computer-assisted techniques, such as data mining to test for anomalies in a


population.

RESPONSES RELATED TO RISK OF MANAGEMENT OVERRIDE OF CONTROL:

l Management is in a unique position to perpetrate fraud because of management’s ability


to manipulate accounting records and prepare fraudulent financial statements by
overriding controls that otherwise appear to be operating effectively.

l Irrespective of the auditor’s assessment of the risks of management override of controls,


the auditor shall design and perform audit procedures to:

1. TEST THE APPROPRIATENESS OF JOURNAL ENTRIES BY


l Making inquiries of individuals involved in the financial reporting process about

inappropriate or unusual activity relating to the processing of journal entries and other
adjustments.

139
FINAL C.A. – AUDIT

l Selecting journal entries and other adjustments made at the end of a reporting period.

l Considering the need to test journal entries and other adjustments throughout the
period.

2. REVIEW ACCOUNTING ESTIMATES BY

l Evaluating whether there are circumstances leading to biased recognition.

l Verifying whether the judgments and decisions are individually reasonable.

l Retrospectively Reviewing estimates reflected in the prior year.

Evaluation of audit Evidence:

l When the auditor identifies a misstatement, the auditor shall evaluate whether such a
misstatement is indicative of fraud.

l If there is such an indication, the auditor shall evaluate the implications of the
misstatement in relation to other aspects of the audit, particularly the reliability of
management representations.

IV. AUDITOR IS UNABLE TO CONTINUE THE ENGAGEMENT:

l If, as a result of a misstatement resulting from fraud or suspected fraud, the auditor
encounters exceptional circumstances that bring into question the auditor’s ability to
continue performing the audit, the auditor shall:

1. Determine the professional and legal responsibilities applicable in the circumstances,


including whether there is a requirement for the auditor to report to the person or
persons who made the audit appointment or, in some cases, to regulatory authorities.

2. Consider whether it is appropriate to withdraw from the engagement, where withdrawal


from the engagement is legally permitted.

2. If the auditor withdraws:

140
FINAL C.A. – AUDIT

(i) Discuss with the appropriate level of management and TCWG, the auditor’s
withdrawal from the engagement and the reasons for the withdrawal and

(ii) Fulfill professional and legal responsibilities applicable in the circumstances.

Written Representation:
l The auditor shall obtain written representations from management and, where applicable,

TCWG that:

1. They acknowledge their responsibility for the design, implementation and maintenance of
internal control to prevent and detect fraud.

2. They have disclosed to the auditor


l the results of management’s assessment of the risk that the FS may be materially

misstated as a result of fraud.

l their knowledge of fraud or suspected fraud affecting the entity

l their knowledge of any allegations of fraud, or suspected fraud, affecting the entity’s
FS

V. COMMUNICATION TO MANAGEMENT AND TCWG:


1. If the auditor has identified a fraud or has obtained information that indicates that
a fraud may exist, the auditor shall communicate these matters on a timely basis to the
appropriate level of management in order to inform those with primary responsibility for
the prevention and detection of fraud of matters relevant to their responsibilities.

2. If the auditor suspects fraud involving management, the auditor shall communicate these
suspicions to those charged with governance and discuss with them the nature, timing
and extent of audit procedures necessary to complete the audit.

VI. COMMUNICATION TO REGULATORY AUTHORITIES:


l If the auditor has identified or suspects a fraud, the auditor shall determine whether

there is a responsibility to report the occurrence or suspicion to a party outside the


entity. The auditor’s legal responsibilities may override the duty of confidentiality in such
circumstances.

141
FINAL C.A. – AUDIT

VII. DOCUMENTATION:
Maintain records for following matters:
1. The identified and assessed ROMMS due to fraud at the financial statement level and at
the assertion level

2. The results of the audit procedures, including those designed to address the risk of
management override of controls

3. Communications about fraud made to management, those charged with governance,


regulators and others.

VIII. COMMON POINTS TO WRITE IN EXAM

1. Definition of fraud:
l The term fraud refers to an intentional act by one or more individuals among management,
TCWG, employees or 3rd parties ,involving the use of deception to obtain an unjust or
illegal advantages.

2. Primary responsibility:
l The primary responsibility for the prevention & detection of fraud rests with both TCWG

& management .

l The auditor , conducting an audit, is responsible for obtaining reasonable assurance that
the FS taken as a whole are free from material misstatement , whether caused by fraud
or error.

3. Inherent limitation of audit:


l Owing to the inherent limitations of an audit , there is an unavoidable risk that some

material misstatements of the FS may not be detected, even though the audit is properly
planned & performed in accordance with SA”s.

4. Risk of not detecting a MM due to fraud:


>
Risk of not detecting a MM due to error.
This is because fraud may involve sophisticated & carefully organized schemes designed to
conceal it.

142
FINAL C.A. – AUDIT

5. Risk of not detecting a MM resulting from management fraud


>
Risk of not detecting a MM resulting from employee fraud because, management is
frequently in a position to directly or indirectly manipulate accounting records, present
fraudulent financial info or override control procedure designed to prevent similar fraud
by other employees.

6. Section 143(12) of companies act,2013.


l If an auditor of a company in the course of the performance of his duties as auditor, has
reason to believe that an offence involving fraud is being or has been committed against
the company by officers or employees of the company , he shall immediately report the
matter to the central govt. within 60 days of his knowledge.

7. Clause (xii) of CARO, 2016

IX. POSSIBLE SOURCES OF FRAUD: ( CEASE RIO”S ASSETS)


C- Concealing, or not disclosing , facts that could the amounts recorded in the FS.

E- Engaging in complex transactions that are structured to misrepresent the financial


position or financial performance.

A- Altering records & terms related to significant & unusual transactions.

S- Stealing physical assets or intellectual property.

E- Embezzling receipts.

R- Recording fictitious journal entries , particularly close to the end of an accounting period,
to manipulate operating results or achieve other objectives.

I- Inappropriately adjusting assumptions & changing judgments used to estimate account


balance.

O- Omitting, advancing or delaying recognition in the FS of events & transactions that


have occurred during the reporting period.
Assets - Using an entity’s assets for personal use.

143
FINAL C.A. – AUDIT

X. OTHER MISCELLANEOUS POINTS:


A. Audit of cash
l The guidance note on audit of cash & bank balance also mentions that if the entity is

maintain an unduly large balance of cash, he should carry out surprise verification of
cash more frequently to ascertain whether it agrees.

l If cash in hand is not in agreement with book balance, he should seek explanations &
if the same are not satisfactory should state the said fact appropriately in his audit
report.

B. Teeming & loading :


l Teeming & loading is a book keeping fraud also known as short banking, delayed accounting

& lapping. It involves the allocation of one customers payment to another in order to
make the books balance & often in order to hide a shortfall or theft.

C. Fraud through supplier ledger: ( SAAI)


S- Suppressing credit notes issued by the suppliers & with drawing the corresponding amount
not claimed by them.

A- Inflating suppliers account with fictitious or duplicate invoices & subsequent


misappropriation as if payments are made to the supplier.

A- Withdrawing amounts which remain unclaimed for more than the normal time limit for
one reason or other by showing the same have been paid to parties.

I- Inflating values of the items purchased & collecting the excess from suppliers.

QUESTIONS ASKED IN PAST/RTP/MOCK TEST


Question 1:
In the course of audit, A Ltd. You suspect the management has indulged in fraudulent financial
reporting ? State the possible source of such fraudulent financial reporting.
Answer:
CEASE RIO”S Assets
Question 2:
You are appointed as an auditor of Global Ltd. Explain the risk factor relating to misstatements
arising from misappropriation of assets.

144
FINAL C.A. – AUDIT

Answer:
Incentives/opportunity/attitude

Question 3:
Briefly duties and responsibilities of an auditor in case of material misstatements resulting
from management fraud.
Answer:

Question 4:
While conducting statutory Audit of ABC Ltd., you come across IOUs amounting to RS 2.10
Crores . You also observe that despite similar high balances throughout the year , small of RS
50000 are withdrawn from the bank to meet day- to – day expenses.
Answer:

Question 5:
Honest Limited has entered into a transaction on 5 th March , 2019 , near year- end ,
whereby it has agreed to pay RS 5 lakhs per month to Mr. Y as annual retainer – ship fee
for “engineering consultation”. No amount was actually paid, but RS 60 lakhs are provided in
books of accounts as on March 31, 2019. Your inquiry elicits a response that need- based
consultation was obtained round the year, but there is no documentary or other evidence of
receipt of the service. As the auditor of M/S Honest Limited , what would be your approach?
Answer:

Question 6:
Is it appropriate for the auditor to make inquiries of management regarding management’s
own assessment of the risks of fraud and the controls in place to prevent and detect it?
Discuss.
Answer:

Question 1:
A Company’s net worth is eroded and trade payables are unpaid due to liquidity constraints.
The management represents to the unsecured loan to meet the liquidity constraints and that
negotiations are underway to secure large export orders.
Answer:
It is not a mitigiating factor, and hence modify opinion

145
FINAL C.A. – AUDIT

SA 250
CONSIDERATIONS OF LAWS AND REGULATIONS
IN AN AUDIT OF FINANCIAL STATEMENTS.

APPLICABLE LAWS AND REGULATIONS

Applicable financial reporting frame work Other laws

Auditors primary responsibility to Material effects on FS Immaterial effect on FS


verify compliances with such laws

Direct effect on FS Indirect effect on FS

Laws determining Non-compliance with such Laws can lead to heavy fines
amount and disclosures and penalties [eg. Child Labour and any protection Act]

Conduct inquiry and verify communication if any with


regulatory authorities

II. RISK ASSESSMENT PROCEDURES

The legal and regulatory framework applicable How the entity is complying with that
to the entity and the industry or sector in framework e.g.
which the entity operates e.g. 1. Inquire of management concerning
1. Update the understanding of those laws the entity’s policies and procedures
and regulations that directly determine regarding compliance with laws and
the reported amounts and disclosures in regulations
the financial statements. 2. Inquire of management regarding the
2. Inquire of management as to other laws policies or procedures adopted for
or regulations that may be expected  identifying, evaluating and accounting

to have a fundamental effect on the for litigation claims


operations of the entity.
146
FINAL C.A. – AUDIT

 The auditor shall obtain SAAE regarding compliance with the provisions of those laws
and regulations generally recognized to have a direct effect on the determination of
material amounts and disclosures in the financial statements:

 The auditor shall perform the following audit procedures to help identify instances of non-
compliance with other laws and regulations (indirect effect) that may have a material
effect on the financial statements

Inquiring of management and, where Inspecting correspondence, if any,


appropriate, TCWG, as to whether the with the relevant licensing or regulatory
entity is in compliance with such laws and authorities.
regulations

 The auditor shall request management and, where appropriate, TCWG to provide written
representations that all known instances of non- compliance or suspected non-compliance
with laws and regulations whose effects should be considered when preparing financial
statements have been disclosed to the auditor

III. AUDIT PROCEDURES WHEN NON-COMPLIANCE IS IDENTIFIED OR SUSPECTED:

STEP 1:

 If the auditor becomes aware of information concerning an instance of noncompliance


or suspected non-compliance with laws and regulations, the auditor shall obtain:
1. An understanding of the nature of the act and the circumstances in which it has
occurred
2. Further information to evaluate the possible effect on the financial statements

STEP 2:

 The auditor shall discuss the matter with management and, where appropriate,
TCWG.

 If management or, as appropriate, TCWG do not provide sufficient information that


supports that the entity is in compliance with laws and regulations

147
FINAL C.A. – AUDIT

 then the auditor shall consider the need to obtain legal advice.

STEP 3:

 If sufficient information about suspected non-compliance cannot be obtained, the auditor


shall evaluate the effect of the lack of SAAE on the auditor’s opinion

 The auditor shall evaluate the implications of non-compliance in relation to other


aspects of the audit, including the auditor’s risk assessment and the reliability of written
representations, and take appropriate action.

IV. REPORTING OF IDENTIFIED OR SUSPECTED NON-COMPLIANCE

1. Reporting Non-Compliance in the Auditor’s Report on the Financial Statements:

If the auditor concludes that the non- If the auditor is precluded by management
compliance has a material effect on the or those charged with governance from
financial statements, and has not been obtaining sufficient appropriate audit
adequately reflected in the financial evidence to evaluate whether non-
statements compliance that may be material to the
financial statements has

Qualified or adverse opinion Qualified opinion or disclaim an opinion

2. Reporting Non-Compliance to Regulatory and Enforcement Authorities:

 If the auditor has identified or suspects non-compliance with laws and regulations, the
auditor shall determine whether the auditor has a responsibility to report the identified
or suspected non-compliance to parties outside the entity

3. Reporting Non-Compliance to Those Charged with Governance:

 The auditor shall communicate with those charged with governance matters involving
noncompliance with laws and regulations that come to the auditor’s attention during the
course of the audit.

148
FINAL C.A. – AUDIT

 If it is believed to be intentional and material then auditor shall communicate the matter
to those charged with governance as soon as practicable.

V. INDICATE THE POSSIBLE AREAS OR ASPECTS WHERE YOU MAY HAVE TO LOOK
OUT FOR FORMING AN OPINION AS TO WHETHER YOUR SUSPICION HAS SOME
BASED TO FURTHER INQUIRE
1. Investigation by regulatory organizations government departments or payment of
fines, additional taxes or penalties.

2. Payment for unspecified services or loans to consultants related parties or employees.

3. Sales commission or agent’s fees that appear excessive in relation to those ordinarily
paid by the entity or in it’s industry or to the services actually received.

4. Purchases at prices significantly above or below market price.

5. Unusual payments in cash.



6. Unusual payments towards legal and retainership fees.

7. Unusual transactions with companies registered in tax havens.

8. Payments for goods or services made other than to the country from which the
goods or services originated.

9. Payments without proper exchange control documentation.

10. Existence of an information system which fails to provide an adequate audit trial.

11. Unauthorized transactions or improperly recorded transactions.

12. Adverse media comment.

149
FINAL C.A. – AUDIT

QUESTIONS ASKED IN PAST/RTP/MOCK TEST

Question 1:
What are the roles and responsibilities of Statutory Auditor in relation to compliance with the
laws and regulations by the entity.

Answer:

Question 2:
As a statutory auditor of the company, comment on the following: While verifying the employee
records of a company, it was found that a major portion of the labour employed was child
labour. On questioning the management, the auditor was told that it was outside his scope of
the financial audit and to look into complainace with other laws.

Answer:
Auditor’s responsibility for consideration of other laws:

 SA 250 “Consideration of laws and regulation in an Audit of Financial statements” requires


the auditor to obtain sufficient appropriate audit evidence regarding the compliance with
the provisions of those laws and regulations generally recognized to have a direct impact
on the determination of material amounts and disclosures in the financial statements
including tax and labour laws.

 For other laws, the auditor’s responsibility is limited to undertake specified audit
procedures to help identify non-compliance with those laws and regulations that may
have a material effect on the financial statements.

 Non-compliance with other laws and regulations may result in fines, litigation or other
consequences for the entity, the costs of which may need to be provided for.

 In the instant case, major portion of the labour employed was child labour.

CONCLUSION: Auditor should enclose the disclosure of the above fact and provision of the

150
FINAL C.A. – AUDIT

cost of fines, litigation or other consequences. In case auditor concludes that non-compliance
may have a material effect on financial statements, he should modify his opinion accordingly.

Question 3:
As an auditor of TRP Ltd., you are suspicious that there might be non-compliance with laws
and regulations to which the company is subject to. Indicate the possible areas or aspects
where you may have to look out for forming an opinion as to whether your suspicion has some
based to further inquire.

Answer:

151
FINAL C.A. – AUDIT

SA 260
COMMUNICATION
WITH TCWG

I. OBJECTIVE:
i. To communicate clearly with TCWG
 the responsibilities of the auditor in relation to the FS audit, and
 an overview of the planned scope and timing of the audit

ii. To provide TCWG with


 timely observations arising from the audit that are significant and
 relevant to their responsibility to oversee the financial reporting process.

iii. To promote effective two-way communication

II. WHO ARE TCWG:


 TCWG refers to person or persons or organisation or organisations

 with the responsibility of overseeing the strategic direction of the company and

 obligation related to accountability of the entity.

 The auditor shall determine the appropriate person(s) within the


entity’s governance structure with whom to communicate.
 Governance structures vary by entities, reflecting influences such as different
cultural and legal backgrounds, and size and ownership characteristics.
 For Example- In some entities, a supervisory (wholly or mainly non-executive)

board exists that is legally separate from an executive (management) board


(a “two-tier board” structure)
 If the auditor communicates with a subgroup of those charged with governance,

for example, an audit committee, or an individual, the auditor shall determine


whether the auditor also needs to communicate with the governing body.

152
FINAL C.A. – AUDIT

III. MATTERS TO BE COMMUNICATED:

a. The Auditor’s b. Planned Scope c. Significant d. Auditor’s


Responsibilities and Timing of Findings from Independence
in Relation to the Audit the Audit
the FS Audit

A. THE AUDITOR’S RESPONSIBILITIES IN RELATION TO THE FINANCIAL


STATEMENT AUDIT:
The auditor shall communicate with TCWG the responsibilities of the auditor in
relation to the financial statement audit, including that:
a) The auditor is responsible for forming and expressing an opinion on the FS that
have been prepared by management with the oversight of TCWG; and
b) The audit of the FS does not relieve management or TCWG of their responsibilities

B. PLANNED SCOPE AND TIMING OF THE AUDIT:


 How the auditor plans to address the significant ROMMS, whether due to fraud or

error.
 How the auditor plans to address areas of higher assessed ROMMS.

 The auditor’s approach to relevant internal control

 The application of the concept of materiality.

 The nature and extent of specialized skill or knowledge needed to perform the planned

audit procedures or evaluate the audit results, including the use of an auditor’s
expert.
 When SA 701 applies, the auditor’s preliminary views about matters that may be
areas of significant auditor attention in the audit and therefore may be key audit
Matters

C. SIGNIFICANT FINDINGS FROM THE AUDIT


 The auditor’s views about significant qualitative aspects of the entity’s accounting

practices, including accounting policies, accounting estimates and financial


statement disclosures.

 Significant difficulties, if any, encountered during the audit. It may include such
matters as:
 Significant delays by management, the unavailability of entity personnel, or

153
FINAL C.A. – AUDIT

an unwillingness by management to provide information necessary for the


auditor to perform the auditor’s procedures.
 An unreasonably brief time within which to complete the audit.

 Extensive unexpected effort required to obtain sufficient appropriate audit

evidence.
 The unavailability of expected information.

 Management’s unwillingness to make or extend its assessment of the entity’s

ability to continue as a going concern when requested


 Restrictions imposed on the auditor by management

 Circumstances that affect the form and content of the auditor’s report, if any.

D. AUDITOR’S INDEPENDENCE:
 In the case of listed entities, the auditor shall communicate with those charged

with governance

 A statement that the engagement team and others in the firm as appropriate,
the firm and, when applicable, network firms have complied with relevant ethical
requirements regarding independence

IV. THE COMMUNICATION PROCESS:


 The auditor shall communicate with TCWG the form, timing and expected general

content of communication

 The auditor shall communicate in writing with TCWG regarding significant findings
from the audit if, in the auditor’s professional judgment, oral communication would
not be adequate.

 Written communications need not include all matters that arose during the course
of the audit.

 The auditor shall communicate in writing with TCWG regarding auditor independence

 The auditor shall evaluate whether the two-way communication between the auditor
and TCWG has been adequate for the purpose of the audit.

154
FINAL C.A. – AUDIT

 If it has not, the auditor shall evaluate the effect, if any, on the auditor’s assessment
of the ROMMS and ability to obtain sufficient appropriate audit evidence, and shall
take appropriate action

V. TIMING OF COMMUNICATIONS:
Communications regarding planning matters :Made early in the audit engagement.

Significant difficulty encountered during audit : as soon as practicable.

 When SA 701 applies,


 the auditor may communicate preliminary views about key audit matters when
discussing the planned scope and timing of the audit and
 may have more frequent communications to further discuss such matters

when communicating about significant audit findings.



VI. TIMING OF COMMUNICATIONS:
 Communication regarding independence may be appropriate whenever significant

judgments are made about threats to independence and related safeguards.

 Communication regarding findings from the audit, may be made as part of the
concluding discussion.

VII. DOMUNETATION:
Where matters are communicated orally

The auditor shall include them in audit documentation, and when to whom they were
communicated

Where matters have been communicated in writing, the auditor shall retain a copy of the
communication as part of the audit documentation

QUESTIONS ASKED IN PAST/RTP/MOCK TEST

Question 1:
Reporting to Shareholders Vs reporting to TCWG

155
FINAL C.A. – AUDIT

Answer:

REPORTING TO SHAREHOLDERS REPORTING TO TCWG


1. SA 700, 701 705 and 706 and section 1. SA 260 deals with provisions relating
143 of the Companies Act, 2013 deals to reporting to those Charged with
with the provisions relating to reporting Governance.
to Shareholders
2. Reporting to shareholder generally 2. Reporting to TCWG generally includes
focuses on true and fair view of financial auditor’s responsibilities, planned scope
statements. and timing of audit, significant findings
from the audit and independence
3. Reporting to Shareholders is an 3. Reporting to TCWG is an internal report
external report and issued in public and not issued in public domain.
domain

KOI SHAK?????

Any questions???????

V. FACTORS AFFECTING MODE OF COMMUNICATION:


 Whether a discussion of the matter will be included in the auditor’s report, e .g. key

Audit Matters.

 Whether the matter has been satisfactorily resolved.

 Whether management has previously communicated the matter.

 The size, operating structure, control environment, and legal structure of the entity.

 In the case of an audit of special purpose F.S., whether the auditor also audits the
entity’s general purpose F.S.

156
FINAL C.A. – AUDIT

 Legal requirements.

 The expectations of TCWG, including arrangements made for periodic meetings or


communications with the auditor.

 The amount of ongoing contact and dialogue the auditors has with TCWG.

 Whether there have been significant changes in the membership of a governing


body.

157
FINAL C.A. – AUDIT

SA 265
COMMUNICATING DEFICIENCIES IN INTERNAL
CONTROL TO TWCG AND MANAGEMENT

I. OBJECTIVE:
 To communicate appropriately to TCWG and management deficiencies in internal
control
 that the auditor has identified during the audit and

 that, in the auditor’s professional judgment, are of sufficient importance to merit

their respective attentions

II. DEFINITION
 Deficiency in internal control – This exists when:

1. A control is designed, implemented or operated in such a way that it is unable


to prevent, or detect and correct, misstatements in the financial statements
on a timely basis; or
2. A control necessary to prevent, or detect and correct, misstatements in the
financial statements on a timely basis is missing.

III. REQUIREMENTS:
 The auditor shall determine whether, on the basis of the audit work performed, the

auditor has identified one or more deficiencies in internal control

 If the auditor has identified one or more deficiencies in internal control, the auditor
shall determine, on the basis of the audit work performed, whether, individually or
in combination, they constitute significant deficiencies

 The auditor shall communicate in writing significant deficiencies in internal control


identified during the audit to TCWG and management on a timely basis

IV. CONTENTS OF COMMUNICATION:


A. A description of the deficiencies and an explanation of their potential effects.

158
FINAL C.A. – AUDIT

i. Auditor may not quantify those effects


ii. May include written communication suggesting remedial action
iii. Whether auditor has taken steps to verify whether management’s responses
have been implemented

B. Sufficient information to enable TCWG to understand the context of the


communication. In particular, the auditor shall explain that:

i. The audit included consideration of internal control relevant to the preparation


of the financial statements in order to design audit procedures that are
appropriate in the circumstances, but not for the purpose of expressing an
opinion on the effectiveness of internal control
ii. The matters being reported are limited to those deficiencies that the auditor
has identified during the audit and that the auditor has concluded are of
sufficient importance to merit being reported to those charged with governance

C. Law or regulation may require the auditor or management to furnish a copy of the
auditor’s written communication on significant deficiencies to appropriate regulatory
authorities. Where this is the case, the auditor’s written communication may
identify such regulatory authorities.

159
FINAL C.A. – AUDIT

SA 299
RESPONSIBILITY OF
JOINT AUDITOR

I. DEFINITION:
 A Joint audit is an audit of financial statements
 of an entity by two or more auditors

 appointed with the objective of issuing the audit report.

 Such auditors are described as joint auditors

II. PRIOR TO THE COMMENCEMENT OF THE AUDIT, THE JOINT AUDITORS SHALL
DISCUSS AND DEVELOP A JOINT AUDIT PLAN. IN DEVELOPING THE JOINT
AUDIT STRATEGY, THE JOINT AUDITORS SHALL:
 Identify division of audit areas and common audit areas amongst the joint auditors

that define the scope of the work of each joint auditor

 Ascertain the reporting objectives of the engagement to plan the timing of the
audit and the nature of the communications required.

 Consider the results of preliminary engagement activities and, where applicable,


whether knowledge gained on other or similar engagements performed earlier by the
respective engagement partner(s) for the entity is relevant.

 Ascertain the nature, timing and extent of resources necessary to perform the
engagement.

 ROMMS need to be considered and assessed by each of the joint auditors and shall
be communicated to other joint auditors, and documented, whether pertaining to
the overall financial statements level or to the area of allocation among the other
joint auditors

 The joint auditors shall discuss and document the nature, timing, and the extent of
the audit procedures for common and specific allotted areas of audit to be performed

160
FINAL C.A. – AUDIT

by each of the joint auditors and the same shall be communicated to those charged
with governance.

 The joint auditors shall obtain common engagement letter and common
management representation letter.

III. RESPONSIBILITY DIVISION

JOINT AND SEVERAL RESPONSIBILITY DISTINCT RESPONSIBILITY

Undivided scope of Audit (NA2MES) Divided scope Audit

N  Audit work which is not divided Divided scope Audit


A1  Decisions taken by All auditors in
respect to NTE Each auditor is entitled to assume
A2  – Presentation and disclosure of that other joint Auditor has
fin. Statement as per applicable discharged his responsibilities
FRF regarding allotted work.

RESPONSIBILITY DIVISION

JOINT AND SEVERAL RESPONSIBILITY DISTINCT RESPONSIBILITY

Undivided scope of Audit (NA2MES) Divided scope Audit

M Matter which are brought to the Divided scope Audit


notice of JA by any one of them
E Examining that the financial Each auditor is entitled to assume
statement comply with requirement that other joint Auditor has
of statutes discharged his responsibilities
S Ensuring report complies regarding allotted work.
with applicable SA’s and
pronouncements issued by ICAI.

161
FINAL C.A. – AUDIT

IV. CO-ORDINATION AMONGST JOINT AUDITOR


a. If a JA comes across matters which are
 relevant to the areas of responsibility of other joint auditors and
 which deserve their attention, or

 which require disclosure or


 require discussion with,

 or application of judgment by other joint auditors

the said JA shall communicate the same to all the other JAs in writing prior to the
completion of the audit.

b. Each joint auditor is entitled to assume that:

i. The other joint auditors have carried out their part of the audit work and the
work has actually been performed in accordance with the SAs issued by the
ICAI.

ii. The other joint auditors have brought to said joint auditor’s notice
 any departure from AFRF or
 significant observations that are relevant to their responsibilities noticed
in the course of the audit.

c. Where financial statements of a division/branch are audited by one of the joint


auditors, the other joint auditors are entitled to proceed on the basis that
 such financial statements comply with all the legal and regulatory requirements
and present a true and fair view of the state of affairs
 and of the results of operations of the division/branch concerned.

 Before finalizing their audit report, the joint auditors shall discuss and
communicate with each other their respective conclusions that would form the
content of the audit report.

III. REPORTING RESPONSIBILITIES :


 The joint auditors are required to issue common audit report

 However, where the joint auditors are in disagreement with regard to the opinion

162
FINAL C.A. – AUDIT

or any matters to be covered by the audit report, they shall express their opinion
in a separate audit report.

 Each joint auditor with a differing opinion would be required to issue a separate
audit report and the reference to the other joint auditors report would be required
to be made by each such joint auditor in their respective audit report.

 A joint auditor is not bound by the views of the majority of the joint auditors
regarding the opinion or matters to be covered in the audit report and shall
express opinion formed by the said joint auditor in separate audit report in case of
disagreement

 In such circumstances, the audit report(s) issued by the joint auditor(s) shall make
a reference to the separate audit report(s) issued by the other joint auditor(s).

 Further, separate audit report shall also make reference to the audit report issued
by other joint auditors.

 Such reference shall be made under the heading “Other Matter Paragraph” as per
SA 706(Revised), “Emphasis of Matter Paragraphs and Other Matter Paragraphs
in the Independent Auditor’s Report.

QUESTIONS ASKED IN PAST/RTP/MOCK TEST

Question 1:
KRP Ltd, at its annual General meeting appointed Mr X ,Mr Y, and Mr Z as joint Auditors
to conduct auditing for the Financial Year, 2014-15.For the valuation of gratuity scheme of
the Co., Mr X , Mr Y and Mr Z wanted to refer their own known Actuaries. Due to difference
of opinion ,all the joint auditors consulted their respective actuaries. Subsequently, major
difference was found in the actuary reports. However Mr X agreed to Mr Y’s actuary report,
though Mr Z did not . Mr X contends that Mr Y actuary report shall be considered in audit
report due to majority of votes. Now, Mr Z is in dilemma.

i) You are required to briefly explain the responsibilities of auditors when they are jointly
and severally responsible in respect of audit conducted by them and also guide Mr Z in
such situation.

163
FINAL C.A. – AUDIT

ii) Explain the responsibility of Auditors, in case,report made by Mr Y’s actuary, later on,
found faulty.

Answer:
Responsibility of Joint Auditor & reporting responsibilities of Joint Auditor

Question 2:
Dice Ltd. appointed two CA firms MN & Associates and PQ & Company as joint auditors
for conducting Audit for the year ending 31st March, 2020. In the course of Audit, it has
been observed that there is a major understatement in the value of inventory. The inventory
valuation work was looked after by MN & Associates but there was no documentation for the
division of work between the joint auditors. Comment on the above situation with regards to
responsibilities among joint auditors.

Answer:
Responsibility of Joint Auditor

Question 3:
P Limited is a Limited company and it’s business activities are divided into three regions. The
Company appointed PY & Co. , and MK & Co., Chartered Accountants to conduct a joint
Audit and report on the financial statements for the Financial year 2019-2020. Explain the
relationship among the joint auditors for the audit of the financial statements for the year
2019-2020.
OR
Write a short note on Responsibility of Joint Auditors

Question 4:
Your firm is one of the joint Auditors of FMP Ltd. Under what circumstances Joint Auditors
are jointly liable for the work in relation to audit of Financial statements? Is there any
restriction on a joint Auditor to communicate a dissenting note differing from the majority
opinion of the other joint auditors in the audit report issued under section 143 of Companies
Act, 2013?

Answer:
Responsibility of Joint Auditor

164
FINAL C.A. – AUDIT

Question 5:
NMN & C0., LLP and ABC Associates, LLP are joint statutory auditors of BHS Ltd. BHS
Ltd is a listed company and has been in existence for the last 50 years. Since the beginning
this company was audited by MQS & Associates but due to audit rotation, the company had
to bring in new auditors. Considering the size of the Company, two auditors were appointed as
Joint Auditors. Since the company is new to these Auditors and the concept of joint auditors
to whom audit work has been divided, management had a discussion and understood that each
joint auditor is responsible only for the work allocated to him, whether or not he has prepared
a separate report on the work performed by him. Advise.

Answer:
Responsibility of Joint Auditor & co ordination

Question 6:
Excellent Bank Ltd. Is a Public Limited Company. The said bank has branches all over India.
The bank appoints 3 joint Auditors for the Financial year ending 31/03/2019. All the 3 Joint
Auditors divide the work with mutual consent. Verification of Consolidation, however, remained
undivided. All branches and zones were divided amongst the 3 Joint auditors. During the Audit
of zones, CA. Z, one of the joint auditors expressed a concern about internal control in one
of the large corporate branches situated in his zone. The irregularity was not reported in
the final account as the other 2 joint auditors were not in favour of reporting and decision
of not reporting the same was taken on the basis of majority. Subsequently, fraud has been
detected in the said branch which was audited by CA Z. The Bank seeks your advice about the
responsibility of the 3 joint auditors in the above situation.

Answer:
Responsibility of Joint Auditor & reporting responsibilities of Joint Auditor

165
FINAL C.A. – AUDIT

SA 300
PLANNING AN AUDIT OF
FINANCIAL STATEMENTS

RECURRING INITIAL

Preliminary Engagement activities Planning  Performing procedures


required by SA220 regarding
 Performing procedures required by SA acceptance of the client
220 regarding continuance of client relationship and the specific
relationship audit engagement
 Evaluating ethical compliance as  Communicate with previous

required by SA 220 auditor


 Terms of engagement as per SA 210

Planning

STRATEGY (CUD Not Report) PLAN


 Characteristics of engagement  NTE of RAP as per SA 315

 Use of knowledge i.e results of preliminary eng  NTE of RAP as per SA 330

activities  Other procedures


 Consider factors in auditors professional judgement

are significant in directing the engagement teams


work
 NTE of recourses necessary to perform the

engagement
 Reporting objectives of the engagement to plan timing

of audit and nature of communication required

Documentation
a) The overall audit strategy
b) The audit plan
c) Any significant changes made during the audit engagement to the overall audit strategy
or the audit plan, and the reasons for such changes

166
FINAL C.A. – AUDIT

SA 315
IDENTIFYING AND ASSESSING THE RISKS
OF MATERIAL MISSTATEMENT THROUGH
UNDERSTANDING THE ENTITY AND ITS
ENVIRONMENT

i. Objective
 The objective of the auditor is to identify and assess the risks of material misstatement,
whether due to fraud or error, at the financial statement and assertion levels,
 Through understanding the entity and its environment,

 Including the entity’s internal control,

 Thereby providing a basis for designing and implementing responses to the assessed risks

of material misstatement.
 This will help the auditor to reduce the risk of material misstatement to an acceptably

low level

II. SIGNIFICANT RISKS:

 An identified and assessed risk of material misstatement that, in the auditor’s judgment,
requires special audit consideration.

 Risks that require special audit consideration

1. Whether the risk is a risk of fraud.

2. Whether the risk is related to recent significant economic, accounting, or other


developments like changes in regulatory environment, etc., and, therefore, requires specific
attention.

3. The complexity of transactions.

4. Whether the risk involves significant transactions with related parties.

5. The degree of subjectivity in the measurement of financial information related to the


risk, especially those measurements involving a wide range of measurement uncertainty.

167
FINAL C.A. – AUDIT

6. Whether the risk involves significant transactions that are outside the normal course of
business for the entity, or that otherwise appear to be unusual.

III. THE RISK ASSESSMENT PROCEDURES SHALL INCLUDE THE FOLLOWING:


Inquiries of management, of appropriate individuals within the internal audit function (if the
function exists), and of others within the entity who in the auditor’s judgment may have
information that is likely to assist in identifying risks of material misstatement due to fraud
or error.

Analytical procedures

Observation and inspection

The auditor shall consider whether information obtained from the auditor’s client acceptance
or continuance process is relevant to identifying risks of material misstatement

Where the engagement partner has performed other engagements for the entity, the
engagement partner shall consider whether information obtained is relevant to identifying
risks of material misstatement.

When the auditor intends to use information obtained from the auditor’s previous experience
with the entity and from audit procedures performed in previous audits, the auditor shall
determine whether changes have occurred since the previous audit that may affect its relevance
to the current audit

The engagement partner and other key engagement team members shall discuss the susceptibility
of the entity’s financial statements to material misstatement.

The engagement partner shall determine which matters are to be communicated to engagement
team members not involved in the discussion.

168
FINAL C.A. – AUDIT

IV. IN ORDER TO PROVIDE A BASIS FOR FURTHER AUDIT PROCEDURES THE AUDITOR
SHALL:
Identify risks throughout the process of obtaining an understanding of the entity and its
environment, including relevant controls that relate to the risks, and by considering the
classes of transactions, account balances, and disclosures in the financial statements.

Assess the identified risks, and evaluate whether they relate more pervasively to the financial
statements as a whole and potentially affect many assertions.

Relate the identified risks to what can go wrong at the assertion level, taking account of
relevant controls that the auditor intends to test;

Consider the likelihood of misstatement, including the possibility of multiple misstatements,


and whether the potential misstatement is of a magnitude that could result in a material
misstatement

V. UNDERSTANDING THE ENTITY AND ITS ENVIRONMENT:

 The auditor shall obtain an understanding of the following:


Relevant industry, regulatory, and other external factors including the applicable financial
reporting framework

The nature of the entity, including:


(i) its operations;
(ii) its ownership and governance structures;
(iii) the types of investments that the entity is making and plans to make, including investments
in special-purpose entities; and
(iv) the way that the entity is structured and how it is financed

The entity’s selection and application of accounting policies, including the reasons for changes
thereto. The auditor shall evaluate whether the entity’s accounting policies are appropriate for
its business and consistent with the applicable financial reporting framework and accounting
policies used in the relevant industry.

169
FINAL C.A. – AUDIT

The entity’s objectives and strategies, and those related business risks that may result in risks
of material misstatement.

The measurement and review of the entity’s financial performance.

170
FINAL C.A. – AUDIT

SA 320
MATERIALITY IN PLANNING
& PERFORMING AN AUDIT

I. INTRODUCTION

Planning Performing Reporting

SA 450
SA 320

II. DETERMINING MATERIALITY

Planning For individual class of transaction,


Materiality balance & disclosures

For eg: 50% of planning


materiality.
i.e performance materiality

The auditor’s determination of materiality is a matter of professional judgment, and is


affected by the auditor’s perception of the financial information needs of users of the
financial statements. In this context, it is reasonable for the auditor to assume that users:

1) Have a reasonable knowledge of business and economic activities and accounting and a
willingness to study the information in the financial statements with reasonable diligence

2) Understand that financial statements are prepared, presented and audited to levels of
materiality

3) Recognize the uncertainties inherent in the measurement of amounts based on the use of
estimates, judgment and the consideration of future events;

171
FINAL C.A. – AUDIT

4) Make reasonable economic decisions on the basis of the information in the financial
statements

Note: The materiality determined when planning the audit does not necessarily establish
an amount below which uncorrected misstatements, individually or in aggregate, will
always be evaluated as immaterial.

III. PERFORMANCE MATERIALITY


The auditor shall determine performance materiality for purposes of
 assessing the ROMMS and

 determining the NTE of further audit procedure.


Planning the audit solely to detect individually material misstatements overlooks the fact
that the aggregate of individually immaterial misstatements may cause the FS to be
materially misstated, and leaves no margin for possible undetected misstatements.

Performance materiality (which, as defined, is one or more amounts) is set to reduce
to an appropriately low level the probability that the aggregate of uncorrected and
undetected misstatements in the financial statements exceeds materiality for the
financial statements as a whole.

The determination of performance materiality is not a simple mechanical calculation and
involves the exercise of professional judgment.

IV. REVISION AS THE AUDIT PROGRESSES


The auditor shall revise materiality for
 the financial statements as a whole

 (and, if applicable, the materiality level or levels for particular classes of transactions,

account balances or disclosures)


in the event of becoming aware of information during the audit that would have
caused the auditor to have determined a different amount (or amounts) initially.

If the auditor concludes that a lower materiality for the financial statements as a whole (and,
if applicable, materiality level or levels for particular classes of transactions, account balances
or disclosures) than that initially determined is appropriate, the auditor shall determine
whether

172
FINAL C.A. – AUDIT

 it is necessary to revise performance materiality, and

 whether the NTE of the FAP remain appropriate


For eg, if during the audit it appears as though actual financial results are likely to be
substantially different from the anticipated period end financial results that were used
initially to determine materiality for the financial statements as a whole, the auditor
revises that materiality.

V. DOCUMENTATION
The audit documentation shall include the following amounts and the factors considered in
their determination:

 Materiality for the financial statements as a whole

 If applicable, the materiality level or levels for particular classes of transactions, account
balances or disclosures

 Performance materiality

 Any revision of (a)-(c) as the audit progressed

VI. BENCHMARK SELECTION


Factors that may affect identification of an appropriate benchmark
(FOCUS on VIOLEN)

Focus- whether there are items on which the attention of the users of the particular entity’s
financial statements to be focused. (for eg: profit, revenue, NA)

V- The relative volatility of the benchmark.


I- Industry and economic environment in which entity operates.
O- The entity’s ownership structure and the way it is financed.
L- Where the entity is at its life cycle.
E- Elements of the financial statements
N- Nature of the entity

173
FINAL C.A. – AUDIT

QUESTIONS ASKED IN PAST/RTP/MOCK TEST


Question 1:
Mr. X was appointed as the auditor of M/s Easy go Ltd. And intends to apply the concept of
materiality for the financial statements as a whole. Please guide him as to the factors that
may affect the identification of an appropriate benchmark for this purpose.
Answer:
Focus on Violen

Question 2:
As an auditor of RST Ltd. Mr. P applied the concept of materiality for the financial statements
as a whole. On the basis of obtaining additional information of significant contractual
arrangements that draw attention to a particular aspect of a company’s business, he wants
to re-evaluate the materiality concept. Please guide him.
Answer:
Revision of materiality

174
FINAL C.A. – AUDIT

SA 330
THE AUDITOR’S RESPONSES
TO ASSESSED RISKS

I. OBJECTIVE
a) The objective of the auditor is to
l obtain SAAE about the assessed ROMMS,

l through designing and implementing appropriate responses to those risks

II. REQUIREMENTS


A. Overall responses to the B. Responses to the assessed risks
assessed risks of material of material misstatement at the
misstatement assertion level

A. Overall responses to the assessed risks of material misstatement ( PUNE’S)

P- Emphasizing to the audit team the need to maintain professional skepticism


U- Incorporating additional elements of unpredictability in the selection of further audit
procedures to be performed
N- Making general changes to the nature, timing or extent of audit procedures
E- Assigning more experienced staff or those with special skills or using experts
S- Providing more supervision

B. Responses to the assessed risks of material misstatement at the assertion level



Auditor Shall determine the approach to be used. For example
a) Compliance approach (includes only test of controls) for a particular assertion
b) Substantive approach (includes only test of details) for a particular assertion
c) Combined approach (includes test of details and test of controls) for verifying
transactions, balances and disclosures.

The nature of an audit procedure refers to its purpose (i.e., test of controls or substantive

175
FINAL C.A. – AUDIT

procedure) and its type (i.e., inspection, observation, inquiry, confirmation, recalculation,
reperformance, or analytical procedure). The nature of the audit procedures is of most
importance in responding to the assessed risks.

Timing of an audit procedure refers to when it is performed, or the period or date to
which the audit evidence applies

Extent of an audit procedure refers to the quantity to be performed, for example, a
sample size or the number of observations of a control activity

III. DEFINITIONS

Test of controls – Substantive procedure –


An audit procedure designed to An audit procedure designed to detect
evaluate the operating effectiveness material misstatements at the assertion level.
of controls in preventing, or detecting Substantive procedures comprise:
and correcting, material i) Tests of details (of classes of transactions,
misstatements at the assertion level account balances, and disclosures), and
ii) Substantive analytical procedures

TEST OF CONTROLS

In designing and performing tests of controls, the auditor shall:


Perform other audit procedures in combination with inquiry to obtain audit evidence about the
operating effectiveness of the controls, including:
i) How the controls were applied at relevant times during the period under audit.
ii) The consistency with which they were applied.
iii) By whom or by what means they were applied

The auditor shall test controls for the particular time, or throughout the period. When the
auditor obtains audit evidence about the operating effectiveness of controls during an interim
period, the auditor shall:
i) Obtain audit evidence about significant changes to those controls subsequent to the
interim period; and

176
FINAL C.A. – AUDIT

ii) Determine the additional audit evidence to be obtained for the remaining period

If the auditor plans to use audit evidence from a previous audit about the operating effectiveness
of specific controls, the auditor shall establish the continuing relevance of that evidence by
obtaining audit evidence about whether significant
changes in those controls have occurred subsequent to the previous audit

The auditor shall determine whether the controls to be tested depend upon other controls
(indirect controls), and if so, whether it is necessary to obtain audit evidence supporting the
effective operation of those indirect controls.
A/c system
Bank Book Controls for this are
indirect control

BRS

Bank Statement

SUBSTANTIVE PROCEDURES (TEST OF DETAILS)

Irrespective of the assessed ROMMS, the auditor shall design and perform substantive
procedures for each material class of transactions, account balance, and disclosure

The auditor shall consider whether external confirmation procedures are to be performed as
substantive audit procedures

The auditor’s substantive procedures shall include the following audit procedures related to the
FS closing process:
i) Agreeing or reconciling the FS with the underlying accounting records; and
ii) Examining material journal entries and other adjustments made during the course of
preparing the FS

The auditor shall perform audit procedures to evaluate whether the overall presentation of the
FS, including the related disclosures, is in accordance with the AFRF

177
FINAL C.A. – AUDIT

If the auditor has not obtained SAAE as to a material financial statement assertion, the
auditor shall attempt to obtain further audit evidence. If the auditor is unable to obtain
SAAE, the auditor shall express a qualified opinion or a disclaimer of opinion

IV. DOCUMENTATION
• overall responses to address the assessed risks of material misstatement at the financial
statement level

• the nature, timing and extent of the further audit procedures performed and The linkage
of those procedures with the assessed risks at the assertion level

• The results of the audit procedures, including the conclusions where these are not
otherwise clear

QUESTIONS ASKED IN PAST/RTP/MOCK TEST


Question 1:
In the course of audit of Z Ltd., its auditor wants to rely on audit evidence obtained in previous
audit in respect of effectiveness of internal controls instead of re-testing the same during the
current audit. As an advisor to the auditor kindly caution him about the factors that may
warrant a re-test of controls
Answer:
As per SA 330 on “The Auditor’s Responses to Assessed Risks”, if the auditor plans to use
audit evidence from a previous audit about the operating effectiveness of specific controls, he
shall establish the continuing relevance of that evidence by obtaining audit evidence about
whether significant changes in those controls have occurred subsequent to the previous audit.
• The auditor’s decision on whether to rely on audit evidence obtained in previous audits for
control is a matter of professional judgment.
Factors that may warrant a re-test of controls are-
(1) A deficient control environment.
(2) Deficient monitoring of controls.
(3) A significant manual element to the relevant controls.
(4) Personnel changes that significantly affect the application of the control
(5) Changing circumstances that indicate the need for changes in the control.
(6) Deficient general IT-controls.

178
FINAL C.A. – AUDIT

Question 2:
While carrying out the statutory audit of a large entity, what are the substantive procedure
to be performed to assess the ROMM?
Answer:
Definition of test of controls & test of details

Question 3:
While commencing the statutory audit of B Company Limited, the auditor undertook the risk
assessment and found that the detection risk relating to certain class of transactions cannot
be reduced to acceptance level. Explain
Answer:
SA315”ldentifying and Assessing the Risk of Material Misstatement Through Understanding
the Entity and its Environment” and SA 330 “The Auditor’s Responses to Assessed Risks”
establishes standards on the procedures to be followed to obtain an understanding of the
accounting and internal control systems and on audit risk and its components.
“SA 315 and SA 330 require that the auditor should use professional judgment to assess
risk of material misstatement and to design audit procedures to ensure that it is reduced to
an acceptably low level.
“Risk of Material Misstatements comprises of Inherent risk and Control Risk. “Detection
risk is the risk that an auditor’s substantive procedures will not detect a misstatement that
exists in an account balance or class of transactions that could be material.
The higher the ROMMS, the more audit evidence the auditor should obtain from the
performance of substantive procedures. When both inherent and control risks are assessed
as high, the auditor needs to consider whether substantive procedures can provide sufficient
appropriate audit evidence to reduce detection risk, and therefore audit risk, to an acceptably
low level.
“The auditor should use his professional judgment to assess audit risk and to design audit
procedures to ensure that it is reduced to an acceptably low level. If it cannot be reduced to
an acceptable level, the auditor should express a qualified opinion or a disclaimer of opinion as
may be appropriate.

Question 4:
While commencing the statutory audit of ABC Company Limited, what should be the
considerations of the auditor to assess Risk of Material Misstatement and his response to
such risks?
Answer:
179
FINAL C.A. – AUDIT

Considerations of Auditor for Assessing the Risk of Material Misstatement:


SA 315 identifying and Assessing the Risk of Material Misstatement through understanding
the Entity and its Environment”, requires the auditor to identify and assess the risks of
material misstatement at the financial statement level; and the assertion level for classes of
transactions, account balances, and disclosures to provide a basis for designing and performing
further audit procedures. For this purpose, the auditor shall:
(i) Identify risks throughout the process of obtaining an understanding of the entity and
its environment, including relevant controls;
(ii) Assess and evaluate the identified risks;
(iii) Relate the identified risks to what can go wrong at the assertion level; and
(iv) Consider the likelihood of misstatement.

Auditor’s Responses to the Assessed Risk of Material Misstatement:


• As per SA 330 The Auditor’s Responses to Assessed Risks”, the auditor shall design and
implement overall responses to address the assessed risks of material misstatement. In
designing the audit procedures to be performed, the auditor shall:
i) Consider the reasons for assessment given to ROMM at the assertion level for each class
of transaction, account balance, and disclosure , and
ii) Obtain more persuasive audit evidence for the auditor’s assessment of risk.

180
FINAL C.A. – AUDIT

SA 402
AUDIT CONSIDERATIONS RELATING
TO AN ENTITY USING A SERVICE
ORGANIZATION.

I. OBJECTIVE
 To obtain an understanding of the nature and significance of the services provided by the

service organization and their effect on the user entity’s internal control relevant to the
audit, sufficient to identify and assess the ROMMS

 To design and perform audit procedures responsive to those risks.

II. REQUIREMENTS: (RAP)


A. Obtaining an Understanding of the Services Provided by a Service Organization Including
Internal Control:

i. If a client uses a service organization, certain policies, procedures and record maintained
by the service organization might be relevant to the audit of FS of the client

ii. The auditor would consider the nature and extent of the activities by SO as to determine
whether those activities are relevant to the audit & if so, to assess their effect on audit
risk.

iii. As per SA 402, when obtaining an understanding of the user entity in accordance with
SA 315, the user auditor shall obtain an understanding of how a user entity , uses the
services of a service organization including:

(NaMa, NaRe Degree wager NaSe)

NaMa: Nature & materiality of the transactions processed


NaRe: Nature of relationship between the user entity & the SO
Degree: Degree of interaction between the user entity & the SO
NaSe: Nature of services provided by the SO & the significance of those services to the
user entity.

181
FINAL C.A. – AUDIT

iv. Info on nature of services provided by SO maybe available from a wide variety of sources,
such as user manuals,
ü system overviews,
ü technical manuals;
ü the contract or service level agreement between user entity & SO.

B. If the user auditor is unable to obtain a sufficient understanding from the user entity
the user auditor shall obtain that understanding from one or more of the following
procedures:
i. Obtaining a Type 1 report, if available
ii. Contacting the service organization, through the user entity, to obtain specific
information
iii. Visiting the service organization and performing procedures that will provide the
necessary information about the relevant controls at the service organization
iv. Using another auditor to perform procedures that will provide the necessary
information about the relevant controls at the service organization

III. TYPE 1 AND TYPE 2 REPORTS:


A. Type 1 report:
Report on description and design of control at a service organization

1. Description of SO’s accounts and internal control system, ordinally prepared by


management of SO &
2. An opinion by SO’s auditor that-
 Above description is accurate
 The systems control have been placed in operations

 The a/c and internal control system are suitably designed to achieve their stated

objective.

B. Type 2 report: Report on description and design of controls at a service organization and
its operating effectiveness
 All above in type 1

 The accounting and internal control systems are operating effectively based on the

results of tests of controls.

182
FINAL C.A. – AUDIT

IV. RESPONDING TO THE ASSESSED RISKS OF MATERIAL MISSTATEMENT IN


RESPONDING TO ASSESSED RISKS IN ACCORDANCE WITH SA 330, THE USER
AUDITOR SHALL: (FAP)
Determine whether SAAE concerning the relevant financial statement assertions is
available from records held at the user entity; and, if not,

Perform further audit procedures to obtain sufficient appropriate audit evidence or use
another auditor to perform those procedures at the service organization on the user
auditor’s behalf.

Test of Controls
 When the user auditor’s risk assessment includes an expectation that controls at the
service organization are operating effectively, the user auditor shall obtain audit evidence
about the operating effectiveness of those controls from one or more of the following
procedures:
1. Obtaining a Type 2 report, if available;
2. Performing appropriate tests of controls at the service organization; or
3. Using another auditor to perform tests of controls at the service organization on behalf
of the user auditor.

V. CAN USER AUDITOR RELY UPON THE TYPE 1 OR TYPE 2 REPORT ISSUED BY
SERVICE AUDITOR: In determining the sufficiency and appropriateness of the audit
evidence provide by a Type 1 or Type 2 report, the user auditor shall be satisfied as to:
(CAR’S PAD)

 C- Determine whether complementary user entity controls* identified by the service


organization are relevant to the user entity and, if so, obtain an understanding of
whether the user entity has designed and implemented such controls.

 A- The adequacy of the standards under which the Type 1 or Type 2 report was issued.

 R - Evaluating whether the tests of controls performed by the service auditor and the
results thereof, as described in the service auditor’s report, are relevant to the assertions
in the user entity’s financial statements

183
FINAL C.A. – AUDIT

 S- Evaluate the sufficiency and appropriateness of the evidence provided by the report
for the understanding of the user entity’s internal control relevant to the audit.

 P - The service auditor’s professional competence (except where the service auditor is a
member of the Institute of Chartered Accountants of India) and independence from the
service organization.

 A- Evaluating the adequacy of the time period covered by the tests of controls and the
time elapsed since the performance of the tests of controls

 D - Evaluate whether the description and design of controls at the service organization
is at a date or for a period that is appropriate for the user auditor’s purposes.

COMPLEMENTARY USER ENTITY CONTROLS


 Controls that the service organization assumes,

 In the design of its service, will be implemented by user entities,

 And which, if necessary to achieve control objectives,

 Are identified in description of its system.

VI. SUB-SERVICE ORGANISATION

The nature and extent of work to be performed by the user auditor regarding the services
provided by a subservice organization depend on the nature and significance of those
services to the user entity and the relevance of those services to the audit

If a service organization uses a subservice organization, the service auditor’s report may
either include or exclude the subservice organization’s relevant control objectives and
related controls in the service organization’s description of it system and in the scope
of the service auditor’s engagement. These two methods of reporting are known as the
inclusive method and the carve-out method, respectively.

If the Type 1 or Type 2 report excludes the controls at a subservice organization, and
the services provided by the subservice organization are relevant to the audit of the user
entity’s financial statements, the user auditor is required to apply the requirements of
this SA in respect of the subservice organization

184
FINAL C.A. – AUDIT

NOTE 1:
 The user auditor shall inquire of management of the user entity whether the service
organization has reported to the user entity, or whether the user entity is otherwise aware
of, any fraud, non-compliance with laws and regulations or uncorrected misstatements
affecting the financial statements of the user entity

NOTE 2:
 The user auditor shall modify the opinion in the user auditor’s report in accordance
with SA 705 if the user auditor is unable to obtain sufficient appropriate audit evidence
regarding the services provided by the service organization relevant to the audit of the
user entity’s financial statements.

NOTE 3: REFERENCE TO SERVICE AUDITOR’S REPORT IN THE AUDIT REPORT OF


USER AUDITOR:

The user auditor shall not refer to If reference to the work of a service
the work of a service auditor in the auditor is relevant to an understanding
user auditor’s report containing an of a modification to the user auditor’s
unmodified opinion unless required opinion then he may refer to the work
by law or regulation to do so. of service auditor.

The user auditor’s report shall indicate


that such reference does not diminish
the user auditor’s responsibility for
that opinion.

QUESTIONS ASKED IN PAST/RTP/MOCK TEST

Question 1:
G Ltd. Is a mobile phone operating company . Barring the marketing function, it had
outsourced the entire operations like maintenance of mobile infrastructure, customer billing,
payroll, accounting functions, etc. Assist the auditor of G Ltd. As to how he can obtain an
understanding of how G. Ltd. Uses the services of the outsourced agency in its operations
Answer:

185
FINAL C.A. – AUDIT

Question 2:
In the course of audit of Raja and Rank Ltd., the audit manager of Sharma and. Co. observed
that Raja and Rank Ltd. has outsourced certain activities to an outsourcing agency.

a. As the engagement partner, guide the audit manager in the assessment of services
provided by the outsourcing agency in relation to the audit.

b. Discuss the procedure to be applied in case the user auditor is unable to obtain a sufficient
understanding from the user entity
Answer:

186
FINAL C.A. – AUDIT

SA 450
EVALUATION OF
MISSTATEMENTS IDENTIFIED

I. REQUIREMENTS
 The auditor shall accumulate misstatements identified during the audit, other than those

that are clearly trivial.

II. MISSTATEMENTS MAY RESULT FROM:

1. An inaccuracy in gathering or processing data from which the financial statements are
prepared;

2. An omission of an amount or disclosure;

3. An incorrect accounting estimate arising from overlooking, or clear misinterpretation of,


facts; and

4. Judgments of management concerning accounting estimates that the auditor considers


unreasonable or the selection and application of accounting policies that the auditor
considers inappropriate.

III. THE AUDITOR SHALL DETERMINE WHETHER THE OVERALL AUDIT STRATEGY
AND AUDIT PLAN NEED TO BE REVISED IF:

The nature of identified misstatements and the circumstances of their occurrence


indicate that other misstatements may exist that, when aggregated with misstatements
accumulated during the audit, could be material

The aggregate of misstatements accumulated during the audit approaches materiality
determined in accordance with SA 320.

If, at the auditor’s request, management has examined a class of transactions, account
balance or disclosure and corrected misstatements that were detected, the auditor shall

187
FINAL C.A. – AUDIT

perform additional audit procedures to determine whether misstatements remain



The auditor shall communicate on a timely basis all misstatements accumulated during
the audit with the appropriate level of management, unless prohibited by law or regulation.
The auditor shall request management to correct those misstatements.

If management refuses to correct some or all of the misstatements communicated by the


auditor,
 the auditor shall obtain an understanding of management’s reasons for not making

the corrections and


 shall take that understanding into account when evaluating whether the FS as a

whole are free from material misstatement.

The auditor shall communicate with TCWG uncorrected misstatements and the effect
that they, individually or in aggregate, may have on the opinion in the auditor’s report,
unless prohibited by law or Regulation. The auditor shall request that uncorrected
misstatements be corrected

IV. WR FROM MANAGEMENT


 The auditor shall request a written representation from management and, where
appropriate TCWG
 whether they believe the effects of uncorrected misstatements are immaterial,
individually and in aggregate, to the financial statements as a whole.
 A summary of such items shall be included in or attached to the written

representation.

V. THE AUDIT DOCUMENTATION SHALL INCLUDE


i. The amount below which misstatements would be regarded as clearly trivial

ii. All misstatements accumulated during the audit and whether they have been corrected
and

iii. The auditor’s conclusion as to whether uncorrected misstatements are material,


individually or in aggregate, and the basis for that conclusion

188
FINAL C.A. – AUDIT

VI. MENTION THE CIRCUMSTANCES UNDER WHICH AUDITOR MAY CONSIDER


MISSTATEMENTS AS MATERIAL EVEN THOUGH THEY ARE BELOW THE LEVEL
OF MATERIALITY (CROMA)

C-Affects compliance with debt covenants or other contractual requirements

C- Makes a change in earnings or other trends, especially in the context of general


economic and industry conditions

R- Affects compliance with regulatory requirements

O- Is an omission of information not specifically required by the AFRF but which, in


the judgment of the auditor, is important to the users’ understanding of the financial
position, financial performance or cash flows of the entity

M- Has the effect of increasing management compensation, for example, by ensuring


that the requirements for the award of bonuses or other incentives are satisfied

A- Relates to the incorrect selection or application of an accounting policy that has


an immaterial effect on the current period’s financial statements but is likely to have a
material effect on future periods’ FS incentives are satisfied

VII. DISCUSS THE IMPACT OF UNCORRECTED MISSTATEMENTS IDENTIFIED


DURING THE AUDIT AND THE AUDITOR’S RESPONSE TO THE SAME

 In accordance with SA 450 “Evaluation of Misstatements identified during the Audit”,


the auditor shall determine whether uncorrected misstatements are material, individually
or in aggregate. In making this determination, the auditor shall consider

a) The size and nature of the misstatements, both in relation to particular classes of
transactions, account balances or disclosures and the financial statements as a whole

b) The effect of uncorrected misstatements related to prior periods on the relevant classes
of transactions, account balances or disclosures, and the financial statements as a
whole

189
FINAL C.A. – AUDIT

 The auditor shall communicate this with TCWG uncorrected misstatements and
the effect that they, individually or in aggregate, may have on the opinion in the
auditor’s report, unless prohibited by law or regulation.

 The auditor’s communication shall identify material uncorrected misstatements


individually. The auditor shall request that uncorrected misstatements be corrected

 Prior to evaluating the effect of uncorrected misstatements, the auditor shall


reassess materiality determined in accordance with SA 320, to confirm whether it
remains appropriate in the context of the entity’s actual financial results

 As per management, if effect of such uncorrected misstatement is immaterial then


the auditor shall request for a written representation from management and,
where appropriate, TCWG that whether they believe the effects of uncorrected
misstatements are immaterial, individually and in aggregate, to the FS as a whole. A
summary of such items shall be included in or attached to the written representation

 If the management refuses to adjust the financial information and the results of
extended audit procedures do not enable the auditor to conclude that the aggregate
of uncorrected misstatements is not material, the auditor should report accordingly

QUESTIONS ASKED IN PAST/RTP/MOCK TEST


Question 1:
In the course of audit of T Ltd., the audit team is not sure of the possible source of misstatements
in the financial statements. As the audit manager identify the sources of misstatements.
Answer:
Refer II.

Question 2:
The auditor of XY & Co. Ltd. Has intimated the management that certain misstatements
identified during the course of audit need to be corrected. As an auditor, discuss the impact of
such misstatements in case the management does not carry out the said corrections.
Answer:
Combo of III & IV

190
FINAL C.A. – AUDIT

SA 500
AUDITING & ASSURANCE
INTER CA

Q.1 Q.1 SUFFICIENT & APPROPRIATE AUDIT EVIDENCE:

LARGE PIZZA DOUBLE CHEEZE BURST PIZZA


SUFFICIENT (Quantity) APPROPRIATE (Quality)

AUDIT EVIDENCE FOR AIR CONDITIONER


1) Invoice
2) Installment voucher
3) Bank statement
4) Physical Verification

Q.1 SUFFICIENT & APPROPRIATE AUDIT EVIDENCE:


SUFFICIENCY: It is the measure of quantity of audit evidence.
The quantity is affected by
• OMMS
• Size & Characteristics of co.
• Materiality
• Population

APPROPRIATENESS:
It is measure of quality of audit evidence ie. Its relevance & reliability in providing
support for the conclusion on which auditors opinion is based.

191
FINAL C.A. – AUDIT

Q.2 METHODS OF OBTAINING AUDIT EVIDENCE

(ICAI observes, re performs, re calculates )


I- Inspection
C- Confirmation
A- Analytical Procedure
I- Inquiry
Observation
Re performance
Re calculate

Q.3 TYPES OF AUIDIT EVIDENCE:

BASED UPON BASED UPON


SOURCE OF INFO NATURE OF INFO

Internal External

Originates from sources Originates from • Visual evidence


within the entity. For sources outside • Oral Evidence
eg: sales invoice & bank the entity. For eg: • Written evidence
reconciliation statement purchase invoice &
bank statement

Q.4 RELIABILITY OF EVIDENCE:

(D2I2O)
D1- AE in documentary form is more reliable than evidence obtained orally.

D2- AE obtained directly by the auditor is more reliable than AE obtained indirectly

I1- Reliability is more when it is obtained from independent sources outside the entity

I2- Reliability is more when internal controls are strong

192
FINAL C.A. – AUDIT

O- Original documents are more reliable than photocopies

Q.5 EVALUATING WORK OF MANAGEMENT EXPERT:


i. Evaluate the competence, capabilities and objectivity of that expert:
• For this purpose, auditor may consider his qualification, membership of a professional
body or industrial association license to practice etc

ii. Obtain an understanding of the work of that expert:


• It may include areas of specialty, applicable professional standards and other legal
requirements.

iii. Evaluate the appropriateness of that expert’s work:


• With respect to following:

(a) Relevance and reasonableness of that expert findings and conclusion.

(b) Relevance and reasonableness of assumptions and methods used, and

(c) Relevance, completeness and accuracy of source data.

A. Using the work of the management’s expert :


The NTE of audit procedures in relation to the use of the work of the management expert
as audit evidence may be affected by such matters as (PK TIC NA)

P The auditor’s previous experience of the work of that expert.

K The auditor’s knowledge & experience of the management s expert’s field of expertise.

T Whether the management’s expert is subject to technical performance standard.

I The extent to which management can exercise control or influence over the work of
management expert.

C The nature and extent of any controls within the entity over the management’s experts
work.

193
FINAL C.A. – AUDIT

N The nature, scope & objectives of the management expert’s Work.

A The availability of alternative Sources of audit evidence.

(B)Information regarding the competence, capabilities & objectives of a management’s expert


may come from a variety of Sources , such as: (PK C2UP D2E)
P Personal experience with previous Work of that expert.

K Knowledge of that experts qualification.

C1 The relevance of that managements experts competence to the matter for which that
expert’s work will be used.

C2 The management expert’s competence with respect to relevant accounting requirements.

U whether unexpected events, changes in conditions, or the audit evidence obtained From
the results of audit procedure indicate that it may be necessary to reconsider the initial
evaluation of the competence, capabilities & objectivity of managements expert as the
audit progresses

P Published papers or books written by the Expert.

D1 Discussions with that expert

D2 Discussions with others who are familiar with the With that Experts work.

Q. 6: WHAT ARE THE VARIOUS ASSERTIONS AN AUDITOR IS CONCERNED WITH


WHILE OBTAINING AUDIT EVIDENCE FROM SUBSTANTIVE PROCEDURES?

(MPCOVER)
M- Measurements
P- Presentation and disclosure
C- Completeness
O- Occurrence
V- Valuation

194
FINAL C.A. – AUDIT

E- Existence
R- Rights and obligations

QUESTIONS ASKED IN PAST/RTP/MOCK TEST

Question 1:
Z ltd appointed an outside expert to assess accrued gratuity liability of the company. Based on
the said report , the company provides Rs.80 lakhs as gratuity in the FS

Answer:
Evaluation of managements experts

Question 2:
The auditor of SS ltd. Accepted the gratuity liability valuation based on the certificate issued
by a qualified actuary. However, the auditor noticed that the retirement age adopted is 65
years as against the existing retirement age of 60 years. The company is considering a proposal
to increase the retirement age.

Answer:
Evaluation of managements experts

Question 3:
GAP ltd possesses some investment for which there is no ready market and to assess its
fair market value it hires an expert, the result of which it can use in preparing its financial
statement. Being an auditor of company, state the matters which may affect the nature,
timing and extent of audit procedure to be adopted by you in the instant case.
Answer:
Evaluation of managements experts

Question 4:
Write a short note on inquiry
Answer:
• Inquiry is used extensively throughout the audit in addition to other audit procedure
• Inquiries may range from formal written inquiries to informal oral inquiries. However, in
oral inquiries , the auditor may consider it necessary to obtain WR from management
and TCWG to confirm such responses

195
FINAL C.A. – AUDIT

• Evaluating responses to inquiries is an integral part of the inquiry process


• Responses to inquiry may provide the auditor with information not previously possessed.
Alternatively, repsonses might provide information that differs significantly from other
information that the auditor has obtained.
• In some cases reponses to inquiries provide a basis for the auditor to modify or perform
additional audit procedure
• Responses to inquiry may provide the auditor with information not previously possessed.
Alternatively, repsonses might provide information that differs significantly from other
information that the auditor has obtained.
• In some cases reponses to inquiries provide a basis for the auditor to modify or perform
additional audit procedure

Question 5:
Reliability of audit evidence
Answer:
Reliability of audit evidence

196
FINAL C.A. – AUDIT

SA 501
AUDIT EVIDENCE- SPECIFIC
CONSIDERATIONS FOR SELECTED
ITEMS

Q.1 . INTRODUCTION
• As per SA501 ‘Audit Evidence- specific considerations for selected items.’ The auditor
should perform audit procedures to obtain SAAE regarding ___________.

• SA501 is additional guidance to that contained in SA 500 ‘Audit Evidence’ wrt certain
specific financial statements amounts & other disclosures.

Q.2 INVENTORIES:
i) Attendance at physical counting: (Ekta Kapoor & Osama Bin Laden Is Performing)
E- Evaluate management’s instructions and procedures for recording and controlling the
results of the entity’s physical inventory counting;
O- Observe the performance of management’s count procedures
I- Inspect the inventory
P- Perform test counts

Attendance at physical counting includes


• Inspecting the inventory to ascertain its existence and evaluate its condition, and
performing test counts;
• Observing compliance with management’s instructions and the performance of procedures
for recording and controlling the results of the physical inventory count
• Obtaining audit evidence as to the reliability of management’s count procedure.

ii) Counting at alternate date


• In addition to above
• Perform audit procedures to obtain audit evidence about whether changes in inventory
between the count date and the date of the financial statements are properly recorded.
• If it is not possible to do so, the auditor shall modify the opinion in auditors report in
accordance with SA 705.

iii) When inventory is under the custody of 3rd party

197
FINAL C.A. – AUDIT

• The auditor shall obtain SAAE by performing one or both of the following: a. Request
confirmation from the 3rd party as to the quantities & condition of inventory held on
behalf of the entry.

• b. Perform inspection or other audit procedures appropriate in circumstances. Examples


of other audit procedures include
Attending or arranging for another auditor to attend, the third party’s physical
counting of inventory, if practicable
Obtaining another auditor’s report, or a service auditor’s report, on the adequacy of
third party’s internal control for ensuring that inventory is properly counted and
adequately safeguarded
Inspecting documentation regarding inventory held by third parties, for example,
warehouse receipts.
Requesting confirmation from other parties when inventory has been pledged as
collateral

iv) If attendance is impracticable


• The auditor shall perform alternative audit procedure to obtain SAAE regarding the
existence & condition of inventory.
• If it is not possible to do so, the auditor shall modify the Report in accordance with SA
705

Q.3 LITIGATIONS & CLAIMS

(I M Late)
I- Inquiry of management & where applicable, others within the entry, including in house
legal counsel.

M- Reviewing minutes of meeting of those charges with governance & correspondence


between entry & external legal counsel.

L- Reviewing legal expense account.

If the auditor assess a ROMMS regarding litigation/ claims that have been identified or
when audit procedures performed indicate that other material litigation/ claim may exist, the
auditor shall in addition to the procedures required by other SA’s seek direct communication

198
FINAL C.A. – AUDIT

with the entry’s external legal counsel.

Q.4 SEGMENT REPORT


a Obtain an understanding of the methods used by management in determining segment
information, and

• Evaluating whether such methods are likely to result in disclosure in accordance with
AFRF

• Where appropriate, testing the application of such methods

b. Performing analytical procedures or other audit procedures appropriate in the


circumstances

QUESTIONS ASKED IN PAST/RTP/MOCK TEST


Question 1:
LMN Ltd. supplies navy uniforms across the country. The company has 4 warehouses at
different locations throughout the India and 5 warehouses at the borders. The major stocks
are generally supplied from the borders. LMN Ltd. appointed M/S OPQ & Co. to conduct its
audit for the financial year 2019-20. Mr. O partner of M/S OPQ & Co., attended all the
physical inventory counting conducted throughout the India but could not attend the same at
borders due to some unavoidable reason.
• You are required to advise M/s OPQ & Co..
• How sufficient appropriate audit evidence regarding the existence and condition of
inventory may be obtained?
• How an auditor is supposed to deal when attendance at physical inventory counting is
impracticable?

Question 2:
Crush Ltd. is a dealer in fast moving consumer goods, The Company has warehouses throughout
the country where the stocks are stored. The Auditor of the Company normally conduct
physical verification of stocks along with the Management at the end of the financial year.
However, the Auditor could not be physically present during stock-tacking at two places on
account of certain disturbances in the region. In the light of the above facts.
• How sufficient appropriate audit evidence regarding the condition and existence of
inventory may be obtained?

199
FINAL C.A. – AUDIT

• How an Auditor is supposed to deal when attendance at physical inventory counting is


impracticable?

Question 3:
You are the auditor of Easy Communications Ltd. for the year 2018-19. The inventory as at
the end of the year i.e. 31.3.19 was RS 2.25 crores. Due to unavoidable circumstances, you could
not be present at the time of annual physical verification. Under the above circumstances how
would you ensure that the physical verification conducted by the management was in order?
Answer:

Question 4:
“If inventory is material to the financial statements, the auditor shall obtain sufficient
appropriate audit evidence regarding the existence of inventory by attending the physical
inventory counting unless impracticable.” Disuses.
Answer:

Question 5:
Your firm has been appointed as the statutory auditors of GBM Private Limited for the
financial year 2018-19. While verification of company’s inventories as on 31st March 2019,
you found that the significant amount of inventories belonging to the company are held by
other parties. However, the company has kept all the records of the inventories maintained by
other parties. What is your duty as an auditor in order to ensure that third parties are not
such with whom the stock should not be held and the stock as disclosed in company’s records
actually belongs to them?
Answer:

200
FINAL C.A. – AUDIT

SA 505

EXTERNAL CONFIRMATION

Confirmation Request

AUDITOR 3RD PARTY

Direct written response



Q.1 MEANING
• Audit evidence obtained as a direct written response
• to the auditor for a third party (confirming party)
• in paper form or by electronic or other medium.

To  The Accounts Manager,


Name of the business
Address of the Business City                             
Sub : Balance Confirmation as on 31st March 20_  

Sir,  
Our auditors, M/s. X Y z & Co. are now engaged on their annual audit.  Would you please
confirm to them direct that the details of the amount due to us/due to you, on 31st March,
2xxx were as shown on the attached statement.  
If you agree to the balance due, please sign the confirmation slip below and return this
form to our auditors in the enclosed reply-paid envelope. If you do not agree the balance due,
please write the amount shown by your records on the confirmation slip below, together with
full particulars of how the amount is made up.  
This is not a request for payment but we should be grateful if you would reply as soon as
possible as the information is needed for audit purposes.   Your co-operation in this matter is
greatly appreciated.    
Thanking You,                                                                                               
Yours Truly,                                                                                                                         

201
FINAL C.A. – AUDIT

Q. 2 SITUATIONS WHERE EXTERNAL CONFIRMATIONS CAN BE USED

(BILLS PAT)
B- Bank Balance.
I- Investment purchased but delivery not taken.
L- Loan from lenders.
L- Long o/s share application money.
S- Stock hold by 3rd party.
P- Property title deeds held by 3rd parties.
A- Account balances.
T- Terms of agreements with 3rd parties.

Q.3 IMPORTANT TERMS:

A. POSITIVE CONFIRMATION
A request that the confirming party responds directly to the auditor indicating whether the
confirming party agrees or disagrees with the info in the request
Effects of positive request:
A positive external confirmation request asks the confirming party to reply to the auditor in
all cases, either by indicating the confirming party’s agreement with the given information, or
by asking the confirming party to provide information

B. NEGATIVE CONFIRMATION
• A request that the confirming party responds directly to the auditor only if confirming
party disagrees with the info in the request.

• Negative confirmations provide less persuasive audit evidence than positive confirmations.
Accordingly, the auditor shall not use negative confirmation requests as the sole
substantive audit procedure to address an assessed risk of material misstatement at the
assertion level unless all of the following are present:

• The auditor has assessed the risk of material misstatement as low and has obtained
sufficient appropriate audit evidence regarding the operating effectiveness of controls
relevant to the assertion

• The population of items subject to negative confirmation procedures comprises a large

202
FINAL C.A. – AUDIT

number of small, homogeneous, account balances, transactions or conditions

• A very low exception rate is expected

• The auditor is not aware of circumstances or conditions that would cause recipients of
negative confirmation requests to disregard such requests

SITUATIONS WHERE NEGATIVE CONFIRMATIONS CAN BE USED

(SALA)
S- Substantial no. of errors are not expected.

A- Auditors have no reasons to believe that respondent will disregard this request.

L- Large no. of small balances involved.

A- Assessed level of Inherent Risk is low.

C. NON-RESPONSE:
• A failure of the confirming party to respond, or fully respond, to a positive confirmation
request, or a confirmation request returned undelivered

D. EXCEPTION:
• A response that indicates a difference between information requested to be confirmed,
or contained in the entity’s records, and information provided by the confirming party

Q.5 MANAGEMENTS REFUSAL TO ALLOW THE AUDITOR TO SEND CONFIRMATIONS

(PIE)

P- Perform alternative audit procedures designed to obtain relevant & reliable audit
evidence.

I- Inquire as to management’s reason for refusal & seek audit evidence as to their validity
& reasonableness.

203
FINAL C.A. – AUDIT

E- Evaluate the implications of management’s refusal on auditors assessment of ROMMS


including the risk of fraud

• If the auditor concludes that management’s refusal to allow the auditor to send a
confirmation request in unreasonable

• or the auditor is unable to obtain relevant & reliable audit evidence from alternative
procedures,

• the auditor shall communicate with TCWG,

• the auditor shall also determine the implications for the audit & auditors opinion in
accordance with SA 705.

Q. 6 FACTORS TO BE CONSIDERED WHEN DESIGNING A CONFIRMATION REQUEST

(ALARM2)
A- The assertions regarding which external confirmation is being prepared.
L- The layout and presentation of the confirmation request.
A- Ability & willingness of the intended confirming party to confirm.
R- Identified risk of material misstatement, including fraud risk factors, if any.

• zM1-The mode of communication, whether it is in paper form, or by electronic or other


medium.

• M2-Management’s authorisation for the 3rd parties to respond to the auditor. It may
be noted that confirming parties may not be willing to respond to a confirmation request
not having management’s authorisation.

Q. 7 FACTORS CREATING DOUBT OVER RELIABILITY OF EXTERNAL CONFIRMATION

(MUDIT)
M- Managements intervention in the confirmation send by 3rd party
U- Unauthorized confirmation
D- Delayed response from 3rd party
I- Incomplete information given by 3rd party

204
FINAL C.A. – AUDIT

T- Doubt over integrity & reliability of third party.

EVALUATING THE EVIDENCE OBTAINED:


• When evaluating the results of individual external confirmation requests, the auditor may
categorise such results as follows:

• A response by the appropriate confirming party indicating agreement with the information
provided in the confirmation request, or providing requested information without exception

• A response deemed unreliable

• A non-response

• A response indicating an exception

RESULTS OF EXTERNAL CONFIRMATION PROCEDURE


• If the auditor identifies factors that give rise to doubts about the reliability of the
response to a confirmation request, the auditor shall obtain further audit evidence to
resolve those doubts

• If the auditor determines that a response to a confirmation request is not reliable, the
auditor shall evaluate the implications on the assessment of the relevant risks of material
misstatement, including the risk of fraud, and on the related nature, timing and extent
of other audit procedures

• In the case of each non-response, the auditor shall perform alternative audit procedures
to obtain relevant and reliable audit evidence

• The auditor shall investigate exceptions to determine whether or not they are indicative
of misstatements.

• If the auditor has determined that a response to a positive confirmation request is


necessary to obtain sufficient appropriate audit evidence, alternative audit procedures
will not provide the audit evidence the auditor requires. If the auditor does not obtain
such confirmation, the auditor shall determine the implications for the audit and the
auditor’s opinion in accordance with SA 705.

205
FINAL C.A. – AUDIT

QUESTIONS ASKED IN PAST/RTP/MOCK TEST

Question 1:
Write short notes on: External Confirmations in Audit

Question 2:
Write short notes on Situations where external confirmations can be used.

Question 3:
M Z. Who is appointed as an auditor of Elite co .Ltd wants to use confirmation request as
audit evidence during the course of audit. What are the factors to be considered by Mr.
Z when designing a confirmation request? Also state the effect of using positive external
confirmation request by M.Z

Question 4:
• The management of SLTd. Request you not to seek confirmation from its debtor. As the
auditor of S LTD., What can be appropriate response?
OR
• Never Permit Limited refused to allow you to get direct confirmation of the outstanding
balances of trade receivables. You want to ensure on grounds of materiality that atleast
outstanding Above a threshold limits to be to be confirmed and reconciliation is to be
carried out before Finalizing audit. If the company does not relent, how will you respond.

Question 5:
• The auditor of H LTD. Wanted to obtain a confirmation from its creditors. But the
management made a request to the auditor not to seek Confirmation from certain
creditors citing disputes. Can the auditor of H LTD. Accede to this Request?
(MAY 13:4MARKS)
OR
• The accountant of C LTD, Has requested you, not to send balance confirmation to
a particular group of trade receivable since the said balances are under dispute and
matters is pending in the court. As a statutory Auditor, how would you deal?

Question 6:
MOON Limited replaced its statutory auditor for the financial year 2018-19. During the
course of audit, the new auditor found a credit item of RS 5 lakhs. On enquiry the company

206
FINAL C.A. – AUDIT

explained him that it is, a very old credits balance. The creditor had either approached for the
payment nor he is traceable. Under the circumstances, no confirmation of the credit balance
is available.
Answer:
Auditor’s duty in case of non-availability of External Confirmation

• SA 505 “External Confirmations” provides that if the auditor has determined that a
response to a positive confirmation request is necessary to obtain sufficient appropriate
audit evidence, and alternative audit procedures will not provide that audit evidence the
auditor requires, he should determine the implications for the audit and the auditor’s
opinion in accordance with SA 705

• In the present case, the identities of trade payables are not traceable to confirm the
credit balance as appearing in the financial statements of the company. It is also not
a case of pending litigation. It might be a case that an income of Rs. 5 Lakhs had been
hidden in the previous year/ s.

• The statutory should examine the validity of the credit balance as appeared in the
company’s financial statements. He should obtain sufficient evidence in support of the
balance. He should apply alternative audit procedures to get documentary proof for the
transactions/ s and should not rely entirely on the management representation. Finally
he should include matter by the way of the qualification in his audit report to the
members.

Question 7:
• As an auditor how would you deal with the following: When the audit team visited the
client to perform substantive audit of debtor, the client produced ledger account of
customers and confirmations for the top 10 customers. One of the debtors was more
than 5 years old, but it had confirmed his balance.
Answer:
Auditor’s duties in case of doubt over reliability of external confirmations:
• SA 505 “External Confirmations” deal with the auditor’s use of external confirmation
procedures to obtain audit evidence. External confirmation is the process of obtaining
and evaluating audit evidence through a direct communication from a third party in
response to a request for information about a particular item affecting assertions made
by the management in the financial statements.

207
FINAL C.A. – AUDIT

• As per SA 505, the auditor is required to maintain a control over the process of selecting
those to whom a request will be sent out, the preparation and sending of confirmation
requests and responses to those requests. This is because there have been several cases
of clients presenting forged confirmation to auditors when such control was absent.

• In the present case, one of the debtors of more than 5 years old had conformed his
balance. The auditor should enquire into the debtor whose dues are outstanding for 5
years or more about his financial abilities and why he has not paid, reasons behind the
same, and if found adverse, the client should be advised to provide for “provisions for bad
debts” and also to confirm that it is not a forged confirmation.

Question 8:
During the course of audit of Star Limited the auditor received some of the confirmation of the
balances of trade payables outstanding in the balance sheet through external confirmation
by negative confirmation request. In the list of trade payables, there are number of trade
payables of small balances except one, old outstanding of 15 Lacs, of whom, no confirmation
on the credit balance received. Comment with respect to Standard of Auditing.
Answer:
Response to negative confirmation request:
• As per SA 505, “External Confirmation”, Negative Confirmation is a request that the
confirming party respond directly to the auditor only if the confirming party disagrees
with the information provided in the request.

• Negative confirmations provide less persuasive audit evidence than positive confirmations.
In case of negative confirmation request, confirming parties may be more likely to respond
indicating their disagreement with a confirmation request when the information in the
request is not in their favour, and less likely to respond otherwise.

• In the instant case, the auditor sent the negative confirmation requesting the trade
payables having outstanding balances in the balance sheet while doing of audit Star
Limited. One of the old outstanding of 15 Lac has not sent the confirmation on the
credit balance.

• Non-response for negative. Confirmation request does not mean that there is some
misstatement as negative confirmation request itself is to respond to the auditor only if
the confirming party disagrees with the information provided in the request

208
FINAL C.A. – AUDIT

• In the present case, considering the materiality of the account balance, the auditor may
examine subsequent cash disbursements or correspondence from third parties, and other
records, such as good received notes.

Question 9:
M/s ABC & Co., LLP are appointed auditors of Sharp Company Ltd. for the year ended 31st
March, 2019. As part of the audit process, they want to use confirmation procedures as audit
evidence during the course of audit. In view of the fact that positive confirmations are not
responded favorably, the firm also intends to use negative confirmation requests. What are
the factors to be considered for the same?

209
FINAL C.A. – AUDIT

SA 510
INITIAL AUDIT
ENGAGEMENT

Q.1 WHAT ARE THE INITIAL AUDIT ENGAGEMENT?

Initial audit engagement are the one in which either

The financial statements for the The financial statements for


prior period were not audited the prior period were
Audited by a predecessor auditor

II: AUDIT PROCEDURE FOR OBTAINIG SAAE REGARDING OPENINIG BALANCES

The auditor shall obtain SAAE by (PAM Also)


P- Closing balance of the preceeding periods have been correctly brought forward to
current period

A- Appropriate accounting policies reflected in the opening balances have been


consistently applied in the current periods FS, or changes are properly accounted
& disclosed

M- Opening balances do not contain any misstatement that materially affect the
current periods FS

ALSO PERFORMING ONE OR MORE OF THE FOLLOWING.


1. If Audited by another auditor:
 Auditor maybe able to obtain SAAE regarding opening balance by perusing the
copies of audited FS, including other relevant documents relating to prior period FS.
 Ordinarily, the current auditor can place reliance on the closing balance contained

in the FS for the preceding period, except when during performance of audit
procedures for the possibility of misstatements in opening balance is indicated.

210
FINAL C.A. – AUDIT

2. Audit for first time:


 Since opening balances represents effect of transaction and events of the preceding

period and accounting policies applied in preceding period, the auditor need to obtain
evidence having regard to nature of opening balance materiality of opening balances
and accounting policies. (NAM)
 Since it will not be possible for auditor to perform certain procedures, eg. Observing

physical verification of inventory auditor may obtain confirmation and perform


other audit procedures w.r.t. fixed assets, investments, etc.

If the auditor obtains audit evidence that the opening balances contain misstatements
that could materially affect the current periods financial statements

The auditor shall perform such additional audit procedures as are appropriate in the
circumstances to determine the effect on the current period’s financial statement

If the auditor concludes that such misstatements exist in the current period’s
financial statements

The auditor shall communicate the misstatements with the appropriate level of
management and TCWG

III. REPORTING

Unable to obtain SAAE Misstatement exist(Refer Cases)

Qualified Disclaim Qualified Adverse

Case 1: If the auditor concludes that the opening balances contain a misstatement that
materially affects the current period’s financial statements, and the effect of the
misstatement is not properly accounted for or not adequately presented or disclosed,
the auditor shall express a qualified opinion or an adverse opinion, as appropriate, in
accordance with SA 705

Case 2:
If the auditor concludes that/:
a. The current periods accounting policies are not consistently applied in relation to opening

211
FINAL C.A. – AUDIT

balances in accordance with the applicable financial reporting framework; or


b. A change in accounting policies is not properly accounted for or not adequately presented
or disclosed in accordance with the applicable financial reporting framework, the auditor
shall express a qualified opinion or an adverse opinion as appropriate in accordance with
SA 705

Case 3: If the predecessor auditor’s opinion regarding the prior period’s financial statements
included a modification to the auditor’s opinion that remains relevant and material
to the current period’s financial statements, the auditor shall modify the auditor’s
opinion on the current period’s financial statements in accordance with SA 705
(Revised) and SA 710

QUESTIONS ASKED IN PAST/RTP/MOCK TEST

Question 1:
You have been appointed as the auditor of good Health Ltd. for 2014-2015 which was audited
by C A Trustworthy in 2013-14. As the Auditor of the company state the steps you would take
to ensure that the closing Balances of 2013-14 have been brought to account in 2014-15. As
Opening balances and the Opening Balances do not contain misstatements.

Answer:
Refer q. no 2 from notes
PAM Also

Question 2:
What are initial Audit Engagements.?

Answer:
Refer q. no 1 from notes

Question 3:
In an initial audit engagement the auditor will have to satisfy about the sufficiency and
appropriateness of opening balance to ensure that may are free from misstatements, which
may materially affects the current financial statements .Lay down the audit procedure, you
will follow in cases (i) when the financial statements are audited for the preceding period by
another auditor; and (ii) when financial statements are audited for the first time.

212
FINAL C.A. – AUDIT

Answer:
Refer q. no 2 from notes
PMA Also

Question 4:
If , after performing the procedure, you are not satisfied about the correctness of Opening
balances ; what approach you will adopt in drafting your audit report .

Answer:
Refer q. no 3 from notes

Question 5:
In an initial audit engagement the auditor will have to satisfy about the sufficiency and
appropriateness of` Opening balances’ to ensure that they free from misstatements, which
may materially affect the current financial statements . Lay down the audit procedure, you
will follow, when financial statements are audited for the first time. If, after performing the
procedure , you are not satisfied about the correctness of ` Opening Balances’ what approach
you will adopt in drafting your report.?

Answer:
Refer q. no 2 from notes
PMA Also

Question 6:
CA Ashutosh has been appointed as an auditor of Awesome Health Ltd. for the financial year
2014-15 which was audited by CA. Amrawati in 2013-14. As the Auditor of Awesome Health
Ltd, state the steps that CA. Ashutosh would take to ensure that the Closing Balances of
the financial year 2013-14 have been brought to account to account in 2014-15 as Opening
Balance and the Opening Balance do not contain any misstatements?

Answer:
Refer q. no 2 from notes
PMA Also

213
FINAL C.A. – AUDIT

Question 7:
CA. Jack a recently qualified practicing Chartered Accountant got his first audit assignment
of Futura (P) L TD. For the financial year 2014-15. He obtained all the relavant appropriate
audit evidence for the items related to statement of profit and Loss. However , while auditing the
Balance Sheet items, CA. Jack left out obtaining appropriate audit evidence, say , confirmations,
from the outstanding Accounts Receivable amounting Rs 150 Lakhs , continued as it is from
the last year , on the affirmation of the management that there is not receipts and further
credits during the year . CA Jack therefore , excluded from the audit programme , the audit
of the accounts receivable on the understanding that it pertains to the preceding year which
was already audited by predecessor auditor, Comment.?

Answer:
Refer q. no 2 from notes
PMA Also

214
FINAL C.A. – AUDIT

SA 520
ANALYTICAL
PROCEDURES

CHAP 8: ANALYTICAL PROCEDURE SA 520

Substantive ( Risk assessment in SA 315 )

Q.1 MEANING:
 Evaluation of Financial Info
 Through analysis of plausible relationship
 Among both financial & non financial data

 It also encompasses investigation of fluctuations or relationships

 that are inconsistent with other relevant info

Q.2 FACTORS TO BE CONSIDERED FOR SAP:

(PADNI)

P- Predictability
Example: Between sales & cost of sales
between trade receivables & cash receipts

A- Availability of data
D- Disaggretation

N- Nature of Assertion
Example: More effective for completeness or valuation than rights or obligations.

I- Inherent risk

215
FINAL C.A. – AUDIT

Q.3 TECHNIQUES FOR SAP

TREND ANALYSIS RATIO REASONABLENESS MODELLING


Example: Comparison ANALYSIS TEST Example: Linear
of current date with Example: Trade Example: Occupancy regression
Prior period balance receivables with rates to room rentals (relationship between
or with trend of 2 or sales dependent variable
more periods and one or more
independent variable)

Q.4 PRECAUTIONS TO BE TAKEN BEFORE APPLYING ANAYTICAL PROCEDURE

(REDS)

R- Evaluate reliability of data

E- Develop an expectation of recorded amounts or ratios

D- Determine the amount of any difference of recorded amounts from expected value
that is acceptable without further investigation

S- Determine suitability of a particular SAP for a given assertion

Q. 5EXTENT OF RELIANCE ON RELIABILITY OF DATA:

(NSC2)
N- Nature & relevance of info
S- Source of information
C1- Comparability of the information available.
C2- Controls over the preparations

Q.6 RESULTS OF ANALYTICAL PROCEDURE:


If analytical procedures performed identify fluctuations, the auditor shall investigate
such differences by:
 Inquiring of management & obtaining appropriate AE for the same

 Performing other audit procedure

216
FINAL C.A. – AUDIT

Q.7 CONSIDERATION SPECIFIC TO PUBLIC SECTOR ENTITIES


1. May not be always relevant in audit of government or other non business public
sector entities

2. Public sector enterprises may have little direct relationship between revenue &
expenditure

3. Also industry data or statistics for comparative purpose may not be available in the
public sector.

4. However, other relationships may be relevant


For example: Variations in the cost per km of road construction
Number of vehicles acquired compared with vehicles retired

Q.8 INVESTIGATING RESULTS OF ANALYTICAL PROCEDURE:


If analytical procedures performed in accordance with the SA identify fluctuations or
relationships that are inconsistent with other relevant information or that differ from
expected values by a significant amount, the auditor shall investigate such differences by:
a) Inquiring of management & obtaining appropriate audit evidence relevant to
managements responses and

b) Performing other audit procedures as necessary in the circumstances/

 AE relevant to management’s responses may be obtained by evaluating those


responses taking into account the auditors understanding of the entity and its
environment, and with other audit evidence obtained during the course of the audit.

 The need to perform other audit procedures may arise when, for examples,
management is unable to provide an explanation, or the explanation, together with
the AE obtained relevant to managements response is not considered adequate

QUESTIONS ASKED IN PAST/RTP/MOCK TEST


Question 1:
What are the considerations to be kept in mind while performing analytical procedures on
date prepared by client.

217
FINAL C.A. – AUDIT

Answer:
REDS

Question 2:
In the audit of Hotel Great Hay Ltd its auditor wants to use the analytical procedure as
substantive procedure in respect of room rental income as well as pay roll costs, guide him as
to how it can be done.

Answer:
Payroll Cost:
No. of employees x Fixed rate

Room rental:
No. of rooms x
Tariff x
Occupancy rate

Question 3:
The reliability of data is influenced by its source and nature and its dependent on the
circumstances under which it is obtained. Accordingly, what are the relevant criteria which
determine whether the data is reliable for the purpose of designing substantive analytical
procedures?

Answer:
Extent of reliance

NSC2

Question 4:
In audit of DEF ltd the auditor had made use of certain analytical procedures with regard to
certain key data in the statement of P&L. The results obtained showed inconsistencies with
other relevant information. State the course of action that the auditor should take to ensure
that the ROMMS would be contained to a low level fixed as materiality level

Answer:
Investigation

218
FINAL C.A. – AUDIT

SA 530

AUDIT SAMPLING

AUDIT SAMPLING

Q.1 MEANING:
Audit Sampling means
 Application of audit procedure
 To less than 100% of the items within a population

 Of audit relevance such that all sampling units have a chance of selection

Q.2 Sampling Risk:


 The risk that that the auditor’s conclusion based on a sample may be different from

the conclusion
 If the entire population were subjected to the same audit procedure.

 Sampling risk can lead to two types of erroneous conclusions

Q.2 SAMPLING RISK

 In case of test of controls that  In case of test of controls that controls


controls are more effective then are less effective then they actually are
they actually are  In case of test of details that a material
 In case of test of details that a misstatement exist when in fact it
material misstatement doesn’t exist doesn’t.
when in fact it does .  This error affects audit efficiency
 The auditor is primarily concerned as it would lead to additional work to
with this type of error because it establish that initial conclusions were
affects audit effectiveness & is more incorrect
likely to lead to an inappropriate
audit opinion

219
FINAL C.A. – AUDIT

NON SAMPLING RISK


 The risk that the auditor reaches an erroneous conclusions for any reason not related

to sampling

Q.3 SAMPLING SELECTION METHOD


Random Sampling
(i) Simple Random Sampling:
Under this method, each unit of the whole population has an equal chance of being
selected
Example: if trade receivable balance within 50,000-2,00,000 are Selected
(ii) Stratified Sampling:
This method involves diving the whole population to be tested in few separate groups
called “strata” & taking a sample from each of them.
Example: Debtors 10-100
100-200
200-300

Q.3 SAMPLING SELECTION METHOD


Systematic Sampling
Sampling Interval = Sample Units/Sample Size
Example: SI= 1,00,000/1,000 = 10th Item

Haphazard Selection
Example: Based upon auditors professional judgment

Monetary unit Sampling


Example: Amounts above Rs.10,000

Block selection
Example: Jan to March full quarter

220
FINAL C.A. – AUDIT

Q.4 STATISTICAL & NON STATISTICAL

STATISTICAL NON STATISTICAL


Meaning Meaning
 This method of auditing is more  Under this method the sample
scientific then testing based on size is determined on the basis of
auditor’s own judgment because it personal experience & knowledge of
involves use of mathematical laws the auditor
of probability in determining the
appropriate sample size

Q.4 STATISTICAL & NON STATISTICAL

STATISTICAL NON STATISTICAL


Advantages Advantages
 More scientific  Simple to operate
 No personal bias  Traditional approach thus
 Results of sample can be evaluated understandable by audit staff
& projected in more reliable way of  Rotation of emphasis is done
sampling  Normally year end transactions
 Widely accepted way of sampling are checked in detail to ensure cut-
 Sample size does not increase in off procedures are adequate
proportion to size of population

Q.4 STATISTICAL & NON STATISTICAL

STATISTICAL NON STATISTICAL


Limitations Limitations
 Complex to operate  Unscientific method
 Not suitable for audit staff  There may be personal bias
having no knowledge of statistical  Sample may not be true
technique representative of population
 It may be time consuming

221
FINAL C.A. – AUDIT

Q.5 TOLERABLE LEVEL

Level up to which auditor can accept


misstatement/deviation

TOLERABLE MISSTATEMENT TOLERABLE RATE OF DEVIATION


 A monetary amount set by auditor  A rate of deviation set by auditor
 In respect of which the auditor  In respect of which the auditor
seeks to obtain seeks to obtain
 Appropriate level of assurance that  Appropriate level of assurance that
 The monetary amount set by the  The rate of deviation set by the
auditor is not exceeded by the auditor is not exceeded by the
actual MS in the popln actual deviation in the popln

Q.6 FACTORS INFLUENCING SAMPLE SIZE

(SIA DT)

S- Size of the organization under audit

I- State of internal Control

A- Adequacy & reliability of books & records

D- Degree of the desired confidence

T- Tolerable error range

Q.7 PRECAUTIONS TO BE TAKEN WHILE APPLYING TEST CHECK TECHNIQUES



(SCAM PABI)

S- Sample size
The number of items in a sample should be appropriately determined

222
FINAL C.A. – AUDIT

C- Classification
Transaction &balances should be classified and stratified if required

A- Thorough study of accounting system


The flow of transactions should be completely studied before adopting sampling

M- Analysis of Misstatement
Errors found in same should be analyzed properly.

P-Proper Planning
Proper plan for test checking should be devised & explained to audit staff

A- Areas not suitable for sampling


Auditor should take care to ensure that detailed complete checking is done for required
items

B- No bias
Same should be checked in unbiased way.

I-Thorough study of internal control system


Proper study of I.C system helps the auditor to devise further test check plans

Q.8 MEANING OF AUDIT SAMPLING, SAMPLE DESIGN,SAMPLE SIZE AND SAMPLE


SELECTION OF ITEMS FOR TESTING
Audit Sampling
Refer Q.1

Designing An audit Sample


While designing an audit sample auditor shall consider & study the population out of
which sample will be selected. If there is need to classify the population, he should apply
stratified sampling

Sample Size
Auditor shall determine sample size which is sufficient to reduce sampling risk to an
acceptably low level. Sample size depends upon expected error, tolerable error & acceptable
sample risk.

223
FINAL C.A. – AUDIT

Selection of Items for testing


The auditor shall select the items for testing in such a way that each sampling unit has a
chance of selection. Thus method of sampling adopted by the auditor should be unbiased.

Q.9 EVALUATING RESULTS OF AUDIT SAMPLING


1. The auditor shall evaluate
 The results of the sample , and

 Whether the use of audit sampling has provided reasonable basis for conclusion

2. Analyzing Deviation Observe Common Extend audit procedures to all


Features those items

3. Misstatement/Deviation Anomaly Obtain high degree of certainty


that such MS is not
representative of population

4. In case auditor concludes that audit sampling has not provided a reasonable basis
for conclusions about the population, the auditor should tailor (i.e extend) the
nature, timing and extent of audit procedures.

QUESTIONS ASKED IN PAST/RTP/MOCK TEST


Question 1:
Write a short note on statistical and non-statistical sampling

Answer:
Refer ans no.4

Question 2:
Write a short note on: Sample Risk
OR
While planning the audit of S Ltd. You want to apply sampling techniques. What are the risk
factors you should keep in mind?

Answer:
Refer ans no.2

224
FINAL C.A. – AUDIT

Question 3:
Write a short note on : Tolerable Misstatement.

OR

What is tolerable misstatement and tolerable rate of deviation.

Answer:
Refer ans no.5

Question. 4 :
Discuss the following. With reference to SA 530, meaning of audit sampling and requirements
relating to sample design, sample size and selection of items for testing.

Answer:
Refer ans no.8

Question 5:
“In cases where audit sample selection has been done on a random basis, no statistical process
for selection of samples needs to be followed.” Comment.

Answer:
Selection of samples on random basis:
 As per SA 530 “Audit Sampling” sampling means application of audit procedures to less
than 100% of items within a population of audit relevance such that all sampling units
have a chance of selection in order to provide the auditor with a reasonable basis on
which to draw conclusions about the entire population

 Statistical sampling is an approach to sampling that has the following characteristics:


a. Random selection of the sample items; and
b. The use of probability theory to evaluate sample results, including measurement of
sampling risk.

Essential features of statistical sampling are random selection and use of probability theory.
Examples of statistical sampling are Random Selection, Systematic Selection and Monetary
Unit Sampling.

225
FINAL C.A. – AUDIT

 Audit Sample Collection on a random basis ensures that all items within a population
have an equal chance of selection by the use of random number tables or random number
generators. This method is considered appropriate provided the population to be sampled
consists of reasonably similar units and false within a reasonable range.

 Conclusion:
For application of statistical sampling techniques, one of the prerequisites is selection on
random basis, no other statistical process for selection of samples need to be followed.

Question 6:
“An auditor while analyzing the errors in a sample need not consider the qualitative aspects of
errors detected.” Comment.

Answer:
Refer ans no.4

Question 7 :
Describe the principal methods of selection of sample
OR
The auditor should select sample items in such a way that the sample can be expected to be
the representative of the population. Comment.
OR
In the course of your Audit assignment of Indraprastha Ltd., you want to guide your
audit assistants in selecting sample items in such a way that sample can be expected to be
representative of the population and all items have an opportunity of being selected. Guide
your assistants with principal methods of collecting samples.

Refer ans no.4

Question 8:
Write a short note on : Haphazard Sampling

Answer:
Refer ans no.3

226
FINAL C.A. – AUDIT

Question 9 :
Write a short note on : Block Selection.

Answer:
Refer ans no.3

Question 10 :
Describe the principal method of design of the samples and its evaluation.

Answer:
Refer ans no.3

KOI SHAK?????

Any questions???????

227
FINAL C.A. – AUDIT

SA 540
ACCOUNTING ESTIMATES, INCLUDING FAIR VALUE
ACCOUNTING ESTIMATES AND RELATED DISCLOSURES

Q.1 OBJECTIVE
The objective of the auditor is to obtain SAAE whether in the context of the AFRF:

1. Accounting estimates, including fair value accounting estimates, in the financial


statements, whether recognised or disclosed, are reasonable; and

2. Related disclosures in the financial statements are adequate

BASIC CONCEPTS
a. Accounting Estimate: An approximation of monetary amount in
absence of a precise means of measurement

b. Fair value estimate: Estimates where there is estimation


uncertainty & absence of a precise means of
measurement of monetary amount

c. Estimation uncertainty: Susceptibility of an accounting estimate &


related disclosure to an inherent lack of
precision in its measurement

Q.2 EXAMPLES OF ACCOUNTING ESTIMATES AND FAIR VALUE ACCOUNTING


ESTIMATES:
A. ACCOUNTING ESTIMATES:
 Allowance for doubtful accounts.
 Inventory obsolescence.

 Warranty obligations.

 Depreciation method or asset useful life.

 Provision against the carrying amount of an investment.

228
FINAL C.A. – AUDIT

B. FAIR VALUE ACCOUNTING ESTIMATES:


 Complex financial instruments

 Share based payments

 Property or equipment held for disposal

 Certain assets or liabilities acquired in a business combination, including goodwill


and intangible assets

 Transactions involving the exchange of assets or liabilities between independent


parties without monetary consideration.

Q.3 A EXAMPLES OF ESTIMATES WITH LOW ESTIMATION UNCERTAINTY:


 Accounting estimates arising in entities that engage in business activities that are

not complex.

 Accounting estimates that are frequently made and updated because they relate to
routine transactions.

 Accounting estimates derived from data that is readily available, such as published
interest rate data or exchange-traded prices of securities. Such data may be
referred to as “observable” in the context of a fair value accounting estimate.

 Fair value accounting estimates where the method of measurement prescribed by


the AFRF is simple and applied easily to the asset or liability requiring measurement
at a fair value.

 Fair value accounting estimates where the model used to measure the accounting
estimate is well-known or generally accepted, provided that the assumptions or
inputs to the model are observable.

Q.3B EXAMPLES OF ESTIMATES WITH HIGH ESTIMATION UNCERTAINTY


 Accounting estimates relating to outcome of litigation.

 Fair value accounting estimates for derivative financial instruments not publicly

229
FINAL C.A. – AUDIT

traded.

 Fair value accounting estimates for which highly specialized entity-developed model
is used for which there are assumptions or inputs that cannot be observed in the
marketplace.

Q.4 AUDITORS RESPONSIBILITY

A. RISK ASSESSMENT B. FURTHER AUDIT


PROCEDURE PROCEDURE (Auditor’s Responses
to the Assessed Risks of Material
Misstatement)

A. RISK ASSESSMENT PROCEDURE: (MAE UAE MC)


M- How management identifies those transactions, events and conditions that may give rise
to the need for accounting estimates.

A- The requirements of the AFRF

E- The estimation making process adopted by the management:
(UAE MC)
U- How the management has assessed the effect of estimation uncertainty
A- The assumptions underlying accounting estimates
E- Whether management has used an expert.
M- Methods including the models used in making accounting estimates
C- Relevant controls

REVIEW OF OUTCOME OF ACCOUNTING ESTIMATES:
The outcome of an accounting estimate will often differ from the accounting estimate recognized
in the prior period financial statements.
By performing risk assessment procedures to identify and understand the reasons for such
differences, the auditor may obtain:

(a) Information regarding the effectiveness of management’s prior period estimation process,
from which the auditor can judge the likely effectiveness of management’s current process.

230
FINAL C.A. – AUDIT

(b) Audit evidence that is pertinent to the re-estimation, in the current period, of prior
period accounting estimates.

(c) Audit evidence of matters, such as estimation uncertainty, that may be required to be
disclosed in the financial statements.

 The review of prior period accounting estimates may also assist the auditor, in the
current period, in identifying circumstances or conditions that increase the susceptibility
of accounting estimates to, or indicate the presence of, possible management bias.

B. IDENTIFYING AND ASSESSING THE RISK OF MATERIAL MISSTATEMENT:


In identifying and assessing the ROMM, as required by SA 315, the auditor shall evaluate
the degree of estimation uncertainty associated with an accounting estimate.

Factors Influencing Degree of Estimation Uncertainty (ROSE for JP)


R- The existence of recognized measurement techniques that may mitigate estimation
uncertainty
O- The extent to which the accounting estimate is based on observable or unobservable
inputs.
S- The sensitivity of accounting estimate to change in assumptions
E- The availability of reliable data from external sources.
J- The extent to which the accounting estimate depends on judgment.
P- The length of the forecast period, and the relevance of data drawn from past
even to forecast future events.

Responses to the Assessed risks


 Based on the assessed risks of material misstatement, the auditor shall determine:

(a) Whether management has appropriately applied the applicable FRF.


(b) Whether the methods are appropriate and have been applied consistently.

Response to Significant Risks Estimation Uncertainty


 For accounting estimates that give rise to significant risks, the auditor shall evaluate the

following:
(a) How management has considered alternative assumptions or outcomes, and why it
has rejected them.
(b) Whether the significant assumptions used by management are reasonable.

231
FINAL C.A. – AUDIT

 If, in the auditor’s judgment, management has not adequately addressed the effects of
estimation uncertainty, the auditor shall, develop a range with which to evaluate the
reasonableness of the accounting estimate.

Measurement and Disclosures Related to Accounting Estimates:


 The auditor shall obtain sufficient appropriate audit evidence about whether the

accounting estimate and their disclosure in the financial statements is appropriate.


 For accounting estimates that give rise to significant risks, the auditor shall check

adequacy of the disclosure of their estimation uncertainty in the financial statements.

Indicators of Possible Management Bias (SOFO)


S- Changes in an accounting estimate, or the method for making it, where management
has made a subjective assessment that there has been a change in circumstances.
0- Use of an entity’s own assumptions for fair value accounting estimates when they are
inconsistent with observable marketplace assumptions.
F- Selection or construction of significant assumptions that yield a point estimate favorable
for management objectives.
0. Selection of a point estimate that may indicate a pattern of optimism or pessimism.

Written representations
 The auditor shall obtain written representations from management whether management

believes significant assumptions used by it in making accounting estimates are reasonable

Documentation
 The audit documentation shall include:
1. The basis for the auditor’s conclusion about the reasonableness of accounting
estimates and their disclosure that give rise to significant risks; and
2. Indicators of possible management bias, if any.

QUESTIONS ASKED IN PAST/RTP/MOCK TEST


Question 1:
State examples of accounting estimates that may have a high estimation uncertainty

Answer:
Examples in Q.3 B

232
FINAL C.A. – AUDIT

Question 2:
While auditing Z ltd, you observe certain financial statement assertions have been based on
estimates made by the management. As the auditor how do you minimize the risk of material
misstatements

Answer:
RAP

Question 3:
A Pvt ltd is engaged in the business of real estate. The auditor of the company requested
information the information from the management to review the outcome of accounting
estimates( like estimated costs considered for percentage completion etc) included in the prior
period financial statements and their subsequent re-estimation for the purpose of the current
period

Answer:
Review of outcome of accounting estimate

KOI SHAK?????

Any questions???????

233
FINAL C.A. – AUDIT

SA 550

RELATED PARTIES

Q.1 MEANING:

i. A related party as defined in the AFRF.

ii. Where the AFRF establishes minimal or no related party requirements

1.
PERSON X LTD PERSON PERSON

X LTD X LTD

Y LTD Y LTD Y LTD Y LTD

a person + or entity + directly + through one


has control or or indirectly or more
significant intermediaries
influence over the reporting
entity
2.
Y LTD Y LTD Y LTD Y LTD

Person Person

PERSON X LTD Z LTD Z LTD

Another entity + over which +directly or + directly or


reporting entity indirectly indirectly
has control Through one or more
or significant intermediaries
influence
234
FINAL C.A. – AUDIT

3. Another entity that is under common control with reporting entity.

X ltd Close Family Members BOD BOD

Y ltd Z ltd BOD BOD ABCD BCDE

X ltd Y ltd X ltd Y ltd

Common Controlling Owners who are close Common key management


ownership Family members personnel

Note: However, entities that are under common control by a state (i.e National , regional or local
government) are not considered related unless they engage in significant transactions
or share resources to a significant extent with one another.

Gujarat Govt

Agriculture Infra

However, Ind AS 24 does not give total exemption from reporting(refer question)

Q.2 AUDITORS RESPONSIBILITY

A. RISK ASSESSMENT B. FURTHER AUDIT


PROCEDURE PROCEDURE (Auditor’s
Responses to the Assessed
Risks of Material
Misstatement)

RISK ASSESSMENT PROCEDURE

Understanding the Entity’s Related Party Relationships and Transactions:


1. The identity of the entity’s related parties, including changes from the prior period
2. The nature of the relationships between the entity and these related parties
3. Whether the entity entered into any transactions with these related parties during the
period, and if so, the type and purpose of the transactions.

235
FINAL C.A. – AUDIT

Evaluating accounting system and Related internal controls:

Identify, account for, and disclose Authorize and approve significant


related party relationships and transactions and arrangements with
transactions in accordance with related parties and outside the normal
the AFRF course of business.

During the audit, the auditor shall remain alert, when inspecting records or documents,
for arrangements or other information that may indicate the existence of related party
relationships or transactions that management has not previously identified or disclosed to
the auditor.

If the auditor identifies significant transactions outside the entity’s normal course of business
when performing the audit procedures the auditor shall inquire of management about:
(a) The nature of these transactions; and
(b) Whether related parties could be involved

B. Further audit procedures (Auditor’s Responses to the Assessed Risks of Material


Misstatement)

If the auditor identifies arrangements or information that suggests the existence
of related party relationships or transactions that management has not previously
identified or disclosed to the auditor, the auditor shall determine whether the underlying
circumstances confirm the existence of those relationships or transactions.

During the audit, the auditor may inspect records or documents that may provide
information about related party relationships and transactions, for example:
 Entity income tax returns.
 Information supplied by the entity to regulatory authorities.

 Shareholder registers to identify the entity’s principal shareholders.

 Statements of conflicts of interest from management and those charged with

governance.

236
FINAL C.A. – AUDIT

 Records of the entity’s investments and those of its pension plans.


 Contracts and agreements with key management or those charged with governance.

 Significant contracts and agreements not in the entity’s ordinary course of

business.
 Life insurance policies acquired by the entity.

 Significant contracts re-negotiated by the entity during the period.

For identified significant related party transactions outside the entity’s normal course of
business, the auditor shall:

(a) Inspect the underlying contracts or agreements, if any, and


(i) Evaluate whether the business rationale (or lack thereof) of the transactions
suggests that they may have been entered into to engage in fraudulent
financial reporting or to conceal misappropriation of assets;
(ii) The terms of the transactions are consistent with management’s explanations

(b) The transactions have been appropriately accounted for and disclosed in accordance
with the applicable financial reporting framework

(c) Obtain audit evidence that the transactions have been appropriately authorised
and approved (internal control)

Assertions That Related Party Transactions Were Conducted on Terms Equivalent to


Those Prevailing in an Arm’s Length Transaction:Management’s support for the assertion
may include:
(i) Comparing the terms of the related party transaction to those of an identical or
similar transaction with one or more unrelated parties.
(ii) Engaging an external expert to determine a market value and to confirm market
terms and conditions for the transaction.
(iii) Comparing the terms of the transaction to known market terms for broadly similar
transactions on an open market.

Q.3 OTHER POINTS:


A. FORMING AN OPINION
In forming an opinion on the financial statements in accordance with SA 700, the
auditor shall evaluate:

237
FINAL C.A. – AUDIT

(a) Whether the identified related party relationships and transactions have been
appropriately accounted for and disclosed in accordance with the AFRF

(b)   Whether the effects of the related party relationships and transactions:
(i) Prevent the financial statements from achieving true and fair presentation
(ii) Cause the financial statements to be misleading

B. WRITTEN REPRESENTATIONS:
Where the AFRF establishes related party requirements, the auditor shall obtain WR
from management and, where appropriate, TCWG that:
(a) They have disclosed to the auditor the identity of the entity’s related parties and
all the related party relationships and transactions of which they are aware;
and
(b) They have appropriately accounted for and disclosed such relationships and
transactions in accordance with the requirements of the ramework.

C. AUDIT DOCUMENTATION:

In meeting the documentation requirements of SA 230 and other SAs, the auditor shall
include in the audit documentation the names of the identified related parties and the
nature of the related party relationships.

QUESTIONS ASKED IN PAST/RTP/MOCK TEST


Question 1:
Elaborate how the Statutory Auditor can verify the existence of related parties for the
purpose of reporting under Accounting Standard 18.

Answer:
FAP= examples

Question 2:
In the course of your audit you have come across a related party transaction which prima
facie appears to be biased. How would you deal with this?

238
FINAL C.A. – AUDIT

ANSWER:
Since it is the management, which is primarily responsible for identification of related parties,
SA 550 requires that to identify names of all known related parties, the auditor may inspect
records or documents that may provide information about related party relationships and
transactions.

In this case, the auditor is finding a related party transaction which prima facie appears to
be biased. So the auditor is required to confirm the same. For identified significant related
party transactions outside the entity’s normal course of business, the auditor shall inspect the
underlying contracts or agreements, if any, and evaluate whether:

1. The business rationale (or lack thereof) of the transactions suggests that they may have
been entered into to engage in fraudulent financial reporting or to conceal misappropriation
of assets
2. The terms of the transactions are consistent with management’s explanations; and
3. The transactions have been appropriately accounted for and disclosed in accordance
with the applicable financial reporting framework

The auditor should also obtain audit evidence that the transactions have been appropriately
authorised and approved. After obtaining further information on significant transactions
outside the entity’s normal course of business enables the auditor to evaluate whether fraud
risk factors, if any, are present and, where the applicable financial reporting framework
establishes related party requirements, to identify the risks of material misstatement

In addition, the auditor needs to be alert for transactions which appear unusual in the
circumstances and which may indicate the existence of previously unidentified related parties.
Where the applicable financial reporting framework establishes related party requirements,
the auditor shall obtain written representations from management and, where appropriate,
those charged with governance that they have disclosed to the auditor the identity of the
entity’s related parties and all the related party relationships and transactions of which they
are aware; and they have appropriately accounted for and disclosed such relationships and
transactions in accordance with the requirements of the framework.
Finally, the auditor should report on the basis of this fact that the related party relationships
and transactions prevent the financial statements from achieving true and fair presentation;
or they are not cause for the financial statements to be misleading.

239
FINAL C.A. – AUDIT

Question 3:
You are the Auditor of Power Supply Corporation Limited, a Government Company for the year
ended on 31st March 2018. The turnover of the Company for the period was ` 12,000 crores
from sale of power. During your audit, you found that the Company had procured Spares for
Transmitters for ` 850 crores from abroad through a Corporation by name Procurement and
Supply India Limited which is also owned and controlled by Government of India.

The Financial Statements of the Power Supply Corporation Limited, prepared in compliance
with Ind AS for the year ended on 31/03/2018 did not contain any additional disclosure
regarding the procurement of spares as referred to above. To your query as to whether any
disclosure regarding Related Party Transaction would be required, the Management of the
Corporation replied that no such disclosure would be necessary for transactions between
State Controlled Enterprises. Analyse this issue in finalizing the Audit Report.

Answer:
(i) As per Ind AS 24, “Related Party Disclosures”, a reporting entity is exempt from the
disclosure requirements in relation to related party transactions and outstanding
balances, including commitments, with
(i) a government that has control or joint control of, or significant influence over, the
reporting entity; and
(ii) another entity that is a related party because the same government has control or joint
control of, or significant influence over, both the reporting entity and the other entity
(ii) If a reporting entity applies the above exemption, it shall disclose the following about the
transactions and related outstanding balances referred to:

(1) the name of the government and the nature of its relationship with the reporting entity
(i.e. control, joint control or significant influence);
(2) the following information in sufficient detail to enable users of the entity’s financial
statements to understand the effect of related party transactions on its financial
statements:
(i) The nature and amount of each individually significant transaction; and
(ii) For other transactions that are collectively, but not individually, significant, a
qualitative or quantitative indication of their extent
(iii) Further, as per SA 550 Related Parties, in forming an opinion on the financial
statements in accordance with SA 700, the auditor shall evaluate whether the
identified related party relationships and transactions have been appropriately

240
FINAL C.A. – AUDIT

accounted for and disclosed in accordance with the applicable financial reporting
framework.
(iv) In the instant case, Power Supply Corporation Limited, a Government Company
has procured spares for transmitters for rupees 850 crore from abroad through a
corporation namely Procurement and Supply India Limited which is also owned and
controlled by Government of India.
Even after applying the exemption of Ind AS 24, Power Supply Corporation Limited
has to disclose the matters specified above (i.e.name of Government, natures of its
relationship with reporting entity, the nature and amount of transaction etc.)
(v) Contention of Management of Corporation regarding no requirement of disclosure
for transactions between State Controlled Enterprise in not tenable.

KOI SHAK?????

Any questions???????

241
FINAL C.A. – AUDIT

SA 560

SUBSEQUENT EVENTS

Q.1 MEANING:

To events occurring between The Facts that become known to the auditor
date of financial statements and after the date of auditors report
the date of auditors report

AS 4 ( Inquire IMP)

(INQUIRE IMP)

 INQUIRE- Inquiring management and where appropriate TCWG as to whether any


subsequent events have occurred which might affect the financial statements.

 I- Reading the entities latest subsequent interim financial statement.

 M- Reading minutes if any, of the meetings of shareholders, management and TCWG


that have been held after the date of financial statements and inquiring about the
matters discussed at any such meetings for which minutes are not yet available.

 P- obtaining any understanding of any procedure management has established to ensure


that subsequent events are identified.

242
FINAL C.A. – AUDIT

Facts that become known


to the auditor after the
date of auditors report

Before issuing After issuing AGM notice


AGM notice

Management Management If amendments are not possible (for


Amends* (if refuses eg LLL)Take appropriate action(if
possible) FS To amend FS management does
Not inform users of FS)
Report Rev
Modification Eg.: Discuss the matter
EOM Para at the AGM, issue public notice,
OM Para consider in the subsequent years report

Q.2 EXAMPLES OF SPECIFIC INQUIRY: ( CS & CA CARE)


C - Whether new commitments, borrowings or guarantees have been entered into

S - Whether sales or acquisitions of assets have occurred or are planned

C - Whether there have been increases in capital or issuance of debt instruments, such
as the issue of new shares or debentures, or an agreement to merge or liquidate has
been made or is planned

A - Whether any unusual accounting adjustments have been made or are


contemplated

C – Whether there have been any developments regarding contingencies

A - Whether any events have occurred or are likely to occur that will bring into question
the appropriateness of accounting policies used in the financial statements, as
would be the case, for example, if such events call into question the validity of the
going concern assumption

R - Whether any events have occurred that are relevant to the recoverability of assets
243
FINAL C.A. – AUDIT

E - Whether any events have occurred that are relevant to the measurement of
estimates or provisions made in the financial statements

Q3. Facts Which Become Known to the Auditor After the Date of the Auditor’s Report but
Before the Date the Financial Statements are Issued:
(A) The auditor has no obligation to perform any audit procedures regarding the
financial statements after the date of the auditor’s report. However, when, after
the date of the auditor’s report but before the date the financial statements are
issued, a fact becomes known to the auditor that, had it been known to the auditor
at the date of the auditor’s report, may have caused the auditor to amend the
auditor’s report, the auditor shall: (DIA)

D - Discuss the matter with management and, where appropriate, TCWG

I - Inquire how management intends to address the matter in the financial


statements

A - Determine whether the financial statements need amendment

(B) If management amends the financial statements, the auditor shall:


 Carry out the audit procedures necessary in the circumstances on the amendment.

(Report rev, EOM,OM etc)


 Unless prohibited by law:

1. Extend the audit procedures referred to such events up to the date of the new
auditor’s report and
2. Provide a new auditor’s report on the amended financial statements. The new
auditor’s report shall not be dated earlier than the date of approval of
3. the amended financial statements.

Q4. FACTS WHICH BECOME KNOWN TO THE AUDITOR AFTER THE FINANCIAL
STATEMENTS HAVE BEEN ISSUED:

(A) After the financial statements have been issued, the auditor has no obligation to
perform any audit procedures regarding such financial statements. However, when,
after the financial statements have been issued, a fact becomes known to the
auditor that, had it been known to the auditor at the date of the auditor’s report,

244
FINAL C.A. – AUDIT

may have caused the auditor to amend the auditor’s report, the auditor shall:
(DIA)

D - Discuss the matter with management and, where appropriate, those charged
with governance.

I - Inquire how management intends to address the matter in the financial


statements.

A - Determine whether the financial statements need amendment

(B) If the management amends the financial statements, the auditor shall:

1. Carry out the audit procedures necessary in the circumstances on the


amendment.

2. Review the steps taken by management to ensure that anyone in receipt of


the previously issued financial statements together with the auditor’s report
thereon is informed of the situation

(C) In some entities, management may not be required by the applicable law, regulation
or the financial reporting framework to issue amended financial statements and,
accordingly, the auditor need not provide an amended or new auditor’s report. However,
when management does not amend the financial statements in circumstances where
the auditor believes they need to be amended, then:

1. If the auditor’s report has not yet been provided to the entity, the auditor
shall modify the opinion as required by SA 705 and then provide the auditor’s
report; or

2. If the auditor’s report has already been provided to the entity, the auditor
shall notify management and, unless all of TCWG are involved in managing the
entity, TCWG, not to issue the financial statements to third parties before
the necessary amendments have been made.

245
FINAL C.A. – AUDIT

 If the financial statements are nevertheless subsequently issued without the


necessary amendments, the auditor shall take appropriate action, to seek to
prevent reliance on the auditor’s report

(D) If management does not take the necessary steps to ensure that anyone in receipt
of the previously issued financial statements is informed of the situation and does
not amend the financial statements in circumstances where the auditor believes
they need to be amended, the auditor shall notify management and TCWG

 If, despite such notification, management or those charged with governance do


not take these necessary steps, the auditor shall take appropriate action to seek to
prevent reliance on the auditor’s report.

Q.4 FORMAT OF ANSWER


SA 560:
 Subsequent events refer to events occurring

 between the date of FS and the date of Auditor’s report,

 and facts that become known to the auditor after the date of auditors report

 AS 4 “Contingencies and Events occurring after the Balance Sheet Date”,

1. adjustments to assets and liabilities are required for events occurring after
the balance sheet date
2. that provide additional information
3. materially affecting the determination of the amounts
4. relating to conditions existing at the balance sheet date
5. or that indicate that the fundamental accounting assumption of going
concern (i.e., the continuance of existence or substratum of the enterprise) is
not appropriate.
 AS 29:

1. AS 29 “Provisions, Contingent liabilities and Contingent Assets”,


2. future events that may affect the amount required to settle an obligation
3. should be reflected in the amount of a provision where there is sufficient
objective evidence that the will occur.

246
FINAL C.A. – AUDIT

KOI SHAK?????

Any questions???????

247
FINAL C.A. – AUDIT

QUESTIONS ASKED IN PAST/RTP/MOCK TEST

Question 1:

 Briefly explain: Audit procedures on subsequent events


OR
 Briefly describe the auditor’s responsibility regarding subsequent events?

Answer:
Meaning + Inquire IMP

Question 2:
Enquiry from management is helpful for the auditor to evaluate subsequent events. Discuss
specific enquiries in reference of SA 560, which might have affect on the financial statement
OR
M/s LMP Associates, Chartered Accountants while conducting the Audit of PQR Ltd. Want
to conduct an inquiry of management and those charged with Governance as to whether
any subsequent events have occurred which might affect the financial statements. Guide M/s
LMP Associates with the matters where the specific enquiry may be conducted to evaluate
subsequent events.

Answer:
Examples

Question 3:
A Co Ltd. has not included in the balance sheet as on 31-03-2015 a sum of Rs 1.50 crores
being amount in the arrears of salaries and wages payable to the staff for the last 2 years
as a result of successful negotiations which were going on during the last 18 months and
concluded on 30-04-2015.The auditor wants to sign the said balance sheet and give the audit
report on 31-05-2015. The auditor came to know the result of the negotiations on 15-05-
2015.

Answer:
The facts indicate the events are of adjusting nature as per AS 4 and requires adjustment in
Assets & liabilities, which has not been made. Auditor shall request management to adjust, if
management does not he shall qualify his report

248
FINAL C.A. – AUDIT

Question 4:
As the auditor of the company, comment on the following: A fire broke out on 15th May, 2019,
in which material worth Rs 50 lakhs which was lying in inventory since 1st March, 2019
was totally destroyed .The financial statements of the company have not been adopted till the
date of fire. The management of the company argues that since the loss occurred in the year,
2014-15., no provision for the loss needs to be made in the financial statements for 2018-19.

Answer:
The event will have no impact , subject to satisfaction in respect of non violation of going
concern concept. Hence management is correct.

Question 5:
Amudhan and Co., are the Auditors of XYZ Company Ltd., for the year ended on 31/03/2019.
the audit report for that year was signed by the Auditors on 04/05/2019. the Annual General
Meeting was decided to be held during the month of August 2019. On 06/05/2019, the
Company had received communication from the Central Government that an amount of Rs.
5800 Crore kept pending on account of incentives pertaining to financial year 2018-2019
had been approved and the amount would be paid to the company before the end of May 2019.
To a query to the Chief Financial Officer of the Company by the Board, it was informed that
this amount has not been recognised in the Audited Financial Statements in view of the same
not being released before the close of the financial year and due to uncertainty of the receipt.
Now, having received the amount, the Board of Directors wished to include this amount in
the Financial Statements of the company for the Financial Year ended on 31/03/2019. On
08/05/2019, the Board amended the accounts, approved the same and requested the Auditor
to consider this event and issue a fresh Audit Report for the year ended on 31/03/2019.
Analyse the issues involved and give your views as to whether or not the Auditor could accede
to the request of the Board of Directors.

Answer:
 Facts which become known to the Auditor After the date of the Auditor’s report but

Before the Date the Financial Statements are Issued:

 As per SA 560, “Subsequent Events”, the auditor has no obligation to perform any audit
procedures regarding the financial statements after the date of the auditor’s report.
However, when, after the date of the auditor’s report but before the date the financial
statements are issued, a fact becomes known to the auditor that, had it been known to

249
FINAL C.A. – AUDIT

the auditor at the date of the auditor’s report, may have caused the auditor to amend
the auditor’s report, the auditor shall:
i. Discuss the matter with management and, where appropriate, those charged with
governance
ii. Determine whether the financial statements need amendment and, if so
iii. Inquire how management intends to address the matter in the financial statements

 If management amends the financial statements, the auditor shall carry out the audit
procedures necessary in the circumstances on the amendment. Further, the auditor shall
extend the audit procedures and provide a new auditor’s report on the amended financial
statements. However, the new auditor’s report shall not be dated earlier than the date
of approval of the amended financial statements

 In the instant case, XYZ Company Ltd. received an amount of rupees 5800 crore on
account of incentives pertaining to year 2017-18 in the month of May 2018 i.e. after
finalisation of financial statements and signing of audit report. Board of Directors of
XYZ Ltd. amended the accounts, approved the same and requested the Amudhan &
Co. (auditor) to consider this event and issue a fresh audit report on the financial
statements for the year ended on 31.03.2018.

 After applying the conditions given in SA 560, Amudhan & Co. can issue new audit report
subject to date of audit report which should not be earlier than the date of approval of
the amended financial statements

250
FINAL C.A. – AUDIT

SA 570

Going Concern

Whether there are events or conditions may cast significant doubt on entity’s
ability to continue as a going concern

Request management to perform preliminary assessment about entity’s plan


to continue for going concern

Management Performs the Management refuses to


assessment Performs the assessment

Management Performs the Management refuses to Performs the


assessment assessment

Discuss with TCWG

TCWG refuses

Modify the opinion as per SA 705


Or withdraw

Management Performs the assessment Evaluate managements assessment

Going Concern is Going Concern is inappropriate


appropriate

Management Management refuses to


Amends FS Amends FS

GC Para or EOM Modify report as per SA 705


para
Adverse opinion

251
FINAL C.A. – AUDIT

Going Concern is appropriate

Material Material Uncertainty


Uncertainty Exists Doesnot Exists

Provide Adequate The end


disclosure

Management Management refuses to provides


provides disclosure
disclosure

The Auditor shall

 Modify the opinion as per SA 705


 In the Basis for Qualified(Adverse)

opinion section the auditor’s report,


state that a material uncertainty
exists that may cast significant doubt
on the entity’s ability to continue as a
GC and that the FS do not adequately
disclose the matter

Management provides disclosure

The auditor shall determine whether the financial statements:

a. Adequately disclose
 the principal events or conditions that may cast doubt on entity’s ability to continue

as a GC &
 Managements plans to deal with these

b. Disclose clearly
 that there is a material uncertainty related to events or conditions that may cast

significant doubt on entity’s ability to continue as a GC


 & therefore, that it may be unable to realize its assets and discharge its liabilities

in the normal course of business

252
FINAL C.A. – AUDIT

EVALUATING MANAGEMENT’S ASSESSMENT


 In evaluating management’s assessment of the entity’s ability to continue as a going

concern, the auditor shall cover the same period as that used by management to make
its assessment as required by the applicable financial reporting framework, or by law or
regulation if it specifies a longer period.

 If management’s assessment of the entity’s ability to continue as a going concern covers


less than twelve months from the date of the financial statements ,the auditor shall
request management to extend its assessment period to at least twelve months from that
date

 The auditor shall consider whether management’s assessment includes all relevant
information of which the auditor is aware as a result of the audit

 The auditor shall inquire of management as to its knowledge of events or conditions


beyond the period of management’s assessment that may cast significant doubt on the
entity’s ability to continue as a going concern

 Evaluating management’s plans for future actions in relation to its going concern
assessment, whether the outcome of these plans is likely to improve the situation and
whether management’s plans are feasible in the circumstances

 Considering whether any additional facts or information have become available since the
date on which management made its assessment

 Where the entity has prepared a cash flow forecast, and analysis of the forecast is

a significant factor in considering the future outcome of events or conditions in the


evaluation of management’s plans for future actions:
 Evaluating the reliability of the underlying data generated to prepare the forecast; and
 Determining whether there is adequate support for the assumptions underlying the
forecast.

253
FINAL C.A. – AUDIT

SIGNIFICANT DELAY APPROVAL OF FS BY MANAGEMENT & TWCG

if auditor believes that the delay could be related to events or conditions


relating to the GC assessment

a. the auditor shall perform those additional audit procedures necessary


b. consider the effect on the auditor’s conclusion regarding the existence of a material
uncertainty.

II: INDICATORS
A. FINANCIAL INDICATORS (FOTO CAN WAN)

 F: fixed terms borrowing approaching maturity W/O realistic prospect of renewals.

 O: substantial operating loss

 T: inability to comply with terms of loan agreement

 O: Inability to obtain financing for essential new product

 C: Inability to pay creditors

 A: Adverse Key financial ratio

 N: Net liability position

 W: indication of withdrawal of financial support by creditors.

 A: Arrears of dividend

 N: Negative operating cash Flows

B. OPERATING INDICATORS- (MK Sale)


 M: Management intentions to liquidate the entity.

 K: Loss of key management without Replacement.

254
FINAL C.A. – AUDIT

 S: Shortage of important supplies

 L: Labour difficulties

 E: Emergence of a highly successful competitor

 C: Other indicators (no puc)

 N: Non-Compliance with statutory requirements

 P: Pending legal proceedings, if successful may result in claims that entity is unlikely to
be able to satisfy.

 U: Uninsured or underinsured catastrophes when they occur.

 C: changes in law.

II. ADDITIONAL AUDIT PROCEDURE


 When events & conditions of Identified auditor shall perform the following additional

audit procedure. (ACA KI FD)

 A: Determining whether there is adequate support for the assumptions underlying


the forecast

 C: Whether the entity has prepared a cash flow forecast

 A: Requesting management to make assessment of entity’s ability to continue as a


going concern.

 F: Evaluating management plan for future actions in relation to its going concern
assessment.

 D. Evaluating the reliability of the under-lying data generated to prepare the


forecast

255
FINAL C.A. – AUDIT

KOI SHAK?????

Any questions???????

256
FINAL C.A. – AUDIT

QUESTIONS ASKED IN PAST/RTP/MOCK TEST

Question 1:
A Company’s net worth is eroded and trade payables are unpaid due to liquidity constraints.
The management represents to the unsecured loan to meet the liquidity constraints and that
negotiations are underway to secure large export orders.

Answer:
It is not a mitigiating factor, and hence modify opinion

Question 2:
Write short notes on financial indications to be considered for evaluating the assumption of
going concern

Answer:
Financial
Operating
Other

Question 3:
Yummy Ltd, dealing in manufacturing and trading if milk butter, has a benchmark in its
product for so many years. Tasty Ltd., a rival company to Yummy Ltd., has introduced its new
product, peanut butter. Due to being health conscious, the consumer has shifted from milk
butter to peanut butter within few months. This has result into massive loss during the year
to Yummy Ltd. Due to non – selling of perishable milk product. The company has also started
having negative net worth. Its production head, finance head and marketing head have also
left the company. The company has no sound action plan to mitigate these situations. Kindly
guide the auditor of Yummy Ltd., how he should deal with the situation.

Question 4:
Mr. Ram, an auditor, identified some events that cast significant doubt on the entity’s ability
to continue as a going concern. What are the additional procedures he should perform as per
the related standard on auditing?

Answer:
Additional Audit procedures

257
FINAL C.A. – AUDIT

Question 5:
Toddle Limited had definite plan of its business being closed within a short period from the close
of the accounting year ended on 31st March, 2017. The Financial Statements for the year
ended 31/03/2017 had been prepared on the same basis as it had been in earlier periods with
an additional note that the business of the Company shall cease in near future and the assets
shall be disposed off in accordance with a plan of disposal as decided by the Management.
The Statutory Auditors of the Company indicated this aspect in Key Audit Matters only by a
reference as to a possible cessation of business and making of adjustments, if any, thereto to
be made at the time of cessation only. Comment on the reporting by the Statutory Auditor as
above.

Answer:
 Closure of Business: As per SA 570 “Going Concern”, management intentions to liquidate

the entity or to cease operations is one of the event or condition that may cast significant
doubt on the entity’s ability to continue as going concern.
 As per SA 570, if events or conditions have been identified that may cast significant

doubt on the entity’s ability to continue as a going concern but, based on the audit
evidence obtained the auditor concludes that no material uncertainty exists, the auditor
shall evaluate whether, in view of the requirements of the applicable financial reporting
framework, the financial statements provide adequate disclosures about these events or
conditions.
 Even when no material uncertainty exists, it requires the auditor to evaluate whether,

in view of the requirements of the applicable financial reporting framework, the financial
statements provide adequate disclosure about events or conditions that may cast
significant doubt on the entity’s ability to continue as a going concern.
 Further, as per SA 701 “Communicating Key Audit Matters in the Independent Auditor’s

Report”, when matters relating to going concern may be determined to be key audit
matters, and explains that a material uncertainty related to events or conditions that
may cast significant doubt on the entity’s ability to continue as a going concern is, by
its nature, a key audit matter. SA 701 also emphasises on auditor’s responsibility to
communicate key audit matters in the auditor’s report.
 As per the facts given in the case, intention of the Toddle Limited had definite plan of its

business being closed down within short period from 31 March, 2017. However, financial
statements for the year ended 31.03.2017 had been prepared on the same basis as it
had been in earlier periods with an additional note.
 Thus, management intentions to liquidate the entity or to cease operations is one of the

258
FINAL C.A. – AUDIT

event or condition that may cast significant doubt on the entity’s ability to continue as
going concern is a key audit matter. Therefore, the auditor is required to Communicate
the Key Audit Matters in accordance with SA 570 in above stated manner. Simple
reference as to a possible cessation of business and making of adjustments, if any, be
made at the time of cessation only by the auditor in his report is not sufficient.

Question 6:
M/s Airlift Ltd., carrying on the business of Passenger Transportation by air is running
into continuous financial losses as well as reduction in Sales due to stiff competition and
frequent break down of its own aircrafts. The Financial Statements for the Year ended on
31/03/2018 are to be now finalized. The Management is quite uncertain as to its ability to
continue in near future and has informed the Auditors that having seized of this matter, it
had constituted a committee to study this aspect and to give suggestions for recovery, if any,
from this bad situation. Till the study is completed, according to the Management, the issue
involves uncertainty as to its ability to continue its business and it informs the Auditor that
the fact of uncertainty clamping on the “Going Concern” would suitably be disclosed in notes
to accounts. State the reporting requirement if any, in the Independent Auditor’s Report in
respect of this matter

Answer:.
 As per SA 570 “Going Concern”, if the auditor concludes that management’s use of the

going concern basis of accounting is appropriate in the circumstances but a material


uncertainty exists, the auditor shall determine whether the financial Statements
i. adequately disclose the principal events or conditions that may cast significant
doubt on the entity’s ability to continue as a going concern and management’s plans
to deal with these events or conditions and
ii. disclose clearly that there is a material uncertainty related to events or conditions
that may cast significant doubt on the entity’s ability to continue as a going
concern and, therefore, that it may be unable to realize its assets and discharge
its liabilities in the normal course of business

 If adequate disclosure about the material uncertainty is made in the financial statements,
the auditor shall express an unmodified opinion and the auditor’s report shall include a
separate section under the heading “Material Uncertainty Related to Going Concern” to:
i. Draw attention to the note in the financial statements that discloses the matters
set out above; and

259
FINAL C.A. – AUDIT

ii. State that these events or conditions indicate that a material uncertainty exists
that may cast significant doubt on the entity’s ability to continue as a going concern
and that the auditor’s opinion is not modified in respect of the matter

 In the instant case, M/s Aircraft Ltd. is running into continuous financial losses as
well as reduction in sales due to stiff competition and frequent break down of its own
aircrafts and management of Aircraft Ltd. is uncertain as of its ability to continue in
near future. Therefore, a committee has been constituted to study this aspect and till
the time study is completed management accordingly decided to suitable disclose this
aspect in notes to accounts

 Therefore, the auditor should disclose about the material uncertainty and express an
unmodified opinion and in his audit report shall include a separate section under the
heading “Material Uncertainty Related to Going Concern” to draw attention to the note
in the financial statements that discloses the matters set out above;

 and state that these events or conditions indicate that a material uncertainty exists
that may cast significant doubt on the entity’s ability to continue as a going concern and
that the auditor’s opinion is not modified in respect of the matter

260
FINAL C.A. – AUDIT

SA 580
WRITTEN
REPRESENTATIONS

Q.1 . MEANING :
 It is a written statement by management provided to the auditor

 To confirm certain matters or

 To support other audit evidence.

 WR do not include FS, the assertions therein, or supporting books & records

Q.2. FORM AND CONTENT :


 It is a written statement by management provided to the auditor to confirm certain

matters or to support other audit evidence


 It does not include financial statements, the assertions therein, or supporting

 books and records

 The auditor shall request management to provide a written representation that

it has fulfilled its responsibility for the preparation of the FS in accordance with
theAFRF, including where relevant their fair presentation, as set out in the terms
of the audit engagement
 The written representations shall be for all financial statements and period(s)

referred to in the auditor’s report


 IT must be obtained at the end of the audit but before signing out report
 It must be in a proper form as a representation letter (not electronic form)

 Contents of WR are

 As required by SA 580
AS required by other SAs
AS required by the auditor

Q.3 IF THE MANAGEMENT REFUSES TO PROVIDE WR

(DAR)
D- Discuss with management

261
FINAL C.A. – AUDIT

A- Take Appropriate actions including determining the possible effect on the opinion in
auditors report.

R- Re-evaluate the integrity of management & evaluate the effect that this may have
on the reliability of representation (oral or written) on audit evidence in general.

The auditor should disclaim an opinion on the financial statement if the management
does not provide WR in accordance with SA 705.

Q.4. DOUBTS OVER RELIABILITY OF WR :

 If the auditor has concerns about the competence, integrity, ethical values or
diligence of management

 The auditor shall determine the effect that such concerns may have on the reliability
of representations. (oral or written)

 If WR are inconsistent with other AE, the auditor shall perform audit procedures
to attempt to resolve the matter.

 If auditor concludes that WR are not reliable, the auditor shall take appropriate
actions, including determining the possible effect on the opinion in auditors report
in accordance with SA 705

Q.5 RELIABILITY OF WR
FAQ: The auditor of ABC ltd. was not able to get the confirmation about the existence
& value of certain machines. However, the management gave him a certificate to prove
the existence & value of m/c as appearing in the books of accounts. the auditor accepted
the same without any further procedure & signed the audit report is he right in his
approach.

Answer:
1 As per SA 500 ‘Audit evidence” the auditor shall obtain sufficient and appropriate evidence.

2. Sufficiency refer to the quantum & appropriateness refers to the quality of the evidence.

262
FINAL C.A. – AUDIT

3. Although WR provide necessary AE, they do not provide SAAE on their own about any of
the matters with which deal.

4. It can be considered just as on additional evidence. Thus, other evidences w.r.t specific
item shall be obtained by the auditor.

5. As written evidence cannot be considered as a substitute for other audit evidences, the
auditor shall also seek corroborative evidence from other sources whether internal or
external to entity.

6. In the present case study, it is given that the auditor of ABC LTd. Was not able to get
the confirmation about the existence & value of the machinery as appearing in books of
accounts.

The auditor accepted the same without any further procedure and signed the audit
report.

7. Machinery being a material item, he should have obtained sufficient & appropriate
evidences and should not have Merely, relied upon written representation alone.

8. In case, he is not able to obtain other corroborative evidences, he should either qualify
or disclaim his audit opinion.

9. Thus, he is not right in his approach as he violated SA 500 & SA 580 as explained.

263
FINAL C.A. – AUDIT

SA 600
USING THE WORK OF
ANOTHER AUDITOR

I. REQUIREMENTS

Acceptance as Principal Auditor


The auditor should consider whether the auditor’s own participation is sufficient to be able to
act as the principal auditor. For this purpose the auditor would consider:

1. The principal auditor’s degree of knowledge regarding the business of the components

2. The materiality of the portion of the financial information which the principal auditor
audits

3. The ROMMS in the financial information of the components audited by the other auditor

4. The performance of additional procedures as set out in this SA regarding the components
audited by other auditor resulting in the principal auditor having significant participation
in such audit

II. THE PRINCIPAL AUDITOR’S PROCEDURES


1. In certain situations, the statute governing the entity may confer a right on the principal
auditor to visit a component and examine the books of account and other records of the
said component, if he thinks it necessary to do so.

2. Where another auditor has been appointed for the component, the principal auditor
would normally be entitled to rely upon the work of such auditor unless there are special
circumstances to make it essential for him to visit the component and/or to examine the
books of account and other records of the said component

3. When planning to use the work of another auditor, the principal auditor should consider
the professional competence of the other auditor in the context of specific assignment if
the other auditor is not a member of the Institute of Chartered Accountants of India

264
FINAL C.A. – AUDIT

III. CO-ORDINATION WITH OTHER AUDITOR:

The principal auditor should perform procedures to obtain SAAE, that the work of the
other auditor is adequate for the principal auditor’s purposes, in the context of the specific
assignment. When using the work of another auditor, the principal auditor should ordinarily
perform the following procedures:

1. Advise the other auditor of the use that is to be made of the other auditor’s work and
report and make sufficient arrangements for coordination of their efforts at the planning
stage of the audit.

2. Inform the other auditor of matters such as areas requiring special consideration,
procedures for the identification of inter-component transactions that may require
disclosure and the time-table for completion of audit

3. Advise the other auditor of the significant accounting, auditing and reporting requirements
and obtain representation as to compliance with them.

4. Discuss with the other auditor the audit procedures applied or review a written summary
of the other auditor’s procedures and findings which may be in the form of a completed
questionnaire or check-list

5. Consider the significant findings of the other auditor.

6. Discuss with the other auditor and the management of the component, the audit findings
or other matters affecting the financial information of the components.

IV. REPORTING CONSIDERATIONS:


1. When the principal auditor concludes, based on his procedures, that the work of the
other auditor cannot be used

• and the principal auditor has not been able to perform sufficient additional
procedures regarding the financial information of the component audited by the
other auditor,
• the principal auditor should express a qualified opinion or disclaimer of opinion
because there is a limitation on the scope of audit.

265
FINAL C.A. – AUDIT

2. In all circumstances, if the other auditor issues, or intends to issue, a modified auditor’s
report,

• the principal auditor should consider whether the subject of the modification is of
such nature and significance, in relation to the financial information of the entity
on which the principal auditor is reporting that it requires a modification of the
principal auditor’s report.

3. When the principal auditor has to base his opinion on the financial information of the
entity as a whole relying upon the statements and reports of the other auditors,

s his report should state clearly the division of responsibility for the financial
information of the entity by indicating the extent to which the financial information
of components audited by the other auditors have been included in the financial
information of the entity,

s e.g., the number of divisions/branches/subsidiaries or other components audited by


other auditors.

NOTE
s Principal auditor can visit the component and review the financial information, however,
he cannot establish his right over the working papers of other auditor because working
papers are property of auditor.

QUESTIONS ASKED IN PAST/RTP/MOCK TEST

Question 1:
There should be sufficient liaison between a principal auditor and
Answer:
The event will have no impact , subject to satisfaction in respect of non violation of going
concern concept. Hence management is correct.

Question 2:
As an auditor, how do u deal with the following: The statutory auditor of the Holding Company
demands for the working papers of the auditors of the subsidiary company, of which you are
the auditor.

266
FINAL C.A. – AUDIT

Answer:
Access to working papers:

s As per SA 230, “Audit Documentation” working papers are the property of the auditor.
The auditor may, at his discretion, make portion of or extracts of his working papers
available to his client.

s SA 600 “Using the Work of Another Auditor” also states that an auditor should respect
the confidentiality of the information acquired during the course of his audit work and
should not disclose such information unless there is a legal or professional duty to disclose.

s As per ICAI Guidelines, statutory auditor of an enterprise does not have right of access
to the audit working papers of the branch auditor. An auditor can rely on the work
of another auditor, without having any right of access to the audit working papers of
another auditor.

s Conclusion: Statutory Auditor of the Holding Company cannot have access to audit
working papers of the Subsidiary companies auditor. He can however, ask the auditor to
answer certain questions about the manner in which the audit is conducted and certain
other clarifications regarding audit.

Question 2:
Describe the relevance of SA 600 while auditing the consolidation of Financial Statements.

Answer:
s SA 600 “Using the work of Another Auditor” guides principal auditor regarding the
procedures to be performed when he is using the work of another auditor. In case
of audit of Consolidated financial statements, principal auditor will use the work of
auditor of subsidiary company, associates and joint venture. As per SA 600, auditor of
Consolidated Financial Statements is supposed to perform the following:

1. When principal auditor plans to use the work of another auditor he should consider the
professional competence of the other auditor in the context of specific assignment if the
other auditor is not a member of the ICAI.

267
FINAL C.A. – AUDIT

2. The principal auditor should perform procedures to obtain sufficient appropriate audit
evidence that the work of the other auditor is adequate for the principal auditor’s purposes
in the context of the specific assignment

3. The principal auditor should consider the significant findings of the other auditor.

4. The principal auditor should document in his audit working papers the followings:

s Components audited by another


s Audit procedures adopted and results thereof ;
s Conclusion that particular component is not material;
s Manner of dealing with modification in another auditor’s report

5. When The principal auditor concludes based on his procedures that the work of other
auditor the cannot be used and the principal auditor has not been able to perform
sufficient additional procedures regarding the financial information of the component
audited by the other auditor the principal auditor should express a qualified opinion or
disclaimer of opinion because, there is a limitation on the scope of audit.

6. When the principal auditor has to base his opinion on the financial information of the
entity as a whole relying upon the statements and the reports of the other auditor his
report should state clearly the division of responsibility for the financial information of
the entity by indicating the extent to which the financial information of components
audited by the other auditors have been included in the financial Information of the
entity.

Question 3:
s B Ltd. Is the subsidiary company of A Ltd. ABC & Associates has been appointed as
the auditor of A Ltd. for the financial year 2019-2020 and XYZ & Associates has been
appointed as the auditor of B Ltd. For the year 2019-2020. Explain the role of ABC
& Associates and XYZ & Associates as auditors of the parent company and subsidiary
respectively.
Answer:

268
FINAL C.A. – AUDIT

SA 610
USING THE WORK OF
INTERNAL AUDITORS

INTRODUCTION

s Internal audit function constitutes a separate component of internal controls & objective

is to determine whether internal controls are well designed & properly operated.

s SA 610 provides a procedure to be applied by their external auditor in assessing works of



internal auditors for the purpose of placing reliance upon that work.

s Though work on internal auditor can be useful to statutory auditor, statutory auditor

will alone be responsible for his report & determination of nature, timing & extent of
auditing procedures.

I. SA 610 USING THE WORK OF INTERNAL AUDITORS

A. Using the work B. Using the work of


of Internal auditor Internal auditor for
without direct direct assistance
assistance

i. Determining areas ii. Using for direct


& extent of use assistance

Direct Assistance

The use of internal auditor to perform audit procedure under direction, supervision and
review of external auditor.

269
FINAL C.A. – AUDIT

A. USING THE WORK OF INTERNAL AUDITOR WITHOUT DIRECT ASSISTANCE.

1. The external auditor shall consider the nature and scope of the work that has been
performed, or is planned to be performed, by the internal audit function and its relevance
to the external auditor’s overall audit strategy and audit plan

2. The external auditor shall perform sufficient audit procedures on the work of the internal
audit of the function as a whole that the external auditor plans to use to determine its
adequacy for the purposes of the audit, including evaluating whether.

s The work of the function had been properly planned, performed, supervised, reviewed
and documented,

s SAAE had been obtained to enable the function to draw reasonable conclusion, and

Conclusions reached are appropriate in the circumstances and the reports prepared
s
by the function are consistent with the results of the work performed.

3. The nature and extent of the external auditor’s audit procedures shall be responsive to
the external auditor’s evaluation of: (JOLA)

a) J- The amount of judgment involved;

b) O- objectivity of the internal auditors;

c) L- The level of competence of the function

d) A- The assessed ROMMS;

4. The external auditor shall discuss the planned use of its work with the function as a
basis for coordinating their respective activities.

5. The external auditor shall read the reports of the internal audit function relating to the
work of the function that the external auditor plans to use

270
FINAL C.A. – AUDIT

B. DIRECT ASSISTANCE:

(i) Determining Whether, in Which Areas, and to What Extent Internal Auditors Can Be
Used to Provide Direct Assistance:

1. The external auditor may be prohibited by law or regulation from obtaining direct
assistance from internal auditors.(for eg if you are appointed by C&AG)

2. If using internal auditors to provide direct assistance is not prohibited by law or


regulation, and the external auditor plans to use internal auditors to provide direct
assistance on the audit, the external auditor shall evaluate the
s existence and significance of threats to objectivity
s the level of competence of the internal auditors who will be providing such assistance.

3. In determining the nature and extent of work that may be assigned to internal auditors
and the NTE of direction, supervision and review that is appropriate in the circumstances,
the external auditor shall consider: (JOLA)
a) J- The amount of judgment involved;
b) O- objectivity of the internal auditors;
c) L- The level of competence of the function
d) A- The assessed ROMMS;

NOTE:
The external auditor shall not use internal auditors to provide direct assistance to perform
procedures that:
(a) Involve making significant judgments in the audit

(b) Relate to higher assessed ROMMS where the judgment required in performing the
relevant audit procedures or evaluating the audit evidence gathered is more than limited

(c) Relate to work with which the internal auditors have been involved and which has
already been, or will be, reported to management or TCWG by the internal audit function

(d) Relate to decisions the external auditor makes in accordance with this SA regarding the
internal audit function and the use of its work or direct assistance

271
FINAL C.A. – AUDIT

B. USING INTERNAL AUDITORS TO PROVIDE DIRECT ASSISTANCE

1. Prior to using internal auditors to provide direct assistance for purposes of the audit, the
external auditor shall:

(a) Obtain written agreement from an authorized representative of the entity that the
internal auditors will be allowed to follow the external auditor’s instructions, and that
the entity will not intervene in the work the internal auditor performs for the external
auditor; and

(b) Obtain written agreement from the internal auditors that they will keep confidential
specific matters as instructed by the external auditor and inform the external auditor of
any threat to their objectivity

2. The external auditor shall direct, supervise and review the work performed by internal
auditors on the engagement in accordance with SA 220. In so doing:

(a) The NTE of direction, supervision, and review shall recognize that the internal auditors
are not independent of the entity; and

(b) The review procedures shall include the external auditor checking back to the underlying
audit evidence for some of the work performed by the internal auditors.

(c) The direction, supervision and review by the external auditor of the work performed by the
internal auditors shall be sufficient in order for the external auditor to be satisfied that
the internal auditors have obtained SAAE to support the conclusions based on that work.

3. If the external auditor uses internal auditors to provide direct assistance on the audit,
the external auditor shall include in the audit documentation:

(a) The evaluation of the existence and significance of


s of threats to the objectivity of the internal auditors, and
s the level of competence of the internal auditors used to provide direct assistance;

(b) The basis for the decision regarding the nature and extent of the work performed by the
internal auditors;

272
FINAL C.A. – AUDIT

(c) Who reviewed the work performed and the date and extent of that review in accordance
with SA 230;

(d) The written agreements obtained from


s an authorized representative of the entity
s and the internal auditors; and

(e) The working papers prepared by the internal auditors who provided direct assistance on
the audit engagement

QUESTIONS ASKED IN PAST/RTP/MOCK TEST

Question 1:
Rajpanth Ltd. Appointed you as it’s statutory auditor for the current financial year. During
the course of auditing, you meticulously analyzed that the work performed by the internal
auditor is likely to be adequate for the purpose of statutory audit. Consequently, you decide
to use the work of the internal auditor in respect of physical verification of tangible assets
specifically. State how would you evaluate the work performed by the internal auditor to
determine its adequacy and who would be responsible for expression of opinion on financial
matters.
Answer:

Question 1:
Mr. A was appointed as statutory auditor of X Ltd. X Ltd. has an internal audit system and
Mr. A is of the opinion that internal auditors can be used to provide direct assistance for the
purpose of statutory audit. Advise Mr. A whether he can take direct assistance of internal
auditor and if yes, what are the precautions he needs to take.
Answer:

Question 1:
OPQ Ltd. is in the business of software consultancy. The company has had a large balance of
accounts receivables in the past years which have been assessed as areas of high risk. For the
year ended 31st March 2020, in respect of the valuation of account receivable, the statutory
auditor has assigned the checking of the accuracy of the aging of the accounts receivable and
the provision based on ageing to the internal auditor providing direct assistance to him. Please
advise.

273
FINAL C.A. – AUDIT

SA 620
USING THE WORK OF AN
AUDITOR’S EXPERT

INTRODUCTION

s This SA does not deal with:


(a) Situations where the engagement team includes a member with expertise in specialized
area of accounting or auditing, which is dealt with in SA 220; or

(b) The auditor’s use of the work of an individual or organization possessing expertise in a
field other than accounting or auditing, whose work in that field is used by the entity to
assist the entity (i.e. management’s expert) in preparing the FS(a management’s expert),
which is dealt with in SA 500.

II. OBJECTIVE

s To determine whether to use the work of an auditor’s expert and

s If using the work of an auditor’s expert, to determine whether that work is adequate for
the auditor’s purposes

III. DEFINITION

1. Auditor’s expert

s An individual or organization possessing expertise in a field other than accounting or


auditing,
s whose work in that field is used by the auditor to assist the auditor in obtaining
SAAE.
s An auditor’s expert may be either an auditor’s internal expert (who is a partner
or staff, including temporary staff, of the auditor’s firm or a network firm), or an
auditor’s external expert.

274
FINAL C.A. – AUDIT

2. Management’s expert :
s An individual or organization possessing expertise in a field other than accounting or
auditing,
s Whose work in that field is used by the entity to assist the entity in preparing the
financial statements.

IV. TYPES OF REPORT (AV VOIT)

A- The actuarial calculation of liabilities associated with insurance contracts or employee


benefit plans.

V- The valuation of environmental liabilities, and site clean-up costs.

V- Valuation of complex financial instruments, land and buildings, plant & machinery ,
jewelry, etc

O- The estimation of oil and gas reserves.

If expertise in a field other than accounting or auditing is necessary to obtain


s
sufficient appropriate audit evidence, the auditor shall determine whether to use the
work of an auditor’s expert. Expertise in a field other than accounting or auditing
may include expertise in relation to such matters as:

The valuation of complex financial instruments, land and buildings, plant and
s
machinery, jewelry, works of art, antiques, intangible assets, assets acquired and
liabilities assumed in business combinations and assets that may have been impaired

I- The interpretation of contracts, laws and regulations.

T- The analysis of complex or unusual tax compliance issues.

V. THE NTE OF THE AUDITOR’S PROCEDURES WILL VARY DEPENDING ON THE


CIRCUMSTANCES. IN DETERMINING THE NTE OF THOSE PROCEDURES, THE
AUDITOR SHALL CONSIDER MATTERS INCLUDING:

1. The nature of the matter to which that expert’s work relates;

275
FINAL C.A. – AUDIT

2. The ROMMS in the matter to which that expert’s work relates;

3. The significance of that expert’s work in the context of the audit;

4. The auditor’s knowledge of and experience with previous work performed by that expert;
and

5. Whether that expert is subject to the auditor’s firm’s quality control policies and
procedures

6. whether the auditor’s expert has the necessary competence, capabilities and objectivity
for the auditor’s purposes

VII. THE AUDITOR SHALL AGREE, IN WRITING WHEN APPROPRIATE, ON THE


FOLLOWING MATTERS WITH THE AUDITOR’S EXPERT:

1. The nature, scope and objectives of that expert’s work;

2. The respective roles and responsibilities of the auditor and that expert;

3. The nature, timing and extent of communication between the auditor and that expert,
including the form of any report to be provided by that expert; and

4. The need for the auditor’s expert to observe confidentiality requirements.

VIII. THE AUDITOR SHALL EVALUATE THE ADEQUACY OF THE AUDITOR’S EXPERT’S
WORK FOR THE AUDITOR’S PURPOSES, INCLUDING:

1. The relevance and reasonableness of that expert’s findings or conclusions, and their
consistency with other audit evidence;

2. If that expert’s work involves use of significant assumptions and methods, the relevance
and reasonableness of those assumptions and methods in the circumstances

3. If that expert’s work involves the use of source data that is significant to that expert’s
work, the relevance, completeness, and accuracy of that source data.

276
FINAL C.A. – AUDIT

IX. REFERENCE TO THE AUDITOR’S EXPERT IN THE AUDITOR’S REPORT:

Required by statute Not Required by statute

For eg Insurance act may If Auditor issued


require valuation by actuary modified opinion

Must provide reference Matter giving rise to modification


relates to experts work
*The auditor shall indicate in the
auditor’s report that such reference If expert gives consent
does not reduce the auditor’s
responsibility for that opinion Auditor may provide
reference*

Question 1:
While doing audit, Ram the auditor requires reports from experts for the purpose of audit
evidence. What types of reports/opinions he can obtain and to what extent he can rely upon
the same.?
Answer:
IV. Types of report

Question 2:
X ltd had a net worth of INR 1300 crores because of which Ind As became applicable to them.
The company had various derivative contracts, interest rate swaps etc. which were required
to be fair valued for which company got the fair valuation done through an external third
party. The statutory auditor of the company involved an auditor’s expert to audit valuation of
derivatives. Auditor and auditor’s expert were new to each other, i.e., they were working for
the first time together but developed a good bonding during the course of audit. The auditor
did not enter into any formal agreement with the auditors expert. Please advise.
Answer:
VII.

Question 3:
Mr. Mohan, an auditor of KTEN Ltd. wants to use the work of an expert. With reference

277
FINAL C.A. – AUDIT

to the Standard on Auditing state of factors which suggest the need for details and written
agreement between the auditor and the auditor’s expert.
Answer:
VII

Question 4:
State what may be the evaluative or review procedures that the Statutory Auditor may do
before concluding as to relevance and reasonableness of Auditors Expert work for using it for
his audit purposes.
Answer:
VIII

Question 5:
O Ltd. Is in the business of manufacturing of steel. The manufacturing process requires
raw material as iron ore for which large stock was maintained by the company at year end
31 March 2020. the nature of raw material is such that its physical verification requires
involvement of an expert. Management hired their expert for stock take and auditors . The
auditor observed that the work of the auditors expert was not adequate for auditors purposes
and the auditor could not resolve the matter through additional audit procedures which
included further work performed by both the auditor’s expert and the auditor.
Basis above, the auditor concluded that it would be necessary to express a modified opinion in
the auditors report because the auditor has not obtained sufficient appropriate audit evidence.
However, the auditor issued a clean report and included the name of the expert in his report
to reduce his responsibility for the audit opinion. Comment.
Answer:
IX reporting responsibility

278
FINAL C.A. – AUDIT

SA 700
FORMING AN OPINION &
REPORTING ON FS

Q.1 OBJECTIVES:

 Form an opinion on FS based on an evaluation of the conclusions drawn from the audit
evidence obtained and

 Express clearly the opinion through a written report that also describes the basics for
the opinion

Q.2 AFRF:

General Purpose framework Specific Purpose framework

Common financial info needs of all Specific info needs of the intended
users users

Fair presentation Compliance


framework framework

Fair presentation framework Compliance Framework

Refer to a financial reporting Refer to a financial reporting


framework that requires framework that requires
compliance with the requirements compliance with the requirements
of the framework and of the framework

Provide disclosures Depart from


beyond those requirement of the
specifically required framework to achieve
by framework fair presentation

279
FINAL C.A. – AUDIT

Q.3 BASIC ELEMENTS OF AUDIT REPORT

 Title
 An addressee, as required by the circumstances of the engagement.
 Auditor’s Opinion: The Opinion section of the auditor’s report shall also:
 Identify the entity whose financial statements have been audited;

 State that the financial statements have been audited;

 Identify the title of each statement comprising the financial statements;

 Refer to the notes, including the summary of significant accounting policies; and

 Specify the date of, or period covered by, each financial statement comprising the

financial statements

 Basis for Opinion:



 The auditor’s report shall include a section, directly following the Opinion section,

with the heading “Basis for Opinion”, that:


u States that the audit was conducted in accordance with Standards on Auditing;

u Refers to the section of the auditor’s report that describes the auditor’s responsibilities

under the SAs;


u Includes a statement that the auditor is independent of the entity in accordance with

the relevant ethical requirements relating to the audit and has fulfilled the auditor’s
other ethical responsibilities in accordance with these requirements.
u States whether the auditor believes that the audit evidence the auditor has obtained is

sufficient and appropriate to provide a basis for the auditor’s opinion.

 Going Concern:
w Where applicable, the auditor shall report in accordance with SA 570 (Revised)

 Key Audit Matters:


w For audits of complete sets of general purpose financial statements of listed

entities, the auditor shall communicate key audit matters in the auditor’s report in
accordance with SA 701

 Other Information
w Where applicable, the auditor shall report in accordance with SA 720

Management Responsibilities for the Financial Statements

280
FINAL C.A. – AUDIT

 Auditor’s Responsibilities for the Audit of the Financial Statements:


 This section of the auditor’s report shall:
a. State that the objectives of the auditor are to:
i. Obtain reasonable assurance about whether the financial statements as a whole
are free from material misstatement, whether due to fraud or error; and
ii. Issue an auditor’s report that includes the auditor’s opinion.

b. State that reasonable assurance is a high level of assurance, but is not a guarantee that
an audit conducted in accordance with SAs will always detect a material misstatement
when it exists; and

c. State that misstatements can arise from fraud or error, and either:
i. Describe that they are considered material if, individually or in the aggregate, they
could reasonably be expected to influence the economic decisions of users taken on
the basis of these financial statements; or
ii. Provide a definition or description of materiality in accordance with the AFRF.

 Other Information
w Where applicable, the auditor shall report in accordance with SA 720

 Signature of the auditor:

 Date of auditors report:

 Place of auditors report

Q.4 INTERNATIONAL STANDARDS ON AUDITING

 An auditor may be required to conduct an audit in accordance with, in addition to the


Standards on Auditing issued by ICAI, the International Standards on Auditing or
auditing standards of any other jurisdiction.

 If this is the case, the auditors report may refer to Standards on Auditing in addition to
the International Standards on Auditing or auditing standards of such other jurisdiction,
but the auditor shall do so only if:

281
FINAL C.A. – AUDIT

a) There is no conflict between the requirements in the ISAs or such auditing standards of
other jurisdiction and those in SAs that would lead the auditor
i. to form a different opinion, or
ii. not to include an Emphasis of Matter paragraph or Other Matter paragraph that,
in the particular circumstances, is required by SAs; and

b) The auditor’s report includes, at a minimum, each of the elements above when the auditor
uses the layout or wording specified by the Standards on Auditing

282
FINAL C.A. – AUDIT

SA 701
COMMUNICATING KEY
AUDIT MATTERS

APPLICABILITY:

 Listed entities

 Voluntary if auditor otherwise decides to communicate key audit matter

Q.1 MEANING
 Those matters that, in the auditor’s professional judgment, were of most significance in

the audit of the financial statements of the current period.

 Key audit matters are selected from matters communicated with those charged with
governance

Q.2 COMMUNICATING KEY AUDIT MATTERS IN THE AUDITOR’S REPORT IS NOT:


 A substitute for disclosures in the financial statements that the AFRF requires management

to make, or that are otherwise necessary to achieve fair presentation

 A substitute for the auditor expressing a modified opinion when required by the circumstances
of a specific audit engagement in accordance with SA 705 (Revised)

 A substitute for reporting in accordance with SA 570 (Revised) when a material


uncertainty exists relating to events or conditions that may cast significant doubt on an
entity’s ability to continue as a going concern

 A separate opinion on individual matters

Q.3 PURPOSE OF COMMUNICATING KEY MATTER PARA (BAJE)


 B – To provide a basis to further engage with the management and TCWG about certain

matters relating to the entity, the audited FS or the audit that was performed

283
FINAL C.A. – AUDIT

 A – To provide additional information to the intended users of the financial statements to


assist them in understanding those matters that, in the auditor’s professional judgement,
were of most significance in the audit of the FS of the current period

 J – To assist intended users in understanding the entity and areas of significant


management judgement in the audited FS

 E – To enhance the communicative value of the Auditors report by providing greater


transparency about the audit that was performed.

Q.4 FACTORS TO BE CONSIDERED FOR DETERMINING KEY AUDIT MATTERS:

 Areas of higher assessed risk of material misstatement, or significant risks identified in


accordance with SA 315 (For examples Refer SA 315)

 Significant auditor judgments relating to areas in the financial statements that involved
significant management judgment, including accounting estimates that have been
identified

 The effect on the audit of significant events or transactions that occurred during the
period.

 The auditor shall determine which of the matters so determined above were of most
significance in the audit of FS of the current period and therefore are the key audit
matters.

Q.5 COMMUNICATING KEY MATTER PARA
 The auditor shall describe each key audit matter, using an appropriate subheading, in a

separate section of the auditor’s report under the heading “Key Audit Matters”.

 The introductory language in this section of the auditor’s report shall state that:
(a) Key audit matters are those matters that, in the auditor’s professional judgment,
were of most significance in the audit of the financial statements [of the current
period]; and

284
FINAL C.A. – AUDIT

(b) These matters were addressed in the context of the audit of the financial statements
as a whole, and in forming the auditor’s opinion thereon, and the auditor does not
provide a separate opinion on these matters.

 The description of each key audit matter in the Key Audit Matters Section of the auditors
report shall include a reference to the related disclosure(s), if any, in the FS and shall
address:
(a) Why the matter was considered to be one of the most significance in the audit and
therefore determined to be a key audit matter ? And

(b) How the matter was address in the audit?

 The auditor shall describe each key audit matter in the auditor’s report unless:
(a) Law of Regulation precludes public disclosure about the matter, or

(b) In extremely rare circumstances, the auditor determines that the matter should not
be communicated in the auditor’s report because the adverse consequences of doing
so would reasonable be expected to outweigh the public interest benefits of such
communication.

PARTICULAR UNQUALIFIED QUALIFIED ADVERSE DICLAIMER


OPINION OPINION OPINION OPINION
Opinion Para 1st We have audited We have audited We have audited We were engaged to
line audit
2nd Line Which comprises the balance sheet at March 31st20XX,and the P&L A/c, (and
statement of Cash flows) for the year ended, and notes to the FS, including a summary
of significant accounting policies.
3rd Line In our opinion and to the best of our information and according to the explanations
given to us,
4th Line Gives true and fair Except for the Because of the Because of the
view of the financial effects/possible significance of the significance of the
position effects of the matters described matters described
matter described in the basis of in the basis of
in the basis for adverse opinion disclaimer of
Qualified opinion para the FS are opinion para the
para the FS are not true & fair auditor doesnot
true & fair express an opinion
on FS

285
FINAL C.A. – AUDIT

PARTICULAR UNQUALIFIED QUALIFIED ADVERSE DICLAIMER


OPINION OPINION OPINION OPINION
Basis of opinion We conducted our audit in accordance with the SAs issued by ICAI. Our responsibilities
para under those standards are further described in the Auditor’s Responsibilities for the
Audit of the Financial Statements section of our report. We are independent of the
entity in accordance with the ethical requirements that are relevant to our audit
of the financial statements in [jurisdiction], and we have fulfilled our other ethical
responsibilities in accordance with these requirements. We believe that the audit
evidence we have obtained is sufficient and appropriate to provide a basis for our
opinion.
Basis of opinion We believe we that We believe we We believe we Because of
para the audit evidence that the audit that the audit the matters
Last line we have obtained evidence we evidence we described in
is sufficient and have obtained have obtained the basis of
appropriate to is sufficient and is sufficient and disclaimer of
provide a basis for appropriate to appropriate to opinion para, we
our opinion provide a basis provide a basis were not able to
for our qualified for adverse obtain SAAE to
opinion opinion provide a basis
for our opinion
Mgt Management is responsible for the preparation and fair presentation of the financial
responsibility statements in accordance with the aforesaid Accounting Standards, and for such
para internal control as management determines is necessary to enable the preparation of
financial statements that are free from material misstatement, whether due to fraud
or error.
In preparing the financial statements, management is responsible for assessing the
entity’s ability to continue as going concern, disclosing, as applicable, matters related to
going concern and using the going concern basis of accounting unless management either
intends to liquidate the entity or to cease operations, or has no realistic alternative
but to do so.
Mgt Those charged with governance are responsible for overseeing the entity’s financial
responsibility reporting process.
para
Auditor Our objectives are to obtain reasonable assurance about whether the financial
responsibility statements as a whole are free from material misstatement, whether due to fraud or
para error, and to issue an auditor’s report that includes our opiníon.
Reasonable assurance is a high level of assurance, but is not a guarantee that an audit
conducted in accordance with SAs will always detect a material misstatement when it
exists.
Auditor Misstatements can arise from fraud or error and are considered material if, individually
responsibility or in the aggregate, they could reasonably be expected to influence the economic decisions
para of users taken on the basis of these financial statements.

286
FINAL C.A. – AUDIT

QUESTIONS ASKED IN PAST/RTP/MOCK TEST

Question 1:
Under the applicable Standards on Auditing, in what circumstances does the report of the
statutory auditor require modifications? What are the types of modifications possible to the
said report?
Answer:

Question 2:
What are the features of a qualified Audit Report
Answer:
 Clarity: The Auditor must express the nature of qualification, in a clear and unambiguous

manner

 Explanation: Where the Auditor answers any of the statutory affirmations in the
negative or with a qualification, his report shall state the reasons for such answer.

 Placement: All qualifications should be contained in the Auditor’s Report. When there are
notes which are subject matter of a qualification, the same should preferably be annexed
to the Auditors’ Report. However, a reference to the notes to Accounts in the Auditors’
Report does not automatically become a qualification

 Except for: A quantified opinion should be expressed as “except for” for the effects of the
matter to which qualification related. It would not be appropriate to use phrases such
as “with the foregoing explanation” or “subject to” in the opinion paragraph as these are
not sufficiently clear or forceful…..

 Quantification: It is also necessary that the auditor should quantify, wherever possible,
the effect of individual as well as the total effect of all qualifications on statement of
profit and loss and/or state of affairs these qualifications on the financial statements in
a clear and unambiguous manner. In circumstances where it is not possible to quantify
the effect of the qualifications accurately the auditor may do so on the estimates made by
the management after carrying out such audit tests as are possible and clearly indicate
that the figures given are based on the estimates of the management….

 Nature of qualification: Vague statements the effect of which on accounts cannot be

287
FINAL C.A. – AUDIT

ascertained like ‘the trade receivables balances are subject to confirmation’, ‘no provision
for taxation has been made in view of the loss during the year’ etc., should be avoided.

 Violation of law: Where the company has committed an irregularity resulting in a breach
of law, the Auditor should bring the same to the notice of the shareholders by properly
qualifying his report….

 Notes – Report Relationship – Where notes of a qualificatory nature appear in the


accounts, the Auditors should state all qualifications independently in their report so
that the user can assess the significance of these qualifications.

Question 3:
Write a short note on Certificate for Special Purpose vs. Audit Report
Answer:
 A certificate is a written confirmation of the accuracy of the facts stated therein and

does not involve any estimate or opinion.

 The term ‘certificate’ is, therefore, used where the auditor verifies the accuracy of facts.

 An auditor may thus, certify the circulation figures of a newspaper or the value of
imports or exports of a company.

 An auditor’s certificate represents that he has verified certain figures and is in a position
to vouch safe their accuracy as per his examination of documents and books of account.

 A report, on the other hand, is a formal statement usually made after an enquiry,
examination or review of specified matters under report and includes the reporting
auditor’s opinion thereon.

 Thus, when a reporting auditor issues a certificate, he is responsible for the factual
accuracy of what is stated therein.

 On the other hand, when a reporting auditor gives a report, he is responsible for ensuring
that the report is based on factual data, that his opinion is in due accordance with
facts, and that it is arrived at by the application of due care and skill .

288
FINAL C.A. – AUDIT

 The ‘report’ involves expression of opinion which may differ from one professional to
another

 There is no question of exactitude in case of a report since the information contained


therein is based on estimates and involves judgement element

Question 4:
Give examples of Emphasis of Matters which may have an adverse effect on the functioning of
the company as well as those which may not affect the functioning of the company.

Answer:
GAAL

Question 5:
CA. Amar has come across certain key matters while auditing the accounts of PR Ltd.
for the financial year 2017-18. He, being the associate of your firm, seeks your advice on
“Communicating Key Audit Matters” in the Auditor’s Report. Guide him.
Answer:

Question 6:
There are certain circumstances in which Emphasis of Matter in Auditor’s Report is mandated
to be included. Explain this statement in the light of mandatory requirements of matters that
are to be emphasised in Auditor’s Report when the Audit Report is on Financial Statements
prepared in accordance with Special Purpose Framework.
Answer:
 As per SA 706, “Emphasis of Matter Paragraphs and Other Matter Paragraphs in
the Independent Auditor’s Report” and/or SA 800, “Special Considerations— Audits of
Financial Statements Prepared in Accordance with Special Purpose Frameworks”, the
auditor’s report on special purpose financial statements shall include an Emphasis of
Matter paragraph alerting users of the auditor’s report that the financial statements
are prepared in accordance with a special purpose frame work and that,

 as a result, the financial statements may not be suitable for another purpose. The
auditor shall include this paragraph under an appropriate heading.

 The special purpose financial statements may be used for purposes other than those for

289
FINAL C.A. – AUDIT

which they were intended. For example, a regulator may require certain entities to place
the special purpose financial statements on public record.

 To avoid misunderstandings, the auditor alerts users of the auditor’s report that the
financial statements are prepared in accordance with a special purpose frame work and,
therefore, may not be suitable for another purpose

 Restriction on Distribution or Use: In addition to the alert required above, the auditor
may consider it appropriate to indicate that he auditor’s report is intended solely for the
specific users.

 Depending on the law or regulation of the particular jurisdiction, this may be achieved
by restricting the distribution or use of the auditor’s report. In these circumstances,
the emphasis of matter paragraph given above maybe expanded to include these other
matters, and the heading may be modified accordingly.

Question 7:
Write a short note on: Purpose of communicating key audit matters
Answer:

Question 8:
Mr. A was appointed as statutory auditor of X Ltd. While doing audit, Mr. A is required to
determine the key audit matters which are required to be mentioned in the audit report. You
are required to advise Mr. A about the considerations which Mr. A shall take into account
while determining key audit matters.
OR
Question 8:
“The auditor shall determine, from the matters communicated with those charged with
governance, those matters that required significant auditor attention in performing the audit
In making this determination, the auditor shall take into account the key factors”. You are
required to define key audit matters and briefly discuss the factors determining the key audit
matters.
OR
QUESTION 8:
As an auditor of a listed company for the year ended 31st March, 2020, how would you
determine the ‘Key Audit Matters’?

290
FINAL C.A. – AUDIT

QUESTION 9:
C.A. Amar has come across certain key matters while auditing the accounts of PR Ltd.
for the financial year 2019-20. He, being the associate of your firm, seeks your advice on
“Communicating KeyAudit Matters” in the Auditor’s report. Guide him.
ANSWER:

QUESTION 10:
State the circumstances in which a matter determined to be a key audit matter is not
required to be communicated in the Auditor’s Report.
ANSWER:

QUESTION 11:
The property, plant and equipment of ABC Ltd. included Rs.25.75crores of earth removing
machines of outdated technology which had been retired from active use and had been kept
for disposal after knock down. These assets appeared at residual value and had been last
Inspected ten years back. As an auditor, what may be your reporting concern as regards
matters specified above?
ANSWER:
 Reporting Concerns in relation to significant events:
 Auditor is required to report under the various requirements of Standards of Auditing
legal and Regulatory provisions. In the present situation, major reporting requirements
will be:
(a) As per the requirement of SA 260 “Communication with Those Charged with
Governance” auditor should communicate significant matters arising during the
audit that were discussed, or subject to correspondence, with management.

(b) The situation as given in the question appears to be a Key Audit Matter and hence
auditor is required to report the situation in the audit report as Key Audit Matter.

(c) Further as per requirement of Para 3(i) of CARO, 2016, auditor is required to
comment
(i) Whether the company is maintaining proper records showing full particulars,
including quantitative details and situation of fixed assets
(ii) whether the fixed assets have been physically verified by the management at
reasonable intervals. In the present case, physical verification of assets held
under disposal.

291
FINAL C.A. – AUDIT

 Conclusion: In the present case, auditor reporting concerns will be as per the requirement
of SA 260, SA 701 and Para 3(1) of CARO, 2016.

QUESTION 11:
When should an auditor make a disclaimer opinion in his Audit Report?
ANSWER:

QUESTION 12:
ADKS & Co LLP are the newly appointed statutory auditors of PKK Ltd. During the course
of audit, the statutory auditors have come across certain significant observations which
they believe could lead to material misstatement of Financial statements. Management has a
different view and does not concur with the view of the statutory auditors. Considering this
the statutory auditors are determining as to how to address these observations in terms of
their reporting requirement. Please advise.
 Circumstances in which a modified opinion may be issued:
 As per SA 705“Modifications to the Opinion in the Independent Auditor’s Report” a
modified opinion may be expressed in the following circumstances:
(a) The auditor concludes that, based on the audit evidence obtained, the ES. as a
whole are not free from material misstatement, may be due to following reasons:
1. Inappropriate method of selection of Accounting Policies;
2. Accounting policies are not consistent with applicable FRF;
3. Disclosures as required by FRF are not given.

(b) The auditor is unable to obtain sufficient appropriate audit evidence to conclude
that the financial statements as a whole are free from material misstatement, may
be due to following reasons:
1. Limitations imposed by management
2. Circumstances beyond entity control (For Ex.: Accounting records destroyed
by fire)
3. Circumstances related to Nature and Timing of auditor’s work.
Types of modified opinion:
1. Qualified opinion 2. Adverse opinion 3. Disclaimer of opinion

QUESTION 13:
After accepting the statutory audit of M/s All in One Ltd., a departmental store, you became
aware of the fact that management of the company have imposed certain limitations on

292
FINAL C.A. – AUDIT

the scope of your assurance function which may adversely affect and result in your inability
to obtain sufficient appropriate audit evidence to discharge your responsibility required by
the statute. Indicate the consequences and your response to the limitations imposed by the
management on your scope.

QUESTION 14:
Compare and Explain the following: Audit Qualification and Emphasis of Matter.
ANSWER:
SA 705 “Modifications to the Opinion in the Independent Auditor’s Report”, deals with the
provisions relating to Audit Qualification. Audit Qualifications are given when auditor is having
reservations on some of the items out of the financial statements.
It is issued under following circumstances:
1. Financial statements are materially misstated which in the auditor’s judgments are not
pervasive.
2. Auditor is unable to obtain Sufficient and appropriate audit evidence which in the auditor
judgment are not pervasive

 As per 706 “Emphasis of Matter Paragraphs and Other Matter Paragraphs in the
Independent Auditor’s Report” Emphasis of Matter is a paragraph which is included
in auditor’s report to draw users’ attention to important matter(s) which are already
disclosed in Financial Statements and are fundamental to users’ for understanding of
Financial Statements.

 EOM is used when there is an uncertainty relating to future outcome of exceptional


litigation, regulatory action, etc; or there is early application (where permitted) of a new
accounting standard that has a pervasive effect on the financial statements in advance
of its effective date.

QUESTION 14:
D Ltd., a Delhi based company having turnover of 25 crores, has a branch at USA having a
turnover of ? 10 lakhs (as converted from US dollars). The area where the branch office is
located in USA was severely affected by storms and the office along with all accounting records
was completely destroyed. Due to the unavailability of records, year 2019-20 did not include
the figures pertaining to the said branch. As the statutory auditor of financial statements of
D Ltd. for the financial D Ltd., how will you report on the same?

293
FINAL C.A. – AUDIT

ANSWER:
 Reporting on financial statements when information of component is not included:

 As per SA 200 “Overall Objectives of the Independent Auditor and Conduct of Audit in

accordance with Standards on Auditing” in conducting an audit of financial statements,


the overall objective of the auditor is to obtain reasonable assurance about whether
the Financial statements as a whole are free from material misstatement, whether
due to fraud or error, thereby enabling the auditor to express an opinion on whether
the financial statements are prepared, in all material respects, in accordance with an
applicable FRE. In all cases when reasonable assurance cannot be obtained and a
qualified opinion in the auditor’s report is insufficient, the SAs require that the auditor
disclaim an opinion or withdraw from the engagement.
 In the present case, D Ltd., a Delhi based company having turnover of 25 crores, has a

branch at USA having a turnover of 10 lakhs (as converted from US dollars). The area
where the branch office is located in USA was severely affected by storms and the office
along with all accounting records was completely destroyed. Due to the unavailability
of records, the financial statements of D Ltd. for the financial year 2019-20 did not
include the figures pertaining to the said branch.
 In the present situation, company is required to make appropriate disclosures in the notes

to accounts in this regard. Based on the disclosures made in the financial statements,
auditor is required to include an Emphasis of Matter Para in the auditor’s report as per
requirement of SA 706, If, however no disclosure is made in the financial statements,
auditor need to qualify the audit report as turnover of the branch is only Rs. 10 lakhs
which does not seems to have pervasive effect as the total turnover of the company is Rs.
25 Crores.
 Conclusion: If appropriate disclosures are given in Notes to Accounts, an unmodified

opinion with emphasis of Matter para need to be issued. However, if appropriate disclosures
are not given in notes to Accounts, auditor should qualify the report in accordance with
SA 705.

QUESTION 14:
Enumerate certain important matters which can be included in “Emphasis of Matter
Paragraph” in an Auditor’s Report.
ANSWER:

294
FINAL C.A. – AUDIT

SA 705
MODIFICATIONS TO THE OPINION
IN THE INDEPENDENT AUDITORS
REPORT

Q.1 OBJECTIVES:

Unmodified Modified

Modified without Modified by


modifying opinion modifying opinion

Emphasis on Others matter


matters Para Para
(EOM) (OM)

Modified by
modifying opinion

Auditor has SAAE Auditor is unable to obtain SAAE


that misstatement exist in FS that FS are free from misstatement

Matter is material Matter is material matter is material


and pervasive but not pervasive and pervasive

ADVERSE QUALIFIED DISCLAIMER


OPINION OPINION OPINION

295
FINAL C.A. – AUDIT

SA 706
EOM PARA & OM PARA IN
INDEPENDENT AR

 Q.1 OBJECTIVE:
The objective of the auditor, having formed an opinion on FS is to draw user’s attention,
when in the auditor’s judgement it is necessary to do so, by way of clear additional
communication in the auditor’s report, to:

 A matter, although appropriately presented or disclosed in FS, is that is of such


importance that it is fundamental to users understanding of the FS.

 As appropriate, any other matter that is relevant to users understanding of the audit,
the auditor’s responsibilities or the auditor’s report.

Q.2 EMPHASIS OF MATTER PARA IN THE AUDITOR’S REPORT

 Include it immediately after the opinion Para in auditor’s report.

 Use the heading ‘Emphasis of Matter’ or other appropriate heading

 Include in the Para a clear reference to matter being emphasized

 Indicate that the auditor’s opinion is not modified in respect of the emphasized matter

Examples: (GAAL)

 G-There is substantial doubt about the entity’s ability to continue as a going concern.

 A-Early application of new accounting standard that has a pervasive effect on FS in


advance of is effective date.

 A-There has been a material, change between periods in accounting principles or in


method of their applications.

296
FINAL C.A. – AUDIT

 L-An uncertainty relating to the future outcomes of an exceptional litigation or regulatory


action.

Q.3 OTHER MATTER PARA IN AUDITORS REPORT:

 If the auditor considers it necessary to communicate a matter other than those that
are presented or disclosed in FS that,

 in auditors judgement, is relevant to users understanding of the audit, the auditors


responsibilities or auditor’s report &

 this is not prohibited by law or regulation, the auditor shall do so in a Para in the
auditor’s report , with the heading ‘ others matter’ or other appropriate heading.

 The auditor shall include this Para immediately after the opinion Para & any EOM Para.

Examples: (RALI)

 R-Reasons why auditor is unable to resign from the engagement even though there is
limitation of scope.

 A-Another set of FS has been prepared by the same entity in accordance with another
general purpose framework & that the auditor had issued a report on those FS.

 L-Law, regulation or generally accepted practise that may require or permit the auditor
to elaborate on matters that provide further explanation.

 I-Auditors report is intended solely for intended users.

297
FINAL C.A. – AUDIT

SA 710
COMPARATIVE INFO-
CORRESPONDING FIGURES

Q.1 MEANING:

 Comparative information
 Where amounts & other disclosures for the preceding period
 Are included
 As part of current period financial statements
 And are intended to be read in relation to the amounts & other disclosures relating to
the current period.
 These corresponding figures are not presented as complete financial statements capable
of standing alone, but are integral part of the current period financial statement intended
to be ready only in relationship to current period figures.

II. AUDITORS RESPONSIBILITIES:

i. The auditor shall determine whether FS include the comparative info required by the
AFRF & whether such info is appropriately classified. For this purpose the auditor shall
evaluate whether:
a) The comparative info agrees with the amounts & other disclosures presented in the
prior period; and

b) The accounting policies reflected in the comparative info are consistent with those
applied in the current period or if there have been changes in accounting policies,
whether those changes have been properly accounted for & adequately presented &
disclosed.

ii. If the auditor becomes aware of a possible material misstatement in the comparative
info while performing the current period Audit
t The auditor shall perform such additional audit procedures as are necessary in

the circumstances to obtain SAAE to determine whether a material misstatement


exists.

298
FINAL C.A. – AUDIT

iii. If the auditor had audited the prior periods FS, the auditor shall also follow the relevant
requirements of SA 560.

iv. As required by SA 580, the auditor shall request WR for all periods referred to in the
auditor’s opinion.
The auditor shall also obtain a specific WR regarding any prior period item that is
separately disclosed in cy statement of P/L.

III. PREVIOUS REPORT CONTAINS A QUALIFICATION:

 When the auditors report on prior  When the auditor report on prior
period as previously issued. period as previously issued.
- included a modified opinion - included a modified opinion.
- and the matter giving rise to - and matter giving rise to
modification is resolved & properly modification is unresolved.
accounted for & disclosed in FS  The auditor shall modify the

according to AFRF. auditor’s opinion on current period


 The auditors opinion of the FS.
current period need not refer to
modifications.

IV. REPORTING:
1. When comparative financial statements are presented, the auditor’s opinion shall refer
to each period for which financial statements are presented and on which an audit
opinion is expressed:

2: If the auditor obtains AE that a MMS exists in the prior period financial statements
on which an unmodified opinion has been previously issued:
 the auditor shall verify whether the misstatement has been dealt with as required

under the AFRF and,


 if that is not the case, the auditor shall express a qualified opinion or an adverse

opinion in the auditor’s report on the current period financial statements

3. If the financial statements of the prior period were audited by a predecessor auditor and

299
FINAL C.A. – AUDIT

 the auditor is permitted by law or regulation to refer to the predecessor auditor’s


report on the corresponding figures and decides to do so, the auditor shall state in
an Other Matter paragraph in the auditor’s report:
a) That the financial statements of the prior period were audited by the
predecessor auditor;
b) The type of opinion expressed by the predecessor auditor and, if the opinion
was modified, the reasons therefore; and
c) The date of that report.

4. If the prior period financial statements were not audited:


 the auditor shall state in an Other Matter paragraph in the auditor’s report that

the corresponding figures are unaudited.

 Such a statement does not, however, relieve the auditor of the requirement to obtain
SAAE that the opening balances do not contain misstatements that materially
affect the current period’s financial statements

Question 1:
Auditors responsibilities in respect of corresponding figures?
Answer:

Question 2:
The audit report of P ltd for the year 2018-19 contained a qualification regarding non-
provision of doubtful debts. As the statutory provision of the company for the year 2019-20,
how would you report it, if
a) The company does not make provision for doubtful debts in 2019-20?
b) The company makes adequate provision for doubtful debts in 2019-20

Question 3:
It was observed from the modified audit report of the financial statements of AS ltd for the
year ended 31st March, 2019 that depreciation of Rs. 2.50 crore for the year 2018-19 had
been charged off to statement of P&L instead of including it in “Carrying Value of asset under
construction”. State in relation to the audit report for the year ended 31st March 2020,
whether such modification in the previous years audit report would have any audit implication
for the CY and is yes, how would you deal with it in your audit report?

300
FINAL C.A. – AUDIT

SA 720
THE AUDITOR RESPONSIBILITIES
RELATING TO OTHER INFORMATION

1. OBJECTIVES

To consider whether there is a material inconsistency between the other information and

the financial statements;


the auditor’s knowledge obtained in the audit

 To respond appropriately when the auditor identifies that such material inconsistencies
appear to exist, or when the auditor otherwise becomes aware that other information
appears to be materially misstated

 To report in accordance with this SA

2. IMPORTANT CONCEPTS
Other Information:
Financial or non-financial information (other than financial statements and the
auditor’s report thereon) included in an entity’s annual report.
Misstatement:
A misstatement of the other information exists when the other information is incorrectly
stated or otherwise misleading (including because it omits or obscures information
necessary for a proper understanding of a matter disclosed in the other information)

3. Audit Procedures
Auditor shall determine, through discussion with mgt, which document(s) comprises the
AR, & the entity’s planned manner & timing of the issuance of such document(s)

Auditor shall make appropriate arrangements with management to obtain in a timely
manner and, if possible, prior to the date of the auditor’s report, the final version of the
document(s) comprising the annual report

When some or all of the document(s) determined in (a) will not be available until after

301
FINAL C.A. – AUDIT

the date of the auditor’s report, request management to provide a written representation
that the final version of the document(s) will be provided to the auditor when available,
and prior to its issuance by the entity, such that the auditor can complete the procedures
required by this SA

To consider whether there is a material inconsistency between the other information and
the financial statements;
the auditor’s knowledge obtained in the audit

If the auditor identifies that a material inconsistency appears to exist (or becomes
aware that the other information appears to be materially misstated), the auditor shall
discuss the matter with management and, if necessary, perform other procedures to
conclude whether:
a.) A material misstatement of the other information exists;
b.) A material misstatement of the financial statements exists; or
c.) The auditor understands of the entity and its environment needs to be updated

If the auditor concludes that a material misstatement of the other information exists,
the auditor shall request management to correct the other information.
If management:
a.) Agrees to make the correction, the auditor shall determine that the correction has
been made
b.) Refuses to make the correction, the auditor shall communicate the matter with
TCWG and request that the correction be made.

If the auditor concludes that a material misstatement exists in other information


obtained prior to the date of the auditor’s report, and the other information is not
corrected after communicating with TCWG, the auditor shall take appropriate action,
including:
a.) Considering the implications for the auditor’s report and communicating with TCWG
about how the auditor plans to address the material misstatement in the auditor’s
report or
b.) Withdrawing from the engagement, where withdrawal is possible under applicable
law or regulation

If the auditor concludes that a material misstatement exists in other information

302
FINAL C.A. – AUDIT

obtained after the date of the auditor’s report, the auditor shall:
a) If the other information is corrected, perform the procedures necessary in the
circumstances; or
b) If the other information is not corrected after communicating with TCWG take
appropriate action considering the auditor’s legal rights and obligations, to seek to
have the uncorrected material misstatement appropriately brought to the attention
of users for whom the auditor’s report is prepared

The auditor’s report shall include a separate section with a heading “Other Information”,
or other appropriate heading, when, at the date of the auditor’s report:
a) For an audit of financial statements of a listed entity, the auditor has obtained, or
expects to obtain, the other information; or
b) For an audit of financial statements of an unlisted corporate entity, the auditor
has obtained some or all of the other information

When the auditor’s report is required to include an Other Information section, this
section shall include:
a) A statement that management is responsible for the other information;

b) An identification of:
i) Other information, if any, obtained by the auditor prior to the date of the
auditor’s report; and
ii) For an audit of financial statements of a listed entity, other information, if
any, expected to be obtained after the date of the auditor’s report;
c) A statement that the auditor’s opinion does not cover the other information and,
accordingly, that the auditor does not express (or will not express) an audit opinion
or any form of assurance conclusion thereon;
d) A description of the auditor’s responsibilities relating to reading, considering and
reporting on other information as required by this SA; and
e) When other information has been obtained prior to the date of the auditor’s report,
either:
i) A statement that the auditor has nothing to report; or
ii) If the auditor has concluded that there is an uncorrected material misstatement
of the other information, a statement that describes the uncorrected material
misstatement of the other information

303
FINAL C.A. – AUDIT

LIABILITIES OF AUDITOR

I - PROFESSIONAL NEGLIGENCE
A) Meaning –
It connotes any failure to perform a duty according to accepted professional standards,
which has resulted in some loss, damage or detriment to the party who had engaged a
professional.

(B) Elements of Professional Negligence :


(i) Existence of duty or responsibility : One party is owned to another party to perform some
act with due care, skill and competency.
(ii) Occurrence of Breach : A breach occurs while performing the duty.
(iii) Loss or damages: Loss or detriment being served by the party to whom the duty was owed
as a result of breach. To establish a liability in respect of third party it is necessary that
such third party should have suffered a loss or damages on account of the professional
negligence.

II - CIVIL AND CRIMINAL LIABILITIES UNDER COMPANIES ACT, 2013


(A) Criminal liability for mis-statements in prospectus-Sec.34 -
l Where any prospectus is issued or circulated or distributed

l which includes any statement which is untrue or misleading in form or context in which

it is included or
l where any inclusion or omission of any matter is likely to mislead,

l then every person who authorises the issue of such prospectus shall be liable u/s 447

(fraud).

Exception :
This shall not apply to a person if he proves that:
(1) Such statement or omission was immaterial, or

(2) He had reasonable grounds to believe, and did up to the time of issue of the prospectus
believe, that the statements was true or the inclusion or omission was necessary.

304
FINAL C.A. – AUDIT

(B) Criminal liability for making false statement – Sec, 448 :


If in return, report, certificate, financial statement, prospectus, statement or other
document required by, or for, the purpose of any of the provisions of this Act or the rules
made thereunder, any person makes a statement –
(i) Which is false in any material particulars, knowing it to be false or
(ii) Which omits any material fact, knowing it to be material he shall be liable under section
447.

(C) Punishment for fraud u/s 447 :
Without prejudice to any liability including repayment of any debt under this Act or any
other law for the time being in force, any person who is found to be guilty of fraud,
involving an amount of at least Rs10 lakh or 1% of the turnover of the company,
whichever is lower

l Imprisonment: Shall be punishable with imprisonment for a term which shall not be less
than 6 months but which may extend to 10 years.
Provided that where the fraud in question involves public interest, the term of imprisonment
shall not less than 3 years.

l Fine: Shall also be liable to fine which shall not be less than the amount involved in the
fraud, but which may extend to 3 times the amount involved in the fraud

(D) Civil liability for mis-statement in prospectus- Sec 35 :


l Where a person has subscribed for securities of a company
l acting on any statement included, or the inclusion or omission of any matter
l in the prospectus
l which is misleading and has sustained any loss or damage as a consequence thereof,
l the company and every person who-

(a) Is a director of the company at the time of the issue of the prospectus;
(b) Has authorised himself to be named and is named in the prospectus as a director the
company or has agreed to become such director, either immediately or after an interval
of time;
(c) Is a promoter of the company;
(d) Has authorised the issue of the prospectus; and
(e) Is an expert,

305
FINAL C.A. – AUDIT

shall be liable to pay compensation to every person who has sustained such loss or
damage.
Exception :
No person shall be liable, if he proves that-
(a) Having consented to become a director of the company he withdrew his consent before
the issue of the prospectus, and that it was issued without his authority or consent; or
(b) The prospectus was issued without his knowledge or consent, and that on becoming aware
of its issue, he forthwith gave a reasonable public notice that it was issued without his
knowledge or consent.
(c) As regards every misleading statement purported to be made by an expert, it was a
correct and fair representation of the statement; and he had reasonable ground to
believe and did up to the time of the issue of the prospectus believe, that the person
making the statement was competent to make it and that the said person had given
the required consent to the issue of the prospectus and had not withdrawn that
consent before delivery of a copy of the prospectus for registration or, to the defendant’s
knowledge, before allotment there under.

III - LIABILITIES UNDER INCOME TAX ACT

(A) Section 288 (Disability as to represent the assessee)


Sec. 288(4) – A person who has been convicted of any offence connected with any
Income Tax proceeding or on who a penalty has been imposed under the said Act [except
Section 271 (1) (ii)] is disqualified from representing an assessees.

The CCIT / CIT has been given powers to determine the period of such disqualification
of a person.
(B) Section 278 :
Any person who abets or induces, in any manner another person to make and deliver to
the Income Tax Authorities a false account, statement, or declaration relating to any
income chargeable to tax which he knows to be false or does not believe to be true is
punishable.
(i) In a case where the amount of tax, penalty or interest which would have been evaded,
if the declaration, account or statement had been accepted a true, or which is wilfully
attempted to be evaded, exceeds `25 Lacs, with rigorous imprisonment for a term which
shall not be less than 6 months but which may extend to 7 years and with fine;
(ii) In any other case, with rigorous imprisonment for a term which shall not be less than
306
FINAL C.A. – AUDIT

3 months but which may extend to 2 years and with fine.


(C) Rule 12 A :
Every authorised representative of an assessee, who has prepared the return of income
furnished by the asssessee shall either before making an appearance before the Assessing
Office having jurisdiction to assess that assessee or immediately after making such
appearance, furnish to that officer -
(i) Particulars of accounts, statements or other documents supplied to him by the
assessee for the preparation of the return of income; and
(ii) Where the authorised representative has for the purpose of preparation of the
return of income carried out any examination of such accounts, statement or
documents, a report on the scope and results of such examination.
(D) Section 271J – Penalty for furnished incorrect information :
Where the Assessing Office or the Commissioner (Appeals), in the course of any proceedings
under this Act, finds that an accountant or a merchant banker or a registered valuer
has furnished incorrect information in any report or certificate furnished under
any provision of this Act or the rules made there under the Assessing Office or the
Commissioner (Appeals) may direct that person to pay a penalty of `10,000 for each
such report or certificate.

IMPORTANT QUESTIONS
Q.2. Explain the liability of the auditor under the Companies Act, 29013, for making an
untrue statement in the report (As an expert forming a part of the prospectus)
Or
Indicate the precise nature of auditor’s liability for misstatement that had occurred in
the prospectus issued by the company.

Q.3. Mr. X, a young chartered accountant, wants to start practice and he required your
advice, among other things, on criminal liabilities of an auditor under the Companies Act,
2013. Kindly guide him.
Or
Mr. Arjun, a newly qualified Chartered Accountant started his practice wants to
specialize in Audits of corporate and required your advice on criminal liabilities of an
auditor under the Companies Act, 2013. Kindly guide him.

Q.4. A chartered Accountant in practice has been appointed as an auditor of a company


which raised finance from the capital market on the basis of a prospectus issued a few
307
FINAL C.A. – AUDIT

years back. The main object for raising the finance was specified to be setting up a
project on information technology. The company advanced the sum so raised to various
firms and private companies in whom the directors of the company were a partner
or a director respectively. These parties had no standing whatsover with information
technology.
In the Balance Sheet, there advance appeared a current asset under the head “Short-
terms Loans and Advances-Unsecured, considered good”. There was no mention to the
notes to accounts about nature and purpose of such advances; and the auditor has issued
routine audit report without any qualifications. On the very next day to the issuance of
audit report, the directors and their related parties gone disappeared. The company, in
which the auditor was conducting audit, has just vanished. You are required to state
whether the auditor will be held guilty for professional misconduct? Is there any liability
subsists under any law?

Q.5. Indicate the precise nature of auditor’s liability in the following situation: certain
weaknesses in the internal control procedure in the payment of wages in a large
construction company were notice by the statutory auditor who in turn brought the
same to the knowledge of the Managing Director of the company. In the subsequent
year huge defalcation came to the notice of the management. The origin of the same was
traced to the earlier year. The management wants to sue the auditor for negligence and
also plans to file a complaint with the Institute.

Q.6. Indicate the precise nature of auditor’s duties in the following situation : Based upon
the legal opinion of a leadings advocate, X Ltd. Made a provision of ` 5 crores towards
Income Tax liability. The assessing authority has worked out the liability at ` 15 crores.
It is observed that the opinion of the advocate was inconsistent with legal position with
regard to certain revenue items.

Q.7. State the nature of liability as provided in the Companies Act, 2013 of an auditor for
not appropriately dealing with a misstatement appearing in audited financial statements
or false statement in Audit Report.

Q.8. In assessment procedure of M/s cloud Ltd., Income Tax office observed some irregularities.
Therefore he stated investigation of Books of Account audited and signed by Mr. Old, a
practicing Chartered Accountant. While going through books he found that M/s cloud
Ltd. used to maintain two sets of Books of Account one is the official set and other is

308
FINAL C.A. – AUDIT

covering all the transaction.


Income Tax Department filed a complaint with the Institute of Chartered Accountants
of India saying Mr. Old had negligently performed his duties. Comment.

Ans. Liability of Auditor:


l It is the auditors responsibility to audit the statement of accounts and prepare tax
returns on the basis of books of account produced before him. After being satisfied with
the books and documents produced to him, he can give his opinion on the basis of those
documents only by exercising requisite skill and care.

l In the present case, Income tax officer observed some irregularities during the assessment
proceeding of M/s Cloud Ltd. Therefore he stated investigation of books of account
audited and signed by Mr. Old, a practicing Chartered Accountant. While going through
the books, he found that M/s cloud Ltd. used to maintain two sets of Books of Account,
one is the official set and other is covering all the transactions. Income Tax Department
filed a complaint with the ICAI saying Mr. Old had negligently performed his duties.

l Mr. Old the auditor was not under a duty to prepare books of account of assessee and he
should, of course, neither suggest noir assist in the preparations of false accounts. He is
responsibility for the books produced before him for audit. He completed his audit work
with official set of books only.

Ans. Conclusion:
As Mr. Old, performed the auditing with due skill and diligence; and, therefore, no question
of negligence arises. It is the duty of the Department to himself investigate the truth
and correctness of the accounts of the assessee.

Q.9. Write a short note on – Auditor’s liability in case of unlawful acts or defaults by clients.

Q.10.Mr. Ram, a chartered Accountant has appeared before the Income Tax Authorities as
the authorized representative of his client and delivers to the income tax authorities a
false declaration. What are the liabilities of Mr. Ram under Income Tax Act, 1961?

Q.11.What are the liabilities of a Chartered Accountant under Income Tax Act, 1961 for
furnishing an incorrect statement in any report or certificate required to be submitted
by him under the Act?

309
FINAL C.A. – AUDIT

RISK ASSESSMENT AND


INTERNAL CONTROL

I - AUDIT PLANNING
(A) In the audit planning process of X Ltd. you would like to consider audit risk at the
financial state meat level What are the factors can influence your decision?
Or
Write Short note on: Evaluation of Inherent Risk at the Level of Financial Statements.
Or
Explain the concept of Audit Risk at the Level of Financial Statements.

Ans. Factors to be evaluated to assess inherent risk at the level of financial statements:
1. Integrity of management.
2. Management’s experience and knowledge and changes in management during the period.
3. Unusual pressures on management.
4. The nature of the entity’s business.
5. Factors affecting the industry in which the entity operates.

(B) As the auditor of a large multi locational company, in the planning process, you are
requested to identify the inherent audit risk at the account balance and class of
transaction level.
Or
Describe how you would identify the inherent risk at the account balance and class of
transaction level in the planning process of the audit of a large multi-locational company.

Ans. Identifying inherent risk at the account balance and class of transaction level:
To evaluate the inherent risk at the account balance and transaction level, auditor
should evaluate the following factors:
1. Quality of the accounting system.
2. Susceptibility of Financial statements to misstatement.
3. The complexity of underlying transactions and other events which might require using
the work of an expert.
Identifying inherent risk at the account balance and class of transaction level:

310
FINAL C.A. – AUDIT

4. The degree of judgment involved in determining account balances.


5. Susceptibility of assets to loss or misappropriation.
6. The completion of unusual and complex transactions, particularly at or near period end.
7. Transactions not subjected to ordinary processing.

(C) While commencing the statutory audit of B Company Limited, the auditor undertook
the risk assessment and found that the detection risk relating to certain class of
transactions cannot be reduced to acceptance level. Explain.

Ans. Assessment of Risk and Acceptable Level:


 SA 315 “Identifying and Assessing the Risk of Material Misstatement Through
Understanding the Entity and its Environment” and SA 330 “The Auditor’s Responses
to Assessed Risks” establishes standards on the procedures to be followed to obtain an
understanding of the accounting and internal control systems and on audit risk and its
components.

 SA 315 and SA 330 require that the auditor should use professional judgment to assess
risk of material misstatement and to design audit procedures to ensure that it is reduced
to an acceptably low level.

 Risk of Material Misstatements comprises of Inherent risk and Control Risk. “Detection
risk” is the risk that an auditor’s substantive procedures will not detect a misstatement
that exists in an account balance or class of transactions that could be material.

 The higher the risk of material misstatement, the more audit evidence the auditor should
obtain from the performance of substantive procedures. When both inherent and control
risks are assessed as high, the auditor needs to consider whether substantive procedures
can provide sufficient appropriate audit evidence to reduce detection risk, and therefore
audit risk, to an acceptably low level.

 The auditor should use his professional judgment to assess audit risk and to design audit
procedures to ensure that it is reduced to an acceptably low level. If it cannot be reduced
to an acceptable level, the auditor should express a qualified opinion or a disclaimer of
opinion as may be appropriate.

(D) Compute the overall Audit Risk if looking to the nature of business there are chances

311
FINAL C.A. – AUDIT

that 40% bills of services provided would be defalcated, inquiring on the same matter
management has assured that internal control can prevent such defalcation to 75%.
At his part the Auditor assesses that the procedure he could apply in the remaining
time to complete Audit gives him satisfaction level of detection of frauds & error to an
extent of 60%. Analyse the Risk of Material Misstatement and find out the overall Audit
Risk.

Determination of Audit Risk:


 As per SA-200, “Overall Objectives of the Independent Auditor and the Conduct of an
Audit in Accordance with Standards on Auditing”, the Audit Risk is a risk that Auditor
will issue an inappropriate opinion while Financial Statements are materially misstated.

 Audit Risk is a function of two components: Risk of material Misstatement and Detection
Risk, i.e. Audit Risk = Risk of material Misstatement x Detection Risk

 Risk of Material Misstatement is anticipated risk that a material Misstatement may


exist in Financial Statement before start of the Audit. It has two components Inherent
risk and Control risk. The relationship can be defined as Risk of material Misstatement
= Inherent risk x control risk.

 Inherent risk is the susceptibility of an assertion about account balance; class of


transaction, disclosure towards misstatements which may be either individually or
collectively with other Misstatement becomes material before considering any related
internal control. Inherent Risk in the given case is 40%.

Control Risk is the risk that material misstatement will not be prevented or detected and
corrected on a timely basis by the internal control system. Control risk in the given case
is 25% (100%-75%).

 Risk of material Misstatement = 40% x 25 % = 10%

 Detection risk is the risk that the substantive procedures performed by the auditor
fails to detect material misstatement. Detection Risk In the given case IS I00 - 60 =
40%

 Overall Audit Risk = Risk Material Misstatement Detection Risk = 10 x 40% = 4%


312
FINAL C.A. – AUDIT

II- RISK BASED AUDIT


(A) What are the General Steps in the conduct of Risk Based Audit? [May 16 (4 marks)]
Or
What are the main phases in the conduct of Risk Based Audit?
Or
ST Ltd is a growing company currently engaged in the business of manufacturing of
tiles. The company is planning to expand and diversify its operations. The management
has increased the focused on the internal control to ensure better governance. The
management had a discussion with the statutory auditors to ensure the steps required to
be taken so that the statutory auditors to ensure the steps required to be taken so that
the statutory audit is risk based and focused on areas of greatest to the achievement
of the company’s objectives. Please advise the management and the auditor on the steps
that should be taken for the same.

Ans. Steps to be followed while conducting Risk Based Audit:
Risk Based audit is an audit approach that analyzes audit risks, sets materiality
thresholds based on audit risk analysis, and develops audit programmes that allocate a
larger portion of audit resources to high-risk areas.
Stages in Risk Based Audit:
1. Understanding the auditee operations: Auditor should understand the auditee operations
in order to identify and prioritize the risks that impact audit of financial statements.
Auditor should obtain understanding of the following:
 Environment in which entity operates.
 Framework of operations;
 Operational performances;
 Information process framework etc.

2. Determination of Residual Risk: Auditor should assess entity management strategies and
controls so as to determine how the controls are designed within the entity.

3. Manage Residual Risk: It requires design and execution of a risk reduction approach
so as to bring the residual audit risk to an acceptable level. More resources should be
allocated to areas of high audit risks.

4. Reporting to Auditee: The auditor should communicate to the auditee immediately his
significant observation w.r.t. following:
313
FINAL C.A. – AUDIT

 Weaknesses in the internal control system,


 Deficiencies in the design and operation of internal control that affact the

organization’s ability to record, process, summarize the report financial data.

III- INTERNAL CONTROL


(A) Pasta Ltd., a manufacturing concern want to develop internal control system. You are an
expert in developing the internal control system, hereby called to brief about the same.
In view of above, you are required to brief about internal control system and inherent
limitations of the internal control?
Ans. Internal Control System and its Inherent Limitations:
SA 315 “Identifying and Assessing the Risk of Material Misstatement Through
Understanding the Entity and its Environment” defines internal control as the process
designed, implemented and maintained by TCWG, management and other personnel to
provide reasonable assurance about the achievement of an entity’s objectives with regard
to
(a) Reliability of financial reporting;
(b) Effectiveness and efficiency of operations;
(c) Safeguarding of assets; and
(d) Compliance with applicable laws and regulations.
Inherent Limitations of Internal Control System:
(a) Management’s consideration that a control should be cost-effective.
(b) The fact that the most controls do not tend to be directed at transactions of
unusual nature.
(c) Potential for human error.
(d) Possibility of circumvention of controls through collusion with parties outside the
entity or with employees of entity.
(e) Possibility that a person responsible for exercising control could abuse that
authority.
(f) Possibility that procedures may become inadequate due to changes in conditions
and compliance with procedures may deteriorate.
(g) Manipulations by management with respect to transactions or estimates and
judgments required in the preparation of financial statements.

(B) XYZ Hospital Private Ltd. is engaged In running a hospital of 200 Beds since last 20
years. Revenue Track of the hospital for last 3 years is as under:
2016-17 : 20 Crores

314
FINAL C.A. – AUDIT

2017-18 : 25 Crores
2018-19 : 35 Crores
Hospital has its own Pharmacy, Laboratory, Blood Bank, Radiology & General Stores.
its manage-ment suspects that leakages/theft is happening in Pharmacy, Radiology,
Laboratory and General Stores departments. It seeks advice of RST & Co., Internal
Auditors of the Company, as to how It can Institute/Improve its Internal Control. In
this context, Management wants to understand the , concept of components of Internal
Control Structure in detail. Advise.
Ans. Key components of Internal Control Structure:
Internal Control structure in an organization is referred to as the policies and procedures
established by the entity to provide reasonable assurance that the objectives are achieved.
The control structure in an organization basically has the following components:
1. Control Environment- Control environment covers the effect of various factors like
management attitude; awareness and actions for- establishing, enhancing or mitigating
the effectiveness of specific policies and procedures.
2. Accounting System –Accounting system means the process by which transactions are
processed for maintaining financial records. Accounting system identifies, assemble,
analyze, calculate, classify, record, summarize and report transactions and other events.
3. Control Procedure - Policies and procedures means those policies and procedures in
addition to the control environment and accounting systems which the management has
established to achieve the entity’s specific objectives. Such Policies and Procedures cover
the followings:
 Segregation of duties.
 Authorisation of Transactions.
 Adequacy of records and documents.
 Accountability and safeguarding of assets.
 Independent checks.

(C) As auditor of Z Ltd., you would like to limit your examination of account balance tests.
What are the control objectives you would like the accounting control system to achieve
to suit your purpose?
Ans. Control Objectives:
The objectives of internal control systems are determined by the management, after
considering the nature of business, scale of operations, the extent of professionalism
of the management etc. The objectives of internal controls relating to the accounting
system are:

315
FINAL C.A. – AUDIT

(i) Transactions are executed through general or specific management authorization.


(ii) All transactions are promptly recorded in an appropriate manner to permit the preparation
of financial information and to maintain accountability of assets.
(iii) Assets and records are safeguarded from unauthorized access, use or disposition.
(iv) Assets are verified at reasonable intervals and appropriate action is taken with regard
to the discrepancies.
Precisely, the control objectives ensure that the transactions processed are complete, valid
and accurate. The basic accounting control objectives which are sought to be achieved by any
accounting control system are:
(a) whether all transactions are recorded;
(b) whether recorded transactions are real;
(c) whether all recorded transactions are properly valued;
(d) whether all transactions are recorded timely;
(e) whether all transactions are properly posted;
(f) whether all transactions are properly classified and disclosed;
(g) whether all transactions are properly summarized.

(D) As an auditor, during your interim visit at Marathon Ltd. you observed that internal
controls were not in use throughout the period covered under audit. What are the
Controls objectives you would like to consider to achieve your purpose?
Ans. Control Objectives to be considered for Audit Purpose:
The objectives of internal control systems are determined by the management, after considering
the nature of business, scale of operations, the professionalism of the management etc.
Auditor’s knowledge about the existence of control activities assists the auditor in determining
whether it is necessary to devote additional attention to obtaining an understanding of control
activities.
To ensure whether the internal controls were in use throughout the period or not, the auditor
may consider the following control objectives:
1. Existence and effective implementation of policies and procedures efficient conduct of
business.
2. Safeguarding of assets.
3. Prevention and detection of frauds and errors.
4. Accuracy and completeness of the accounting records.
5. Timely preparation of reliable financial information.
6. Compliance with applicable laws and regulations.
7. Verification of assets at reasonable intervals.

316
FINAL C.A. – AUDIT

8. Proper authorization of transactions.


9. Monitoring of accounting/financial controls.
10. Reviews of performance.
11. Segregation of duties. so as to ensure orderly and

(E) In the use of standardized Internal Control Questionnaire (ICQ), certain basic
assumptions about elements of a good internal control system are taken into account. List
down few such assumptions.
Ans. Assumptions presumed about elements of good control while using standardized internal
control questionnaire:
1. Certain procedures in general used by most business concerns are essential in achieving
reliable internal control. For example, deposits into bank of the entire receipts of a day
or daily balancing of the cash book and ledgers or periodic reconciliation with the control
accounts
2. Extensive division of duties and responsibilities within the organisation.
3. Separation of accounting function with the custodial function.
4. No single person is entrusted with the responsibility of completing a transaction all by
himself.
5. There should always be evidence to identify the person who has done the work whether
involving authorisation, implementation or checking.
6. The work performed by each one is expected to come under review of another in the usual
course of routine.
7. There is proper documentation and recording of the transactions.

(F) Explain briefly the Flow Chart technique for evaluation of the Internal Control system.
Ans. Flow Chart Technique for evaluation of Internal Control System:
1. It is a graphic presentation of internal controls in the organisation and is normally
drawn up to show the controls in each section or sub-section.
2. It Provides the most concise and comprehensive way for reviewing the internal controls
an the evaluator’s findings.
3. A flow chart is a diagram full with lines and symbols and if judicious use of them can be
made, it is -probably an effective way of presenting the state of internal controls in the
client’s organisation.
4. A properly drawn up flow chart can provide a neat Visual picture of the whole activities
of the section or department involving flow of documents and activities. More specifically
it can show:

317
FINAL C.A. – AUDIT

 At what point to document is raised internally or received from external sources;


 The number of copies in which a document is raised or received:
 The intermediate stages set sequentially through which the document and the
activity pass;
 Distribution of the documents to various sections, department or operations;
 Checking authorisation and matching at relevant stages;
 Filing of the documents; and
 Final disposal by sending out or destruction.

(G) A newly qualified professional has received his first appointment as auditor of a large
company and is very much concerned about the effectiveness of internal control and
wants to assess and evaluate the control environment as part of his audit program.
Towards achieving his objective, he seeks your help in knowing the Standard Operating
Procedures (SOPs) of assessment and evaluation of control.
Ans. Standard Operating Procedures (SOPs) of assessment and evaluation of control:
1. Enterprise Risk Management: Organization having robust processes to identify &
mitigate risks across the entity & its periodical review will assist-in early identification
of weaknesses in internal control and taking effective control measures. In such entities,
surprises of failures in controls is likely to be few.

2. Segregation of Job Responsibilities: Segregation of duties is an important element of


control which ensures that no two commercial activities should be conducted by the same
person.

3. Job Rotation in Sensitive Areas: In key commercial functions, job rotation is regularly
followed to avoid degeneration of controls.

4. Documents of delegation of Financial Powers: Document on delegation of powers allows


controls to be clearly operated without being dependent on individuals.

5. IT based Controls: In an IT Environment, it is much easier to embed controls through


the system instead of being human dependant. The failure rate for IT embedded controls
is likely to be low, is likely to have better audit trail & is thus easier to monitor.

(H) Y Co. Ltd. has five entertainment centers to provide recreational facilities for public
especially for children and youngsters at 5 different locations in the peripheral of 20

318
FINAL C.A. – AUDIT

kilometers. Collections are made in cash. Specify the adequate system towards collection
of money.
Ans. System towards collection of money:
(a) Printing of tickets: Tickets should be serially numbered and pre-printed Serial numbers
should not he repeated during a reasonable period, say a month or year depending on the
turnover.
(b) Sale of Tickets: Tickets should be sold from the Central ticket office at each of the 5
centers, preferably through machines.
(c) Reconciliation of daily cash: Cash collection at each office should be reconciled with the
number of tickets sold.
(d) Banking of daily cash collection: Daily collection should be deposited in the bank on next
working day of the bank. Till that time, the cash should be in the custody of properly
authorized person.
(e) Cancellation of Entrance ticket: Entrance tickets should be cancelled at the entrance
gate when public enters the centre.
(f) Advance booking: If advance booking of facility is made available, the system should
ensure that all advance booked tickets are paid for.
(g) Discounts and free pass: The discount policy should be such that the concessional rates
should be properly authorized and signed forms for such authorization should be preserved.
(h) Surprise checks: Internal audit system should carry out periodic surprise checks for cash
counts, daily banking, reconciliation and stock of unsold tickets etc.

(I) During the course of his audit, the auditor noticed material weaknesses in the Internal
control system and he wishes to communicate the same to the management. You are
required to elucidate the important points the auditor should keep in mind while drafting
the letter of weaknesses in internal control system.
Ans. Points to be considered while drafting letter of weaknesses:
As per SA 265, “Communicating Deficiencies in Internal Control to Those who Charged with
Governance and Management”, the auditor shall include in the written communication of
significant deficiencies in internal control—
(a) A description of the deficiencies and an explanation of their potential effects; and

(b) Sufficient information to enable those charged with governance and management to
understand the context of the communication.
This communication should be, preferably, in writing through a letter of weakness.
Important points with regard to such a letter are as follows:

319
FINAL C.A. – AUDIT

(a) It lists down the area of weaknesses in the internal control system and recommends
suggestions for improvement.
(b) It should clearly indicate that this letter covers only weaknesses which have come
to the attention of the auditor during his evaluation of internal control for the
purpose of determining nature, timing and extent of further audit procedures.
(c) Letter should clearly indicate that his examination of internal control has designed
to determine the adequacy of internal control for management.
(d) This letter serves as a significant means for management and governing body for
the purpose of improving the system and its strict implementation.
(e) The letter may also serve to minimize legal liability in the event of a major defalcation
or other loss resulting from a weakness-in internal control.
IV- Internal Chek
(A) Write short note on: Objectives of Internal Check System.
Ans. Objectives of Internal Check System:
1. To detect error and frauds with ease.
2. To avoid and minimize the possibility of commission of errors and fraud.
3. To increase the efficiency of the staff working within the organization.
4. To locate the responsibility area or the stages where actual fraud and error occurs.
5. To protect the integrity of the business by ensuring that accounts are always subject to
proper scrutiny and check.
6. To prevent and avoid the misappropriation or embezzlement of cash and falsification of
accounts.
(B) The Auditor of S Limited has just commenced the statutory audit. What should be
considerations for the effectiveness of a system of internal check?
Or
State the considerations on which effectiveness of an efficient system of internal check
depends.
Ans. Considerations for effectiveness of a system of internal check:
1. No single person should have an independent control over any important aspect of the
business.
2. The duties of members of the staff should be changed from time to time without any
previous notice so that the same officer or subordinate does not, without a break, perform
the same function for a considerable length of time.
3. Every member of the staff should be encouraged to go on leave at least once in a year.
4. Persons having physical custody of assets must not be permitted to have access to the
books of account.

320
FINAL C.A. – AUDIT

5. To prevent loss or misappropriation of cash, mechanical devices, such as the automatic


cash register, should be employed.
6. Budgetary control would enable the management to review from time to time the progress
of trading activities.
7. The financial and administrative powers should be distributed very judiciously among
different officers and the manner in which these are actually exercised should be reviewed
periodically.
8. Procedures should be laid down for periodical verification and testing of different sections
of accounting records to ensure that they are accurate.
9. Accounting procedures should be reviewed periodically, for, even well-designed and carefully
installed procedures, in course of time, cease to be effective.

(C) New life Hospital is a multi-specialty hospital which has been facing a lot of pilferage
and troubles regarding their inventory maintenance and control. On investigation into
the matter it was found that the person in charge of inventory inflow and outflow from
the store house is also responsible for purchases and maintaining inventory records.
According to you, which basic system which needs to be maintained and checked by the
management.
Ans. Deficiencies in internal Control System:
An organisation is required to segregate the responsibilities of its employees in such a manner
that no single person person should have an independent control over any important aspect
of the business. In the present case, person in charge of inventory is not Only responsible for
inflow of inventory from store house but also responsible for purchase and maintaining inventory
records. So in this case, one of the essential of internal check system that independent and
complete control should not be given to a single person has been violated.

(D) BSF Limited is engaged in the business of trading leather goods. You are the internal
auditor of the company for the year 20113-19, in order to review internal controls of the
Sales Department of the company, you visited the Department and noticed the work division
as follows:
(1) An officer was handling the sales ledger and cash receipts.
(2) Another official was handling dispatch of goods and issuance of Delivery challans.
(3) One more officer was there to handle customer/debtor accounts and issu.e of receipts.
As an internal auditor, you are required to briefly discuss the general condition pertaining
to the internal check system prevalent in internal control system. Do you think that
there was proper division of work in BSF Limited? If not, why?

321
FINAL C.A. – AUDIT

Ans. General Conditions pertaining to internal check:


Refer the considerations specified in Q. No. 22.
In the given scenario, Company has not done proper division of work as:
(i) The receipts of cash should not be handled by the official handling sales ledger and
(ii) Delivery challans should be verified by an authorised official other than the officer
handling despatch of goods.

322
FINAL C.A. – AUDIT

INTERNAL, MANAGEMENT
AND OPERATIONAL AUDIT

Part 1 Content

Part A: Internal Audit

1. Meaning
1.1 The Institute of Chartered Accountants of India has constituted a Committee on
Internal Audit (CIA) as a non-standing committee on February 5, 2004. The CIA
was constituted (now known as Internal Audit Standard Board) with the object of
formulating Standards and Guidance Notes on Internal Audit
1.2 As defined in scope of the Standards on Internal Audit, Internal Audit means “an
independent management function, which involves a continuous and critical appraisal of
the functioning of an entity with a view to suggest improvements thereto and add value
to and strengthen the overall governance mechanism of the entity, including the entity’s
strategic risk management and internal control system.
1.3 The internal auditing need not to be contined to financial transactions and its scope may
be extended to the task of reviewing whether the resource utilisation of the enterprise is
efficient and economical.

2. Sec 138 of Companies Act, 2013


2.1 Eligibility:
F Either be a chartered accountant or

F A cost accountant (whether engaged in practice or not), or

F Such other professional as may be decided by the Board to conduct internal audit
of the functions and activities of the companies.
The internal auditor may or may not be an employee of the company.
2.2 Applicability
(a) Every listed company
(b) Every unlisted public company having-
i) Paid up share capital of fifty crore rupees or more during the preceding financial
year; or

323
FINAL C.A. – AUDIT

(ii) Turnover of two hundred crore rupees or more during the preceding financial year;
or
iii) Outstanding loans or borrowings from banks or public financial institutions exceeding
one hundred crore rupees or more at any point of time during the preceding financial
year; or
(iv) Outstanding deposits of twenty five crore rupees or more at any point of time during
the preceding financial year; and
(c) Every private company having-
(i) Turnover of two hundred crore rupees or more during the preceding financial year;
or
(ii) Outstanding loans or borrowings from banks or public financial institutions exceeding
one hundred crore rupees or more at any point of time during the preceding financial
year.

3. Primary areas of internal auditor’s responsibility


3.1 To maintain adequate system of internal control by a continuous examination of
accounting procedures, receipts and disbursements and to provide adequate safeguard
against misappropriation of assets.
3.2 To operate independently of the accounting staff and must not in any way divest himself
of any of the responsibilities placed upon him.
3.3 Not to involve himself in the performance of executive functions in order that his objective
outlook does not get obscured by the creation or vested interest
3.4 To observe facts and situations and bring them to notice of authorities who would otherwise
never know them; also, they critically appraise various policies of the management and
draw its attention to any deficiencies, wherever these require to be corrected.
3.5 To associate closely with management and his knowledge must be kept up to date by his
being kept informed about all important occurrences and events affecting the business,
as well as the changes that are made in business policies. He must enjoy an independent
status

4. Scope
4.1 Review of Internal Control System and Procedures:
(a) The internal auditor should determine whether the internal control system is in
consonance with the organisational structure. As far as possible, controls should
be in-built in the operating functions, if they are to be cost effective.
(b) Each control should be reviewed and analysed in terms of its costs and benefits.

324
FINAL C.A. – AUDIT

4.2 Review of Custodianship and Safeguarding of Assets


(a) The internal auditor should review the control systems to ensure that all assets
are accounted for fully. He should review the means used for safeguarding assets
against losses
(b) He should review the control systems for intangible assets e.g. the procedures
relating to credit control. Where an enterprise uses electronic data processing
equipment, the physical and systems control on processing facilities as well as on
data storage should be examined and tested
(c) He should also verify the existence of the assets
4.3 Review of Compliance with Policies, Plans, Procedures and Regulations
(a) It is essential that the various functional segments of an enterprise comply with
the relevant policies, plans, procedures, laws and regulations so that the operations
are carried out in coordinated manner
(b) He should examine the system of periodical review of existing policies particularly
when there is a change in the method and nature of operations of the enterprise.
4.4 Review of Relevance and Reliability of Information
(a) He should examine the accuracy and reliability of financial and operational records
(b) The usefulness of the reports as well as of the records should be evaluated with
reference to their costs
(c) The internal auditor should examine whether the reporting is by exception i.e. the
reports highlight the significant and distinctive features
4.5 Review of the Organisation Structure :
The internal auditor should conduct an appraisal of the organisation structure to
ascertain whether it is in harmony with the objective of the enterprise and whether the
assignment of responsibilities is in consonance therewith. For this purpose:
a) Review the manner in which the activities of the enterprise are grouped for managerial
control
b) Review whether responsibility and authority are in harmony with the grouping pattern.
c) Examine the organisation chart to find out whether the structure is simple and economical
and that no function enjoys an undue dominance over the others.
d) Responsibilities of managerial staff at headquarters do not overlap with those of chief
executives at operating units
e) Whether there is a satisfactory balance between authority and responsibility of
important executives.
(f) hether there is a unity of command i.e., whether each person reports only to one superior.

325
FINAL C.A. – AUDIT

(g) Where dual responsibilities cannot be avoided, the primary one should be specified and
the specific responsibility to each senior fixed.

4.6 Review of Utilisation of Resources:


(a) Whether proper operating standards and norms have been established for measuring
economical and efficient use of resources
(b) Where there is a wide divergence between actual performance and the corresponding
standards, reasons may be considered. As a part of evaluating resources utilisation,
identifying the facilities which are under-utilized is an important function of the internal
auditor.
4.7 Review of Accomplishment of Goals and Objectives:
(a) The internal auditor should review the overall objectives of the enterprise to evaluate
whether they are clearly stated and are attainable.
(b) The internal auditor should examine whether to the extent possible, objectives are
expressed in precise quantifiable terms (both monetary and non-monetary) to facilitate
detailed planning to be made for achieving them.
(c) There should also be sufficient flexibility in the plans to permit such improvements in
their implementation, as would benefit the enterprises as a whole.

5. Overall qualifications of internal auditor Accounts


5.1 The internal auditor should have the special expertise necessary for evaluating management
control systems, especially financial and accounting controls.
5.2 Accounting and finance functions provide basic data for management control of an
enterprise. Therefore the internal auditor must have accounting and financial expertise
to be able to discharge his duties.
5.3 The internal auditor is also expected to evaluate operational performance and non-
monetary, operational controls. This requires a basic knowledge of the technology and
commercial practices of the enterprise.
5.4 He should also have a basic knowledge of commerce, 1laws, taxation, cost accounting,
economics, quantitative methods and EDP systems
5.5 An understanding of management principles and techniques is another essential
qualification of an internal auditor as also the ability to deal with people.
5.6 By his conduct the internal auditor should provide an assurance to the management that
confidentiality of such information would be maintained.

6. Internal audit report- Elements


326
FINAL C.A. – AUDIT

6.1 Title;
6.2 Addressee;
6.3 Report Distribution List;
6.4 Period of coverage of the Report;
6.5 Opening or introductory paragraph;
(a) ldentification of the processes/functions and items or financial statements audited;
and
(b) A statement of the responsibility of the entity’s management and the responsibility
of the internal auditor;
6.6 Objectives paragraph - statement of the objectives and scope of the internal audit
engagement;
6.7 Scope paragraph (describing the nature ofan internal audit):
(a) A reference to the generally accepted audit procedures in India, as applicable;
(b) A description of the engagement background and the methodology of the internal
audit together with procedures performed by the internal auditor; and
(c) A description of the population and the sampling technique used.
6.8 Executive Summary, highlighting the key material issues, observations, control weaknesses
and exceptions;
6.9 Observations, findings and recommendations made by the internal auditor.
6.10 Comments from the local management;
6.11 Action Taken Report - Action taken/ not taken pursuant to the observations made in the
previous internal audit reports
6.12 Date, Place, Signature, Membership number

7. Features of Good internal audit report


7.1 Objectivity -To maintain the credibility of internal audit function the comments and
opinions expressed in the report should be as objective and unbiased as possible.
7.2 Clarity - The language used should be simple and straight-forward. As far as practicable
use of technical terms and jargon should be avoided. Each draft of the report should be
reviewed by a senior who should attempt to read it from the point of view of the users of
the report
7.3 Accuracy -The information contained in the report, whether quantified or otherwise,
should be accurate. Where approximation or assumptions have been made the fact should
be clearly stated along with reasons, if material.
7.4 Conciseness -Brevity is vital subject, of course, to the condition that important information
should not be omitted

327
FINAL C.A. – AUDIT

7.5 Constructiveness - Destructive criticism should carefully be avoided in the report. The
report should clearly demonstrate that the internal auditor is trying to assist the auditor
in an effective discharge of his responsibilities.
7.6 Readability -The reader’s interest should be captured and retained throughout. For this,
appropriate paragraph heading may be used.
7.7 Timeliness - The report should be submitted promptly because if the time lag between
the occurrence of an event and its reporting is considerable, the opportunity for taking
action may be lost or a wrong decision may be taken in the absence of the information.
7.8 Findings and Conclusions - These may be given either department- wise or in the order
of importance. All the facts and data pertaining to the situation should be assembled,
classified and analysed. Each conclusion and opinion should normally follow the findings.
Tabies or graphs may be used for the presentation of statistical data in appendices.
7.9 Recommendations - An internal audit report usually includes recommendations for
potential improvements. In order to enable the management to accept and implement
the recommendations, the internal auditor should be able to convince the management
that the conclusions are logical and valid and the recommendations represent effective
and feasible ways of taking action.
7.10 Auditee’s views - The auditee’s views about audit conclusions or recommendations may
also be included in the audit report in appropriate circumstances.
7.11 Summary - A summary of conclusions and recommendations may be given at the end.
This is particularly useful in long reports.
7.12 Supporting information - The internal auditor should supplement his report by such
documents and data which adequately and convincingly support the conclusions.
Supporting information may include the relevant standards or regulations
7.13 Draft Report - Before writing the final report, the internal auditor should prepare a
draft report. This would help him in finding out the most effective manner of presenting
his reports. It would also indicate whether there is any Superfluous information or a gap
in reasoning.

8. Relationship Between Internal and external auditor


8.1 The internal auditor operates in various areas such as review of accounting system and
internal control; examination of financial and operating information for the benefit of
management, examination of the economy, efficiency and effectiveness of operations
including nonfinancial controls of various tangible assets of the entity. While operating
in these areas, there is lot of overlapping between the work of internal auditor and
external auditor.

328
FINAL C.A. – AUDIT

8.2 The work done by internal auditor has an important bearing on the Work performed by
the statutory auditor as evaluation done by the internal auditor in respect of internal
controls, reliability of financial information, verification of assets etc. is also required to
be done by the external auditor. The function of an internal auditor is an integral part
of the system of internal control.
8.3 It is statutory requirement too as per section 138 of the Companies Act, 2013 where the
Audit Committee of the company or the Board shall, in consultation with the Internal
Auditor, formulate the scope, functioning periodicity and methodology for conducting the
internal audit.
8.4 However, it is obligatory for a statutory auditor to examine the scope and effectiveness
of the work carried out by the internal auditor. For the purpose, he should examine the
Internal Audit Department of the organisation, the strength of the internal audit staff,
their qualification and their powers.
8.5 The extent of independence exhibited by the internal auditor in the discharge of his
duties and his status in the organisation are important factors for determining the
effectiveness of his audit.. But so far, the practice of audit being conducted jointly by the
internal auditors is of great assistance to statutory auditors.
8.6 The external auditor should, as part of his audit, evaluate the internal audit function
to the extent he considers that it will be relevant in determining the nature, timing and
extent of his compliance and substantive procedures, Depending upon such evaluation,
the external auditor may be able to adopt less extensive procedures than would otherwise
be required.

9. Distinguish between Internal and External audit Requirements


BASIS FOR INTERNAL AUDIT EXTERNAL AUDIT
COMPARISON
1. Meaning It refers to an on-going audit It is an audit function
function performed within an performed by the independent
organization by a separate body which is not a part of
internal auditing department. the organization.
2. Examination Internal auditor examines the External auditor examines
Operational efficiency of the the Accuracy and Validity of
organisation. Financial Statements.
3. Appointment Internal auditor is appointed External auditor is appointed
by the Management by the Members.

329
FINAL C.A. – AUDIT

4.Users of Report User of Internal audit report User of external audit report
is the Management. is Stakeholders.
5. Period Internal audit is a Continuous External audit is done once in
Process throughout the year a year.
6.Opinion Opinion is provided on the Opinion is provided on the
effectiveness of the operational truthfulness and fairness of
activities of the organization. the financial statement of the
company.
7. Status of Internal auditor is employee External auditor is an
Auditor of the company, thus, less independent person
independent.

10. Read SA 610


Part B Management Audit
1 Meaning:
According to T.G. Rose,
The management audit would therefore concern itself with the whole
field of activities of the concern, from top to bottom, starting, as always
where management control is concerned, from the top, because we are
primarily concerned with whether the general management is
functioning smoothly and satisfactorily.
If it is not, it may be due to the functional management being faulty and,
therefore, we pass on to examine that in its turn, in order to find the missing
or faulty link which is causing the trouble”.

330
FINAL C.A. – AUDIT

2 Scope
(a) Management audit is wider in scope compared to operational audit
(b) In management audit, the auditor is to make his tests to the level of
top management, its formulation of objectives, plans and policies and
its decision making.
(c) From the point of view of the management auditor, knowledge about
the following is essential:
(i) Purpose for which the organisation has been created.
(ii) Management structure including delegation of authority planning
and budgeting.
(iii) Reports required for a proper management and the reports actually
received.
(iv) Internal controls.
(v) Nature of production of the business concerned in the broad way so
that he can understand the flow and content of work leading to
production and their mutual relationships. Some ideas about the
techniques, formulas, raw materials and Personnel requirement
would be of direct assistance to the management auditor.
(vi) Production planning
(vii) Factory layout, design and installed capacity.
(Viii) Personnel policy and personnel management including requirements,
training, welfare, incentives and disincentives.
(ix) Materials management including sources of important raw materials,
receipt of materials of the quality and quantity needed, storage,
supervision and safe custody, insurance and the procedure for
issue of materials.
(x) Sales management and sales planning including advertisement policy.
(xi) Decision making process.
3 Why is management audit desirable?
(a) The principal reason for undertaking a management audit is the need
for detecting and overcoming current managerial deficiencies (and
resulting operational problems) in ongoing operations.
(b) Evaluates how well management accomplishes its stated organisational
objectives; how effective management is in planning, organising, directing,
and controlling the organisation’s activities; and how appropriate
management’s decisions are for reaching stated organisation objectives.
331
FINAL C.A. – AUDIT

(c) The managerial problems and related operational difficulties can be


spotted before the fact rather than after the fact as with a financial
audit.
(d) The benefit of management auditing is that it represents another
management tool to assist the organisation in accomplishing desired
objectives.
(e) Clearly helpful in the case of ailing industries, to isolate the problems and
account for their ailments.
4 Organising Management audit
(a) Devising Statement of policy-scope discussion in consultation with top
management
(b) Location of audit personnel within the organisation- Management audit
function should be independent.
(c) Allocation of personnel- technical and experienced audit team
(d) Time allocation and cost consideration
(e) Frequency of reporting and conducting audit- to be fixed in consultation
with top management.

332
FINAL C.A. – AUDIT

5 Conducting Management Audit


(1) Getting the facts through interviews
(a) The management auditors should know what information is desired,
and they should be prepared to ask a number of direct questions to get
the desired information.
(b) The auditors should begin by stating the purpose of the audit. Emphasis
should be placed on getting the facts that are essential to review and
appraise the functional area(s) under study
(2) Measuring performance through the Management Audit Questionnaire-
(a) The questionnaire is concerned with the appraisal of management
actions in accomplishing organisation objectives. Its primary
objective is to highlight weaknesses and deficiencies of the
organisation for possible improvements.
(b) Its primary objective is to highlight weaknesses and deficiencies
of the organisation. It includes a review of how well or badly
the management functions of planning organising, directing and
controlling are being performed.
(c) The questionnaire provides a means for evaluating an organisation’s
ongoing operations by examining its major functional areas. There
are three possible answers to the management audit questions-
YES, NO and NOT NOT APPLICABLE.
6 Categories of Management audit
(a) Reports prepared by the management audit staff after their visits to
a unit. Periodical reports prepared by senior members of management
(b) audit department which summarise the main audit findings and
recommendations for the period under consideration and which afford
(a) concise review of the department’s activities for that period.
(c) Reports on the results of special investigations and inquiries,
(d) An annual audit report
7 Types of Report
(a) Oral reports - In many situations, the reporting of results will be on an
oral basis. To some extent, this is inevitable since a part of the actual
audit effort is carried on in conjunction with company personnel. In
other cases, it is a result of emergency action needs. It may also be a
prelude to more formal written reports.

333
FINAL C.A. – AUDIT

(b) Interim written reports - In situations where it is deemed advisable to


inform management of Significant developments during the course of
the audit, or at least preceding the release of the regular report, there
may be some kind of interim written report.
(c) Regular written reports - In the typical situation, the particular audit
assignment will include the preparation of a formal written report. The
form and content of such written reports will vary widely, both as
between individual audit assignments and individual companies.
(d) Summary written reports - These summary reports are also referred to
as ‘flash’ reports’. In a number of companies the practice has developed
of issuing an annual (or sometimes more frequent) report summarising
the various individual reports issued, and describing the range of their
content. These summary reports in some cases are primarily for audit
committees of Boards of Directors, but in other cases for higher level
management
8 Management audit report- Elements
Organisation of the Written Report-
Format -Though it is difficult to lay down a format applicable to all situations,
yet the following general guidelines may be observed:
(i) Title - The management audit report should have a short but descriptive
title so that its subject matter can be easily identified
(ii) Objectives -The management auditor may describe the objectives of the
audit assignment.
(iii) Scope - The management auditor may give a brief description of the
activities audited by him.
(iv) Findings conclusions and opinions - These may be given either department
wise or in the order of importance. All the facts and data pertaining
to the, situation should be assembled, classified and analysed. Each
finding should be discussed comprehensively and correlated with other
findings. Conclusions and opinions should normally follow the findings.
Tables or graphs may be used for the presentation of statistical data
in appendices.
(v) Recommendations - A management audit report may include
recommendations for potential improvements.
(vi) Auditee’s views - The auditee’s views about audit conclusions or
recommendations may also be included in the audit report in appropriate
circumstances.
(vii) Summary - A summary of conclusions and recommendations may be
given at the end. This is particularly useful in long reports.

334
FINAL C.A. – AUDIT

9 Behavioural aspects encountered in management audit


1) Staff/line conflict -The staff/line relationship is inherently prone to
conflict. Management auditors are staff. And line people in the sense all
members of other departments of the organisation are likely to regard
the management auditor the same way as they regard other staff
people. Management auditors being specialists in their field may think
that their approach and solutions are the only answers.
(2) Control- amount of fear that his actions when reported are likely to
cause adverse effect on those who receive the auditor’s report. viz, top
management.
10 Solution to behavioural problems
(1) Create an atmosphere of trust and friendliness
(2) Constructive criticism -It is essential that the auditor should concentrate
only on constructive criticism. He should also make obvious in his report
the value of his comments in tangible terms.
(3) Reporting methods- To achieve this objective, the auditor has to make a
concerted effort to convey effectively his role by adopting a friendly but
firm tone in his report. It is always possible to disagree without being
disagreeable.
(4) Participative approach - It is well established that auditor’s reports
have better acceptability if the improvements suggested are discussed
with those who have to implement them and made to feel that they
have participated in the recommendations made for improvements.
Part C-Operational Audit
1 Meaning:
Operational auditing is a systematic process of evaluating an organisation’s
effectiveness, efficiency and economy of operations under management’s
control and reporting to appropriate persons the results of the evaluation
along with recommendations for improvement

335
FINAL C.A. – AUDIT

2 Relationship with Internal Audit


(a) Internal auditing is an independent appraisal activity within an
organisation for the review of operations as a service to organisation.
Naturally, when an auditor is concerned with the appraisal of operations,
he becomes an operational auditor
(b) There probably may not be much of difference in viewing operational
audit as a review and appraisal of operations of an organisation carried
on by a competent independent person. Operational audit also involves
appraisal of controls as one of its primary objective.
3 Relationship with management audit
(a) Management audit is concerned with the “Quality of managing whereas
operational audit focuses on the “Quality of operations”.
(b) Management audit is the “Audit of Management” while operational audit
is the “Audit for the management”.
(c) The basic difference between the two audits, then, is not in method, but
in the level of appraisal. In management audit, the auditor is to make
his tests to the level of top management, its formulation of objective,
plans and policies and its decision making. It is not that he just verifies
the operations of control and procedures and fulfillment of plans in
conformity with the prescribed policies.
(d) It S safe to conclude that management audit includes the coverage
of operational audit. The Two Audits are Complementary and
Supplementary to One Another

336
FINAL C.A. – AUDIT

4 Objectives of operational audit


(a) Appraisal of controls- if controls are weak or breaking down, however
well-equipped or well-manned the organisation may be, it will fail to
operate effectively
(b) Appraisal of performance - In the task of performance evaluation, an
operational auditor is heavily dependent upon availability of acceptable
standards.
(c) Appraisal of objectives and plans- He prepares his evaluation programme
in such a manner that it will show how well or how poorly the department
has fared by reference to applicable standards, procedures, rules,
policies and plans.
(d) Appraisal of organisational structure - Organisational structure provides
the line of relationships and delegation of authority and tasks. This is
an important element of the internal control design.
5 Types of operational audit
(1) Functional Audits - Functions are a means of categorizing the activities
of a business, such as the billing function or production function. There
are many ways to categorize and subdivide functions.
(2) Organizational Audits - An operational audit of an organization deals
with an entire organizational unit. such as a department, branch,
or subsidiary. An organizational audit emphasizes how efficiently and
effectively functions interact.
(3) Special Assignments - In operational auditing, special assignments
arise at the request of management. There are a wide variety of such
audits. Examples include determining the cause of an ineffective IT
system, investigating the possibility of fraud in a division, and making
recommendations for reducing the cost of a manufactured product.

337
FINAL C.A. – AUDIT

6 Why operational audit is desirable:


(a) Executives and managers are too preoccupied with implementation of
plans and achieving of targets. They are let with very little time to
collect information and locate problems.
(b) Managers or their aides are generally relied upon for transmitting
information than for booking for information or for analysing situations
(c) The information that is transmitted by managers is not necessarily
objective often it may be biased for various reasons.
(d) Conventional internal audit reports are often routine and mechanical in
character and have a definite leaning towards accounting and financial
information. They are also historical in nature.
(e) Surveys and special these investigations, no doubt, are very useful Du
are at the best occasional in character. Also, they are costly time
consuming and keep the departmental key personnel busy during the
period they are on. These are basically an attempt to carry out a post-
mortem rather than to enlighten the management about the ways on
improvement.
7 Qualities of operational auditor
(a) In areas beyond accounting and finance, his knowledge ordinarily would
be rather scanty and this is a reason which should make him even more
inquisitive.
(b) He should ask the who, why, how of everything. He should to visualise
whether simpler alternative means are available to do a particular work.
(c) He should try to see everything as to whether that properly fits In the
business frame and organisational policy. He should be persistent and
should possess an attitude of skepticism.
(d) He should not give up or feel satisfied easily. He should imbibe a
constructive approach rather than a fault-finding approach and
should give a feeling that his efforts are to help attaining an improved
operation and not merely fault finding.
(e) If the auditor succeeds in giving a feeling of help and assistance through
constructive criticism, he will be able to obtain co-operation of the
persons who are involved in the operations.

338
FINAL C.A. – AUDIT

8 Review of systems and procedures


(a) The word ‘system’ is commonly defined as “a set of objects, together with
relationships between the objects and their attributes, connected or
related to each other and to their environment in such a manner as to
form an entire or integral whole”.
(b) Procedures are the means by which policies are implemented. Most
often, procedures entail the use of documents in accordance with precise
instructions or methods to be used
(c) In the study of the systems and procedural functions, the auditor
should ask himself:
(1) Is the function properly located in the organisation?
(2) Do the staff personnel have the necessary training and experience
to perform the work?
(3) Has a definite programme been established and has been taken
for its attentive accomplishment?
(4) Is productivity satisfactory?
9 Difference between financial and operational auditing
The major differences between financial and operational auditing can be
described as follows:
(i) Purpose - The financial auditing is basically concerned with the opinion
that whether the historical information recorded is correct or not,
whereas the operational auditing emphasizes on effectiveness and
efficiency of operations for future performance
(ii) Area- Financial audits are restricted to the matters directly affecting
the appropriateness of the presented financial statements but the
operational auditing covers all the activities that are related to efficiency
and effectiveness of operations directed towards accomplishment of
objectives of organization.
(iii) Reporting - The financial audit report is sent to all stock holders
bankers and other persons having stake in the Organisation. However
the operational audit report is primarily for the management.
(iv) End Task - The financial audit has reporting the findings to the persons
getting the report as its end objective, however, the operational auditing
is not limited to reporting only but include suggestions for improvement
also.

339
FINAL C.A. – AUDIT

AUDIT OF CONSOLIDATED
STATEMENTS FINANCIAL

Part 1 Content

1. Consolidated Financial Statements includes


1.1 Consolidated Balance Sheet
1.2 Consolidated Statement of Profit and loss
1.3 Consolidated Cash Flow Statement
1.4 Consolidated Statement of equity, if applicable
1.5 Any explanatory notes annexed to or forming part thereof

2. The formats for preparation of balance sheet, statement of profit and loss and a
statement of change in equity (it applicable) are prescribed under the Schedule llI of the
Companies Act, 2013

3. Accounting Standard (AS) 21 consolidated Financial Statements and Indian Accounting


Standard (Ind Ad) 110, “Consolidated Financial Statements’ lay down principles and
procedures for preparation and presentation of consolidated financial statements under
AS and Ind AS respectively.

4. Companies Act, 2013


F According to Section 129(3) of the Companies Act, 2013, where a company has one or

more subsidiaries, including associate company and joint venture, it shall, in addition to
its own financial statements prepare a consolidated financial statement of the company
and of all the subsidiaries in the same form and manner as that of its own.
F Further, section 129(4) of the said Act, provides that the provisions applicable to the

preparation, adoption and audit of the financial statements of a holding company shall,
mutatis mutandis, also apply to its the consolidated financial statements financial
F The consolidated financial statements shall also be approved by the Board of Directors

before they are signed on behalf of the Board, along with its standalone financial
statements and shall also be laid before the annual general meeting of the company along
with the laying of its standalone financial statement.

340
FINAL C.A. – AUDIT

F The company shall also attach along with its financial statement, a separate statement

containing the salient features of the financial statement of its subsidiary(ies) in Form
AOC-1.
F According to the Companies (Accounts) Rules, 2014, the consolidation of financial

statements of the company shall be made in accordance with the| provisions of Schedule
III to the Act and the applicable accounting standards.
F A company which is not required to prepare consolidated financial statements under the

Accounting Standards, it shall be sufficient if the | company complies with provisions of


consolidated financial statements provided in Schedule IlI of the Act
F However, the requirement related to preparation of consolidated financial statements

shall not apply to a company if it meets the following conditions:


(a) It is a wholly-owned subsidiary, or is a partially-owned subsidiary of another company
and all its other members, including those not otherwise entitled to vote, having
been intimated in writing and for which the proof of delivery of such intimation
is available with thei company, do not object to the company not presenting
consolidatea financial statements;
(b) it is a company whose securities are not listed or are not in the process of listing
on any stock exchange, whether in India or outside India
(c) Its ultimate or any intermediate holding company files consolidated (financial
statements with the Registrar which are in compliance with the applicable
Accounting Standards
F As per sub-section 6 of the section 129 of the Companies Act, 2013, the Central

Government may, on its own or on an application by class classes of companies, by


notification, exempt any class or classes of companies from complying with any of
the requirements of section 129 or the rules made thereunder, if it is considered
necessary to grant such exemption in the public interest and any such exemption
may be granted either unconditionally or subject to such conditions as may be
specified in the notification

5. Investment entity exemption


5.1 An investment entity need not present consolidated financial statements if it is required,
in accordance with paragraph 31 of Ind AS 110, to measure all of its subsidiaries at
fair value through profit or loss. A parent shall determine whether it is an investment
entity.
5.2 An investment entity is an entity that:
a) Obtains funds from one or more investors for the purpose of providing those

341
FINAL C.A. – AUDIT

investor(s) with investment management services


b) Commits to its investor(s) that its business purpose is to invest funds solely for
returns from capital appreciation, investment income, or both; and
c) Measures and evaluates the performance of substantially all of its investments on
a fair value basis
5.3 However, as per paragraph 33 of Ind AS 110, parent of an investment entity shall
consolidate all entities that it controls, including those controlled through an investment
entity subsidiary, unless the parent itself is an investment entity

6. Special adjustments in CFS


6.1 Permanent Consolidation Adjustments:
Permanent consolidation adjustments are those adjustments that are made only on the
first occasion or subsequent occasions in which there is a change in the shareholding of
a a particular entity which is consolidated. Permanent consolidation adjustments are
(a) Determination of goodwill or capital reserve as per applicable accounting standards.
(b) Determination of amount of equity attributable to minority/ non-controlling
interests.
Auditor’s Role:
The auditor should verify that the above calculations have been made appropriately.
 The auditor should pay particular attention to the determination of pre-acquisition

reserves of the components. Date(s) of investment in components assumes importance


in this regard.

 The auditor should also examine whether the pre-acquisition reserves have been allocated
appropriately between the parent and the minority interests/ non-controlling interests
of the subsidiary.

 The auditor should also verify the changes that might have taken place in these
permanent consolidation adjustments on account of subsequent acquisition of shares in
the components, disposal of the components in the subsequent years.

6.2 Current Period Consolidation Adjustments


Current period adjustments are those adjustments that are made in the accounting
period for which the consolidation of financial statements is done.
Current period consolidation adjustments primarily relate to elimination of intra-group
transactions and account balances including:

342
FINAL C.A. – AUDIT

(a) Intra-group interest paid and received, or management fees, etc.;


(b) Unrealised intra-group profits on assets from/ to other subsidiaries; acquired/ transferred
(c) Record deferred taxes on unrealised intercompany profits elimination in accordance with
Ind AS 12
(d) Intra-group indebtedness
(e) Adjustments related to harmonising the different accounting policies being followed by
the parent and its components
(f) Adjustments to the financial statements (of the parent and the components being
consolidated) for recognized subsequent events or transactions that occur between the
balance sheet date and the date of the auditor’s report on the consolidated financial
statements of the group
(g) Adjustments for the effects of significant transactions or other events that occur
between the date of the components balance sheet and not already recognised in its
financial statements and the date of the auditor’s report on the group’s consolidated
financial statements when the financial statements of the component to be used for
consolidation are not drawn upto the same balance sheet date as that of the parent.
(h) In case of a foreign component, adjustments to convert a component’s audited financial
statements prepared under the component’s local GAAP to the GAAP under consolidated
financial statements are prepared.
(i) Determination of movement in equity attributable to the minorities interest/non-
controlling interest since the date of acquisition of the subsidiary. It should also be noted
that under Ind AS, non-controlling interest can also result in negative halance. Unlike
earlier AS, as per paragraph 28 of Ind AS 27, if the net worth of subsidiary is negative,
non-controlling interest could have deficit balance
(j) Adiustments of deferred tax on account of temporary differences arising out of elimination
of profit and losses resulting from intragroup transactions and undistributed profits of
the component in case of consolidated financial statements prepared under Ind AS

8.4 Planning the audit of consolidation: (SA 300)


Before commencing an audit of consolidated financial statements, the auditor should plan his
work to enable him to conduct an effective audit in an efficient and timely manner.
The auditor should make plans, among other things, for the following:
(a) Understanding of the group structure and group-wide controls including assessment of
Information Technology (IT) system and related general and applications IT related
controls (manual and automated) for consolidation process
(b) Understanding of accounting policies of the parent and its components as well as of the
343
FINAL C.A. – AUDIT

consolidation process including the process of translation of financial statements of


foreign Components foreign
(c) Determining and programming the nature, timing, and extent of the audit procedures
to be performed based on the assessment of the risk of material misstatement in
consolidation process
(d) Determining the extent of use of other auditor’s work in the audit;| and
(e) Coordinating the work to be performed.
(f) Obtain a listing of all the components included in the consolidated financial statements
and review the information provided by the management of the parent identifying the
components. The auditor should verify that all the components have been included in the
consolidated financial statements unless these components meet criterion for exclusion.

8.5 Using the work of another auditor (SA 600)


The principal auditor, if he decides to use the work of another auditor in relation to the audit
of consolidated financial statements, should comply with the requirements of SA 600.
In carrying out the audit of the standalone financial statements, the computation of
materiality for the purpose of issuing an opinion on the standalone financial statements of
each component would be done component-wise on a standalone basis. However, with regard to
determination of materiality during the audit of consolidated financial statements (CFS), the
auditor should consider the following:
(a) The auditor is required to compute the materiality for the group as a whole. This materiality
should be used to assess the appropriateness of the consolidation adjustment (i.e.
permanent consolidation adjustments and currents period consolidation adjustments)
that are made by management in the preparation of CFS.
(b) The parent auditor can also use the materiality computed on the group level to determine
whether the component’s financial statements are material to the group to determine
whether they should scope in additional components, and consider using the work of
other auditors as applicable.
(c) The principal auditor also computes materiality for each component and communicates to
the component auditor, if he believes is required for true and fair view on CFS
(d) The principal auditor also obtains certain confirmations from component auditor like
independence, code of ethics, certain information required for consolidation and disclosure
requirements etc.
In respect of completeness of the information that all the components which should have been
considered have been actually considered, the auditor should perform the following procedures:

344
FINAL C.A. – AUDIT

(a) Review his working papers for the prior years for the known components
(b) Review the parent’s procedures for identification of various
(c) Make inquiries of the management to identify any new components or any component
which goes out of consolidated financial statement;
(d) Review the investments of parent as well as its components to determine the shareholding
in other entities;
(e) Review the joint ventures and joint arrangements as applicable:
(f) Review the other arrangements entered into by the parent that have not been included
in the consolidated financial statements of the group;
(g) Review the statutory records maintained by the parent, for example registers under
section 186, 190 of the Companies Act, 2013
(h) ldentify the changes in the shareholding that might have taken place during the reporting
period.

8.7 Written Representations


F SA 580, “Written Representations” requires the auditor to obtain written representations
from management and, appropriate, those charged with governance.
F The auditor of the consolidated financial statements should obtain evidence that the

management of the parent acknowledges its responsibility for a true and fair presentation
of the consolidated financial statements in accordance with the financial reporting
framework applicable to the parent and that parent management has approved the
consolidated financial statements.
F In addition, the auditor of the consolidated financial statements obtains written
representations from parent management on matters material to the consolidated
financial statements
(a) Completeness of components included in the CFS;
(b) Identification of reportable segments for segmental reporting
(c) Identification of related parties and related party transactions for reporting;
(d) Appropriateness and completeness of permanent and current period consolidation
adjustments, including the elimination of intra-group transactions.

Reporting
(a) When the parent’s auditor is also the auditor of all its components
issue audit report after considering applicable financial reporting framework on
consolidated and standalone financial statements
(b) When parent’s auditor is not the auditor of all its components

345
FINAL C.A. – AUDIT

While considering the observations (For instance modification and /or emphasis of
matter/other matter in accordance with SA 705/706) of the component auditor in his
report on the standalone financial statement, the parent auditor should comply with the
requirements of SA 600, “Using the Work of Another Auditor”
(c) Component auditor reports on FS under an accounting framework different than that
of the parent
Request ftor conversion into INDIAN GAAP and audit or conversion adjustments should
be done.
(d) Component auditor reports on FS under an auditing framework different than that of
the parent
Audit the FS under Indian Auditing framework
either parent entity auditor can himself audit or send a checklist as per Indian auditing
standards)
(e) When Components are unaudited
If component is material then Modify opinion because evidence not obtained

346
FINAL C.A. – AUDIT

PEER REVIEW AND QUALITY


REVIEW

I MEANING, OBJECTIVES AND SCOPE OF PEER REVIEW

(A) Meaning and Concept of Peer review


 The term “peer” means a person of similar standing.
 The term “Review” means conduct of re-examination or retrospective evaluation of the
subject matter.

(B) Objectives of Peer Review


(1) To ensure that members while carrying out assurance service assignment comply with
Technical, Professional and Ethical Standards issued by the institute.
(2) To ensure that member has in place system, including documentation thereof, for
maintaining the quality of assurance services.
(3) To ensure adherence to various statutory and regulatory requirements.
(4) To identify and address patterns of non-compliance with quality control standards.
Note: Primary objective of peer review is not to find out deficiencies but to improve the quality
of services rendered by members of the profession.

(C) Scope of Peer Review


The Peer Review process shall apply to all the assurance services provided by a Practice
Unit. While carrying out review, the reviewer examines the assurance engagement records
of the Practice Unit So as to cover the following. (TC3S -Q)
(C1) Compliance with Technical, Professional and Ethical Standard.
(Q) Quality of reporting.
(S) Systems and procedures for carrying out assurance services.
(T) Training programmes for staff (including articled and audit assistants) concerned
with assurance functions, including availability of appropriate infrastructure.
(C2) Compliance with directions and/or guidelines issued by the Council to the Members,
including Fees to be charged. Number of audits undertaken, register for Assurance
Engagements conducted during the year and such other related records.
(C3) Compliance with directions and/or guidelines issued by the Council in relating

347
FINAL C.A. – AUDIT

to article assistants and/or audit assistants, including attendance register, work


diaries, stiped payments, and such other related records.

(D) Technical, Professional and Ethical Standards


(i) Accounting Standards issued by ICAI and /or prescribed and notified by the C.G. of
India;

(ii) Standards issued by the Institute of Chartered Accountants of India including


(a) Engagement standards
(b) Statements
(c) Guidance notes
(d) Standards on Internal Audit
(e) Statements on Quality Control
(f) Notifications / Directions / Announcements / Guidelines / Pronouncements /
Professional standards issued from time to time by Council or any of its committees.

(iii) Framework for the preparation of financial statements, framework of statements and
Standard on Auditing Standard on Assurance Engagements, Standards on Quality
Control and Guidelines Notes on related services issued, from time to time, by the institute
of Chartered Accountants of India and framework for assurance engagements;

(v) Provisions of the various relevant statutes and/or regulations which are applicable in
the context of the specific engagements being Reviewed including instructions, guidelines,
notifications, directions issued by regulatory bodies as covered in the scope of assurance
engagements.

(E) Assurance Services


Assurance Services’ is used interchangeably with Audit Services, Attestation Functions, and
Audit Functions, but shall not include the following:
M Management Consultancy Engagements;
R Representation before various Authorities;
T Engagements to prepare tax returns or advising clients in taxation matters;
C Engagements for the compilation of financial statements;
A Engagements solely to assis the client in preparing compiling or collating information
other than financial statements;
E Testifying as an expert witness;

348
FINAL C.A. – AUDIT

E Providing expert opinion on points of principle, such as Accounting Standards or


the applicability of certain laws, on the basis of facts provided by the client; and
B Engagement for Due diligence.

(F) Once in three years


Applicable for LEVEL – I Entities which covers a PU which has undertaken any of the under-
mentioned assurance services in the period under review:
(a) Central Statutory Audit of Public Sector Banks, Private Sector Banks, Foreign Banks,
Cooperative banks and PFI;
(b) Central Statutory Audit of Central or State PSU & Central Cooperative Societies as may
be decided by the Board;
(c) Central Statutory Audit of Insurance Companies;
(d) Statutory Audit of asset management companies or mutual funds;
(e) Statutory Audit enterprises whose equity or debt securities are listed in India or abroad;
(f) Statutory Audit of Entities which have raised funds from public or banks or financial
institutions of over `50 Crores during the period under Review;
(g) Statutory Audit of Entities which have raised donations and/or contributions over `50
Crores during the period under review;
(h) Statutory Audit of entities having Net Worth of more then `500 Crores at any time
during the period under Review:
(i) Statutory Audit of entities which have been funded by CG and/or SG(s) schemes of over
`50 Crores during the period under Review;

ONCE IN FOUR YEARS


Applicable for LEVEL – II Entities which cover a PU which has undertaken any of the under-
mentioned assurance services in the period under Review:
(a) Statutory / Internal / Concurrent / Systems/ Tax audit and / or Departmental Review
of Branches / Offices of (1) Public Sector or Private Sector and / or Foreign Banks (2)
Insurance Companies (3) Co-operatives Banks (4) Statutory Audit of Regional Rural
Banks (e) Statutory Audit of NBFCs.
(b) Statutory Audit of entities having Net Worth over ` 5 Crores or an annual turnover of
more than ` 50 Crores during the period under review.
ONCE IN FIVE YEARS
 Applicable for LEVEL – III Entities which overs a PU not covered in Level I and Level
II.
Any PU not selected for Peer Review, may suo motu apply to the Board for the conduct

349
FINAL C.A. – AUDIT

of its Peer Review. The Board shall act upon the same within 30days from the date of
receipt of such request.

II PEER REVIEW BOARD (PRB)


(A) Constitution The PRB shall be constituted by the Council of ICAL
(B) Composition
 The Board shall consist of a maximum of 12 members to be appointed by the Council
of whom not less than 50% shall be from amongst the members of the Council.
 The Council may nominate members to the Board from outside bodies and from
amongst prominent individuals of high integrity and reputation, including but
not limited to regulatory authorities, bankers, academicians, economists, legal
Professionals and business executives.
 The Council shall appoint the Chairman and the Vice-Chairman from amongst its
elected Council members appointed on the Board.
(c) Tenure of Members
 The term ofa member shall be for 1year, or such other period as may be prescribed
by the Council from time to time.
 Casual vacancies on the Board shall be filled by the Council.

(D) Persons Disqualified


Members of the Disciplinary Committee or the Disciplinary Board or the Committee on
Ethical Standards or the Committee on Financial Reporting and Review Board of the
ICAl shall not be a member of the Board.

(E) Eligibility to be a Reviewer


(a) A Peer Reviewer shall:
 Be a member with at least 10 years of experience in practice.

 Is in Practice as per the Chartered Accountants Act, 1949.


 Should have undergone the requisite training as prescribed by the Board.
 Should furnish a declaration as prescribed by the Board, at the time of acceptance

of Peer Review appointment.


 Should have signed the Declaration of Confidentiality as prescribed by the Board.

 Should have conducted audit of Level I Entities for at least 7 years to be eligible for

conducting Peer Review of Level I Entities


(b) For being a Reviewer a member should not have:
 Disciplinary action/proceedings pending against him

350
FINAL C.A. – AUDIT

 Been found guilty by the Council or the Disciplinary Board or Committee at any
time.
 Been convicted by a Competent Court whether within or outside India, of an offence

involving moral turpitude and punishable with transportation or imprisonment.


 Any Obligation or conflict of interest in the Practice Unit or its Partners/

 Personnel.

III PEER REVIEW PROCESS

Stage I- Selection 1. Notification to PU should be notified of its selection for peer


review.
Name of 3 reviewers shall be recommended to
PU.
Appointment of 2. Selection of The PU shall select(one out of three reviewer &
Reviewer Reviewer intimate to the
Board within 7 days of receipt of the names
Stage II 1. Information to On intimation by the Board, of the Reviewer’s
Planning Be furnished by consent, the PU shall within 15 days furnish
the PU the following information to the Reviewer:
• Duly filled-in Questionnaire sent by the
Board.
• Complete list of assurance service clients
indicating the nature of service provided
and the fees charged.

351
FINAL C.A. – AUDIT

Stage II 1. Information to • A note on the policies and procedures


Planning Be furnished by adopted by the PU in relation to
the PU Independence, Staff Supervision &
Development, ‘Second Person’ Review &
the process generally followed in carrying
out assurance services.
• Details of any proceedings against the
PU or any of its partners or qualified
assistants relating to investigation or
allegation of deficiency in the conduct
of Attest function by them during the
period of 3 years preceding the period of
Review or at any time thereafter i.e. till
the date of submission of the duly filled-
in Questionnaire.
Stage II 2. Selection of • The Reviewer shall within 15 days of
Planning Sample by the receiving the information from the
Reviewer PU select a sample of the assurance
services that he would like to Review and
intimate the same to the Practice Unit.
• The Reviewer may also seek further/
additional clarification from the PU on
the information furnished/not furnished.
Stage II 3. Plan for Initial • The Reviewer shall plan for an on-
Planning Meeting site Review visit or initial meeting in
consultation with the PU. The Reviewer
shall give the PU at least 15 days time
to keep ready the necessary records of
the selected assurance services.
• The Reviewer and PU shall mutually
cooperate and ensure that the entire
Review process is completed within 90
days from the date of notifying the PU
about its selection for Review.

352
FINAL C.A. – AUDIT

Stage 111 The execution stage involves the actual conduct of review and, thus,
Execution Stage begins with initial meeting and ends with review of records by the
reviewer. It should not extend Beyond 7 working days.
1. Initial Meeting An initial meeting shall be held between the
reviewer and the practice unit to confirm the
accuracy of responses to the questionnaire

2. Compliance To begin with, the reviewers expected to carry


Review out the compliance review of the key controls,
• Independence,
• Maintenance of professional skills and
standards,
• Outside consultation,
• Staff selection & supervision and
• Office administration,
to gain an understanding of the working of the
PU and specific control procedures existing at
the PU.
3. Selection of It depend upon:
Attestation • Standard of quality controls generally
Service prevailing.
Engagements • The size and nature of assurance service
engagements undertaken by the PU.
• The methodology generally adopted by the
PU in providing assurance services.
• The number of partners/members involved
in assurance service engagements in the
PU.
• The number of locations/branch offices
of the practice Unit
• The Fees charged/received/service tax
paid by the Practice unit.

353
FINAL C.A. – AUDIT

4. Review of • The reviewer would conduct adequate


Records- compliance procedures to gain evidence
Compliance that those general controls on which
and Substantive the reviewer intends to rely upon,
Approach actually exists and functioning effectively
throughout the period of reliance.
• Based on the results of compliance
procedures, the reviewer concludes either
to rely or not to rely on the general
controls
• In case the reviewer decides to rely on
the general controls he would also need
to determine the extent of reliance to De
placed on such controls.
• In such a situation, the NTE of
substantive procedures would be, normally,
less extensive and vice-versa.
• The substantive approach involves
application of such review procedures that
provide the reviewer with the evidence as
to the appropriateness of the factors on
which the review is required to be focused
on.
Stage IV 1. Preliminary At the end of review, the reviewer is required
Reporting stage in Report of to send a preliminary report to the PU (before
Peer Review Reviewer making any report to the
Board) in case systems and procedures of the
PU have been
found to be deficient or where non-compliance
with reference to any other matter has been
noticed by the reviewer
NO preliminary report is required in case no
deficiencies or non-compliance are noticed by
the reviewer.

354
FINAL C.A. – AUDIT

2. Response to The PU has to send its representations, in


Preliminary writing, to the reviewer, on the areas mentioned
Report in the preliminary report within a period of
115 days from the receipt of the preliminary
report from the reviewer
3. Modified • If the reviewer is not satisfied with
Report the reply of the PU, the reviewer shall
submit a modified report to the Board
incorporating his reasons for the same.
• The Reviewer shall also submit initial
findings (i.e. Preliminary Report),
response by the Practice Unit (Response
to Preliminary Report) and the manner
in which the responses have been dealt
with.
• A copy of the modified report shall also be
forwarded to the PU.
• In case of a modified report, the Board
shall order for a “Follow On” Review after
a period of one year from the date of issue
of modified report. If the Board so decides,
the period of one year may be reduced but
shall not be less than six months from
the date of issue of the report.
4.Peer review At the end of an on-site Review if the Reviewer
Report is satisfied with the
reply received from the PU, he shall submit a
Peer Review Report to the Board along with his
initial findings, response by the Practice Unit
and the manner in which the responses have
been dealt with. Report
A copy of the report shall also be forwarded to
the PU.

355
FINAL C.A. – AUDIT

Stepwise Approach Step 1 – obtain The reviewer should obtain an understanding


adopted by the an understanding of the assignment
Peer Reviewer of assignment so as to plan the review of documentation.
Step 2-Selection Reviewer should ascertain the number of
of assurance assurance engagements to be reviewed based
engage-ments on the evaluation of replies given in the
questionnaire and the size of the practice unit.
Step 3-Examine The reviewer should, wherever possible, examine
the Policies and that the policies and procedures for acceptance
proce-dures of of audit have been complied with and necessary
Practice Unit documentation with regard to the same exists,
Step 4- Review • The reviewer would conduct adequate
and Evaluation compliance procedures to gain evidence
of records that those general controls on which
Compliance the reviewer intends to rely upon,
and Substantive actually exists and functioning effectively
Checking throughout the period of reliance. Based
on the results of compliance procedures,
the reviewer concludes either to rely or
not to rely on the general controls. In case
the reviewer decides to rely on the general
controls, he would also need to determine
the extent of reliance to be placed on
such controls. In such a situation, the
NTE of substantive procedures would be,
normally, less extensive and vice versa.
• The substantive approach involves
application of such review procedures that
provide the reviewer with the evidence as
to the appropriateness of the factors on
which the required to be focused on.
1. Framework
1.1. Quality review is directed towards evaluation of audit quality and adherence to various
statutory and other regulatory requirements.
1.2. It is done under the authority of Quality Review Board.

356
FINAL C.A. – AUDIT

1.3. It is not the purpose to review all of a firm’s audits or to identify every aspect in which
a reviewed audit is deficient. Accordingly, a review should not be understood to provide
any assurance that the firm’s audits, or its clients’ financial statements or reporting
thereon, are free of any deficiencies.
1.4 Presently, the review undertaken by QRB covers statutory audit services only and does
not extend to internal audit services provided by the members of the Institute.
1.5 The Board may constitute one or more Quality Review Groups (hereinafter referred
to as Review Groups) to conduct preliminary reviews of the general purpose financial
statements, with a view to assessing the quality of audit and reporting by the auditors,
in consultation with the Board.
1.6 Each of the Review Group would be assisted by Technical Reviewer(s), who may be an
outsourced service provider.
The job of the Technical Reviewer(s) would be to prepare a report on the review of
general purpose financial statements, with a view to assessing the quality of audit and
reporting by the auditors, and the review of quality control framework adopted by the
auditors/audit firms in conducting audit.

2. Objective & Scope


2.1 Objective:
(a) Compliance with the applicable technical standards in India, other applicable professional
and ethical standards and relevant laws and regulations;
(b) Implementation of a system of quality control with reference to the applicable quality
control standards;
(c) Consideration of SA 240, “The Auditors’ Responsibilities relating to Fraud in an Audit
of Financial Statements” issued by The Institute of Chartered Accountants of India
(ICA); and
(d) Whether there is no material misstatement of assets and liabilities as at the reporting
date in respect of the Company/entity audited by the AFUR.

2.2 Scope:
(a) Examining whether the Engagement Partner has ensured compliance with the applicable
technical standards in India and other applicable professional and ethical standards
and requirements.
(b) Examining whether the Engagement Partner has ensured compliance with the relevant
laws and regulations.
(c) Examining whether the Audit firm has implemented a system of quality control as

357
FINAL C.A. – AUDIT

envisaged in line with the Standard on Quality Control (SQC) 1, Quality Control for
Firms that Perform Audits and Reviews of Historical Financial Information, and Other
Assurance and Related Services Engagements.

3. Technical Standards:
As per the QRB, the term “Technical Standards” in the context of the Chartered Accountants
(Procedures of Meetings of Quality Review Board, and Terms and Conditions of Service and
Allowances of the Chairperson and Members of the Board) Rules, 2006 includes.
3.1 The Accounting Standards notified under section 133 of Act, 2013;
3.2 The Accounting Standards issued by the Institute of Chartered Accountants of India
3.3 The Framework for the Preparation and Presentation of Financial Statements issued by
the Institute of Chartered Accountants India
3.4 The applicable Quality Control and Engagement Standards issued by the Institute of
Chartered Accountants of India
3.5 The Statements on Auditing issued by the Institute of Chartered Accountants of India
3.5 The Statements on Auditing issued by the Institute of Chartered Accountants of India
3.6 The Guidance Notes on accounting and auditing matters issued by the Institute of
Chartered Accountants of India
3.7 The Notifications/Directions/Guidelines issued by the Institute of Chartered Accountants
of India including those of a self-regulatory nature
3.8 The Code of Ethics issued by the Institute of Chartered Accountants of India.

4. Quality Review Board


4.1 The Quality Review Board (hereinafter “QRB”/ “the Board”) has been set up by the Central
Government under section 28A of the Chartered Accountants Act, 1949 (hereinafter
“the Act”)
4.2 In terms of section 28A of the Chartered Accountants Act, 1949, the Board comprises
of a Chairperson and ten other members.
4.3 Sub-section (2) of section 28A of the aforesaid Act requires that the Chairperson and
members of the Board shall be appointed from amongst the persons of eminence having
experience in the field of law, economics, business, finance or accountancy.
4.4 Sub-section (3) of section 28A of the Act provides that five members of the Board shall
be nominated by the Central Government and other five members shall be nominated by
the Council of the Institute of Chartered Accountants of India.
4.5 Functions of QRB :
(a) Review quality of services provided by the members.

358
FINAL C.A. – AUDIT

(b) Make recommendations to the Council for quality of services provided by the
members.
(c) Guide the members to improve the quality of services and adherence to the various
statutory and other regulatory requirements.
4.6 Powers of QRB
(a) On its own or with assistance of the ICAI evaluate and review the quality of the
audit work
(b) Lay down the procedure for evaluation criteria i.e. set the benchmarks for evaluating
quality of the services
(c) Call for information from the ICAI, Council or its Committees, Members, Clients of
Members etc.
(d) Make recommendations to the council to guide the members of the institute to
improve their professional competence and qualifications, quality of work and
services offered.
5. Selection of Audit Firms
5.1 Selection of Audit Firm based on the Financial Statements of the Enterprise/s Audited
by the latter: Under this category, in the initial stage, the audited accounts of companies
having wider public interest, such as listed companies, insurance companies, NBFCs,
unlisted public sector undertakings, asset management companies may be selected by
QRB.
It is done on the basis of one or more of the following:
 Suo moto or random selection from particular class of enterprises/audit firms.
 On account of being a part of a sector otherwise identified as being susceptible to
risk on the basis of market intelligence reports
 Serious accounting irregularities reported in media or other Reports
 Reference made to it by any regulatory body such as Reserve Bank of India,
Securities and Exchange Board of India, Insurance Regulatory and Development
Authority, Ministry of Corporate Affairs, etc.
5.2 Criteria based on Audit Firms Auditing the Accounts: Selection of audit firms may also
be made for review of their work on random basis, the volume of work handled by them
represented by the number and nature of clients, their involvement in sectors that may
be identified as facing high risk, as well as on account of their reported involvement in
fraud or likelihood of fraud. Audit firms auditing large as well as mid – cap/small cap
companies may be selected for the purpose.

359
FINAL C.A. – AUDIT

6. QUALIFICATIONS OF TECHNICAL REVIEWER


6.1 A member should not have disciplinary proceeding under the Chartered Accountants
Act, 1949 pending against him/her or any disciplinar action under the Chartered
Accountants Act, 1949/ penal action under any other law taken/pending against you
during last three financial years and/or thereafter.
6.2 A member or his/her firm or any of the network firms or any of the partners of the firm
or that of the network firms should not have been the statutory auditor of the company,
as specified, or have rendered any other services to the said company/entity during last
three financial years and/or thereafter.
6.3 A member or his/her firm or any of the network firms or any of the partners of the
firm or that of the network firms should not have had any association with the specified
statutory audit firm, during the last three financial years and /or thereafter.
6.4 A member to comply with all the eligibility conditions laid down for appointment as
an auditor of a company u/s 141(3) of the Companies Act, 2013 which apply mutatis
mutandis in respect of the review of the quality of statutory audit of the company/entity,
as specified, so far as applicable.
6.5 Apart from the above, the board decided the following criteria empanelment of technical
reviewer with the board from financial year 2015 – 2016:
 Reviewer should have minimum fifteen years of post qualification experience as a
chartered accountant and be currently active in the practice of accounting and
auditing
 Reviewer should have handled as a signing partner/proprietor at least three
statutory audit assignments as a Central Statutory Auditor of Banks/Public
Limited Companies/Government Companies/Private Limited Companies having
annual turnover of rupees fifty crores and above during the last ten financial years;
Provided that out of the aforesaid three statutory audit assignments, at least one
must be in respect of entities other than Private Limited Companies.
 Reviewer should not have any disciplinary proceeding under the Chartered
Accountants Act, 1949 pending against you or any disciplinary action under the
Chartered Accountants Act, 1949, penal action under any other law taken/pending
against you during last three financial years and/or thereafter.
 Reviewer should not currently be a Member of the QRB or lCAl’s Central Council/
Regional Council/Branch level Management Committee

360
FINAL C.A. – AUDIT

7. Can Technical Reviewer seek help from assistants?


For the purpose of on-site review, they could also take the assistance of not more than
three assistants who:
7.1 Shall be chartered accountant
7.2 Do not attract any of the disqualifications prescribed under the Chartered Accountants
Act, 1949
7.3 Shall also have to sign the statement of confidentiality in a prescribed format
7.4 Shall have no direct interface either with the audit firm under review or the Board.
7.5 Should have been working with them for atleast one year as a member / a partner in the
CA firm with them.
7.6 Should not have been associated with the Statutory auditor / audit firm under review
and the company / entity selected during last three financial years and / or thereafter.

8. QUALITY REVIEW PROCESS


8.1 Selection and appointment:
 Selection of Audit Firm and Technical Reviewer to conduct Quality Review and
sending Offer Letter of Engagement to the Technical Reviewer.
 Technical Reviewer to convey his acceptance of Letter of Engagement by sending
necessary declarations for meeting eligibility conditions and furnishing statement
of confidentiality by the Technical Reviewer and his assistant/s, if any
 Intimation to the Audit Firm about the proposed Quality Review and acceptance of
the assignment by the Technical Reviewer. Also marking a copy of the intimation to
the Technical Reviewer.
8.2 Execution by Technical reviewer:
 Technical Reviewer to send the specified Quality Review Program General Questionnaire
to the Audit firm for filling-up and call for additional information from the Audit
Firm, if required.
 Technical Reviewer to carry out the Quality Review by visiting the office of the Audit
Firm by fixing the date as per mutual consent.
 The technical reviewers for carrying out the quality review assignment, could
undertake a maximum of one on-site visit to the Statutory Audit firm which shall
not extend beyond seven days or, in exceptional circumstances, such other extended
period, for specific reasons to be recorded in writing, with the prior approval of
the Chairperson Quality Review Board, which shall not, in any case, extend beyond
fourteen days.

361
FINAL C.A. – AUDIT

8.3 Reporting stage and subsequent actions:


 Technical Reviewer to send the preliminary report to Audit firm.
 Audit firm to submit representation on the preliminary report to the Technical
Reviewer.
 Technical Reviewer to submit final report along with a copy of Annual report of
the company/entry for the year, to the Board in the specified format, on their
(individual) letterhead, duly signed and dated within 45 days from the date of
acceptance of the assignment.
 Technical Reviewer should also send a copy of their final report to the Statutory
Auditor/Audit firm, requesting the firm to send their submission thereon to the
Board within 7 days for receipt of the final report with a copy to Technical Reviewer.
 Upon receipt of their final submission, Technical Reviewer shall submit within next
7 days a summary of their findings, reply of the audit firm thereon along with their
final comments in the specified format.
 Quality Review Group to consider the report of the Technical Reviewer and responses
of the Audit firm and make recommendations to Quality Review Board.
 Quality Review Board to consider the report of the Quality Review Group and decide
the final course of action.

9. FACTORS AFFECTING QUALITY OF A QUALITY REVIEW


9.1 Knowledge and experience of the technical reviewer.
9.2 Time devoted by the technical reviewer.
9.3 Composition of the quality review team.
9.4 Understanding of the objective and scope of work.
9.5 Monitoring, direction and supervision of the quality review team by the technical reviewer.

362

You might also like